194
© Surgent • cpenow.com 237 Lancaster Avenue Devon, PA 19333 P: 610-688-4477 F: 610-688-3977 [email protected] www.surgent.com Advanced Selected Issues for Trusts, Estates, and Their Beneficiaries ITEB/16/01

Advanced Selected Issues for Trusts, Estates, and Their ... · 28/07/1997  · 3. Availability of §179 expensing 2 C. Administrative stance 2 1. Estates 2 2. Trusts 3 D. Judicial

  • Upload
    others

  • View
    1

  • Download
    0

Embed Size (px)

Citation preview

Page 1: Advanced Selected Issues for Trusts, Estates, and Their ... · 28/07/1997  · 3. Availability of §179 expensing 2 C. Administrative stance 2 1. Estates 2 2. Trusts 3 D. Judicial

© Surgent • cpenow.com

237 Lancaster Avenue Devon, PA 19333P: 610-688-4477F: [email protected]

Advanced Selected Issuesfor Trusts, Estates, andTheir Beneficiaries

ITEB/16/01

Page 2: Advanced Selected Issues for Trusts, Estates, and Their ... · 28/07/1997  · 3. Availability of §179 expensing 2 C. Administrative stance 2 1. Estates 2 2. Trusts 3 D. Judicial
Page 3: Advanced Selected Issues for Trusts, Estates, and Their ... · 28/07/1997  · 3. Availability of §179 expensing 2 C. Administrative stance 2 1. Estates 2 2. Trusts 3 D. Judicial

This product is intended to serve solely as an aid in continuing professional education. Due to the constantly changing nature of the subject of the materials, this product is not appropriate to serve as the sole resource for any tax and accounting opinion or return position, and must be supplemented for such purposes with other current authoritative materials. The information in this manual has been carefully compiled from sources believed to be reliable, but its accuracy is not guaranteed. In addition, Surgent McCoy CPE, LLC, its authors, and instructors are not engaged in rendering legal, accounting, or other professional services and will not be held liable for any actions or suits based on this manual or comments made during any presentation. If legal advice or other expert assistance is required, seek the services of a competent professional. Revised April 2016 cpenow.com / [email protected] Copyright © 2016 Surgent McCoy CPE, LLC -- ITEB/16/01

Table of Contents

Depreciation and Depletion Deductions for Estates and Trusts....................... 1 Charitable Remainder Trusts ................................ 2 The Alternative Minimum Tax for Estates and Trusts .................................... 3 Terminating an Estate or Trust ............................. 4 Generation-Skipping Transfers ............................ 5 Charitable Lead Trusts .......................................... 6 Hot Topics in Fiduciary Taxation ......................... 7 Rate Schedules and Computations ...................... 8 Problems and Tax Forms ...................................... 9 Solutions .............................................................. 10

Page 4: Advanced Selected Issues for Trusts, Estates, and Their ... · 28/07/1997  · 3. Availability of §179 expensing 2 C. Administrative stance 2 1. Estates 2 2. Trusts 3 D. Judicial

NOTES

Page 5: Advanced Selected Issues for Trusts, Estates, and Their ... · 28/07/1997  · 3. Availability of §179 expensing 2 C. Administrative stance 2 1. Estates 2 2. Trusts 3 D. Judicial

cpenow.com / [email protected] Copyright © 2016 Surgent McCoy CPE, LLC -- ITEB/16/01 1-i

Depreciation and Depletion Deductions for Estates and Trusts

Learning objectives 1 I. Background 1

A. Nature of problem 1 1. Estate or trust’s deduction 1 2. Purpose of module 1

B. Statutory guidance 1 1. Depreciation and depletion 1 2. Apportionment between entity and beneficiaries 2 3. Availability of §179 expensing 2

C. Administrative stance 2 1. Estates 2 2. Trusts 3

D. Judicial interpretations 5 1. Apportionments to discretionary income beneficiaries 5 2. Effect of reserve on apportionment 6 3. Effect of charitable contributions when depreciation follows income 7

E. Apportionment of depreciation and depletion during loss or no income years 8 1. In general 8 2. Limiting criteria 9

F. Tax-planning implications 10 1. What is the intent? 10 2. Insuring proper allocation 10 3. Charitable income beneficiaries 10 4. Drafting 10

G. Impact of final regulations on the definition of trust income 17 H. Supplement: The 3.8-percent net investment income tax 20

1. Details 20 2. Adjusted gross income 21 3. Exceptions 21 4. Traditional five steps 22

Page 6: Advanced Selected Issues for Trusts, Estates, and Their ... · 28/07/1997  · 3. Availability of §179 expensing 2 C. Administrative stance 2 1. Estates 2 2. Trusts 3 D. Judicial
Page 7: Advanced Selected Issues for Trusts, Estates, and Their ... · 28/07/1997  · 3. Availability of §179 expensing 2 C. Administrative stance 2 1. Estates 2 2. Trusts 3 D. Judicial

cpenow.com / [email protected] Copyright © 2016 Surgent McCoy CPE, LLC -- ITEB/16/01 1-1

Depreciation and Depletion Deductions for Estates and Trusts

Learning objectives After completing this chapter, you will be able to: • Identify the statutory, administrative, and judicial guidelines affecting the deductibility of

depreciation and depletion by estates and trusts; • Describe the critical elements in several controversial areas of depreciation and depletion

deductions; and • Determine tax-planning strategies for individuals faced with the potential inclusion of

qualified property in the corpus of an estate or trust.

I. Background

A. Nature of problem

1. Estate or trust’s deduction

Even though §§167, 611, and 642 of the Internal Revenue Code entitle an estate or trust to a deduction for depreciation and depletion on qualified property and define limits and exceptions to the laws, fiduciaries are left, in many situations, without precise guides in classifying depreciation and depletion as either allowable or not allowable to an estate or trust.

2. Purpose of module

Consequently, the purpose of this module is twofold. First, we will review the various statutory, administrative, and judicial guidelines on the deductibility of depreciation and depletion by estates and trusts. Second, we will delineate the critical elements in several controversial areas of depreciation and depletion deductions, and, as a result of this analysis, provide tax-planning criteria for individuals faced with the potential inclusion of qualified property in the corpus of an estate or trust.

B. Statutory guidance

1. Depreciation and depletion

An estate or trust may be entitled to deductions for depreciation and depletion if it holds qualified property; see §167(a). Depreciation and depletion are deductible by an estate or trust only to the extent not allowable to beneficiaries.1

a. In the case of an estate, this requires apportionment between the estate and the heirs, legatees, and devisees on the basis of the income of the estate allocable to each.2 As a result, an estate has only a residual claim on depreciation and depletion deductions.

1 I.R.C. §642(e). 2 I.R.C. §§167(h) and 611(b)(3).

Page 8: Advanced Selected Issues for Trusts, Estates, and Their ... · 28/07/1997  · 3. Availability of §179 expensing 2 C. Administrative stance 2 1. Estates 2 2. Trusts 3 D. Judicial

cpenow.com / [email protected] Copyright © 2016 Surgent McCoy CPE, LLC -- ITEB/16/01 1-2

b. For property held in trust, the deductions for depreciation and depletion §§167(h) and 611(b), must be apportioned between the income beneficiaries and the trustee. The extent to which the deductions are allowable to income beneficiaries depends primarily on the pertinent provisions of the instrument creating the trust.3 In the absence of such provisions within the trust instrument and local law, the deduction for depreciation or depletion is apportioned between the fiduciary and the income beneficiaries on the basis of the trust accounting income allocable to each.4

2. Apportionment between entity and beneficiaries

Essentially, §642(e) of Subchapter J cross-references to §§167(h) and 611(b); it provides that “an estate or trust will be allowed the deductions for depreciation and depletion only to the extent not allowable to beneficiaries under §§167(h) and 611(b).” As previously mentioned, §167(h) provides that the depreciation deduction shall be “apportioned between the income beneficiaries and the trustee in accordance with the pertinent provisions of the instrument creating the trust, or, in the absence of such provisions, on the basis of the trust income allocable to each.” Consequently, if the trust instrument is silent and the income beneficiary is entitled to all the trust income, then he or she will receive the entire depreciation deduction. This provision dates back to the Revenue Act of 1928.5 Confusion arises because this old provision was taken over into the 1954 Code, but not correlated with the new distributable net income (DNI) approach adopted at that time. Moreover, §643 defines distributable net income to be taxable income with specified modifications, but depreciation is not designated as a modification in §643. Naturally, taxable income would normally mean income after taking the depreciation deduction. Seemingly, the intent was to keep the depreciation deduction separate and apart from DNI. That is, taxable income in §643 means taxable income before allowance for depreciation.

3. Availability of §179 expensing

Also, it should be noted that estates and trusts do not qualify for the election to expense certain depreciable business assets.6

C. Administrative stance

1. Estates

a. The regulations, promulgated under the 1954 Code, provide that where property is held by an estate the allowable deduction for depreciation is apportioned between the estate and the heirs, legatees, and devisees on the basis of the “income of the estate” allocable to each.7 However, the regulations pertaining to depletion state that the deduction is to be allocated on the basis of the income from the depletable property held by the estate.8 In contrast, the regulations require depreciation to be apportioned on the basis of the state law income of the estate without defining the income as that from the estate’s depreciable property. This incongruency is illustrated in the following scenario.

3 Id. 4 Id. 5 45 Stat. 800. 6 I.R.C. §179. 7 Treas. Regs. §1.167(h)-1(c). 8 Treas. Regs. §1.611-1(b)(3).

Page 9: Advanced Selected Issues for Trusts, Estates, and Their ... · 28/07/1997  · 3. Availability of §179 expensing 2 C. Administrative stance 2 1. Estates 2 2. Trusts 3 D. Judicial

cpenow.com / [email protected] Copyright © 2016 Surgent McCoy CPE, LLC -- ITEB/16/01 1-3

Example: Assume an estate holds a particular mineral property and an apartment rental complex. Further assume that all the income from the mineral property is payable to Beneficiary A, and that rental income from the apartments is divided equally between three remaining beneficiaries, B, C, and D. In this situation, if the regulations are applied literally, Beneficiary A would receive benefit of the entire depletion deduction and his pro rata share of the depreciation deduction apportioned on the basis of the total income of the estate even though he received no income from the depreciable property. Beneficiaries B, C, and D would receive their pro rata portion of the depreciation deduction, but since they are not entitled to income from the depletable property, no portion of the depletion deduction is allocated to them.

b. It seems that a more equitable allocation of the depreciation deduction could be achieved

if the estate’s income from depreciable property allocable to either the estate or the beneficiaries is used as the basis of apportionment. In this manner, those beneficiaries entitled to income from a depreciable property would receive the benefit of the entire depreciation deduction attributable to that property, even though other estate income is payable to other beneficiaries. This method of apportionment seems to be a more logical interpretation of the code and regulations. Nonetheless, the literal language of the code and regulations requiring apportionment of depreciation on the basis of the income of the estate does not support this interpretation.

c. If an estate, as a member of a partnership or as an income beneficiary of a trust, is entitled to a portion of the depreciation or depletion deduction allowable to the partnership or the trust, its distributive share of such deductions must be apportioned between the recipient estate and its heirs, legatees, and devisees in the same manner set forth in the regulations.9 The allowable deduction is limited to property owned by the estate in its capacity as a separate taxable person. Although the depreciation deduction is apportioned on the basis of the income allocable to each of the estate beneficiaries, the amount allocated may be greater than the beneficiaries’ pro rata share of the income of the estate.10 That is, the beneficiaries’ depreciation deductions are not limited to the amount of their share of the estate’s income.

2. Trusts

In the case of a trust, the deduction for depreciation and depletion otherwise allowable must be apportioned between the income beneficiaries, including charitable beneficiaries,11 and the trustee in accordance with the pertinent provisions of the trust instrument or, in the absence of such terms, on the basis of the income allocable to each.12 The depreciation deduction may be taken by the trust only when the trustee is directed by local law or by the trust instrument to retain income in a depreciation reserve or a depletion reserve for the purpose of preserving corpus.13 In such a case, the deduction is first allocated to the trustee to the extent that income is set aside for the reserve. The excess is then apportioned between the income beneficiaries and the trustee on the basis of trust income (in excess of the income set aside for the reserve) allocable to each.14

9 Rev. Rul. 74-71, 1974-1 C.B. 158; Rev. Rul. 66-278, 1966-2 C.B. 243; Rev. Rul. 61-211, 1961-2 C.B. 124. 10 Rev. Rul. 74-530, 1974-2 C.B. 188. 11 Treas. Regs. §1.642(e)-1. 12 I.R.C. §§1.167(h) and 611(b)(3); Treas. Regs. §§1.167(h)-1(b) and 1.611-1(c)(4). 13 Treas. Regs. §§1.167(h)-1(b) and 1.611-1(c)(4). 14 Id.

Page 10: Advanced Selected Issues for Trusts, Estates, and Their ... · 28/07/1997  · 3. Availability of §179 expensing 2 C. Administrative stance 2 1. Estates 2 2. Trusts 3 D. Judicial

cpenow.com / [email protected] Copyright © 2016 Surgent McCoy CPE, LLC -- ITEB/16/01 1-4

a. The significance of a reserve for depletion or depreciation is that the trustee may claim the entire deduction to the exclusion of income beneficiaries, provided the trustee sets aside an amount to the reserve that is within deductible depreciation limits.15 If the amount authorized or required as a reserve is greater than the allowable deduction, under federal law the trustee may claim no deduction for the excess. On the other hand, if the allowable deduction is greater than the required or authorized credit to the reserve, the excess allowable deduction over the amount set aside is allocated in accordance with the general rule of apportionment.

Example: Assume a trust receives gross income of $500,000 from rent of an office building,

and that depreciation for the year amounts to $100,000. If the trust instrument does not require or authorize a reserve for depreciation and the trustee is specifically directed to distribute the entire $500,000 equally among five beneficiaries, the apportionment of depreciation would be as follows:

Fiduciary’s portion of depreciation -- $0 / $500,000 x $100,000 = $0 Each beneficiary’s portion of depreciation -- $100,000 / $500,000 x $100,000 = $20,000

If the trust instrument required or authorized the trustee to set aside a yearly reserve for depreciation of $60,000, the deduction would be apportioned in the following manner:

Total deductible depreciation $100,000 Less: depreciation reserve $ 60,000 Balance to be apportioned $ 40,000 Fiduciary’s portion of depreciation -- Depreciation reserve $ 60,000 Each beneficiary’s portion of depreciation -- $88,000 / $440,000 x $40,000 = $ 8,000

As depicted in the example, a required or authorized reserve for depreciation or depletion allows the trustee to deduct an amount greater than his pro rata share of trust income. This is the only instance where the regulations permit a taxpayer to claim a larger amount as a deduction than his pro rata share of the trust income. Any allocation of depreciation or depletion in excess of the pro rata share will to that extent be disregarded, regardless of any provisions within the trust instrument.16

b. A trust, as a member of a partnership or as an income beneficiary of another trust, may be entitled to a portion of the depreciation or depletion deduction allowable to the partnership or other trust. Such deductions may be apportioned between the recipient trust and its income beneficiaries in the manner set forth in the regulations.17 Although the deductions are apportioned on the basis of income allocable to each of the income beneficiaries, the amount allocable may be greater than the beneficiaries’ pro rata share of trust income.18

15 Treas. Regs. §§1.167(h)-1(b) and 1.611-1(c)(4). 16 Treas. Regs. §§1-167(h)-1(b) and 1.611-1(c)(4). 17 Rev. Rul. 74-71, 1974-1 C.B. 158; Rev. Rul. 66-278, 1966-2 C.B. 243; Rev. Rul. 61-211, 1961-2 C.B. 124. 18 Rev. Rul. 74-530, 1974-2 C.B. 188.

Page 11: Advanced Selected Issues for Trusts, Estates, and Their ... · 28/07/1997  · 3. Availability of §179 expensing 2 C. Administrative stance 2 1. Estates 2 2. Trusts 3 D. Judicial

cpenow.com / [email protected] Copyright © 2016 Surgent McCoy CPE, LLC -- ITEB/16/01 1-5

Example: Depreciation Allocation for a Complex Trust

A complex trust had the following items of income for the taxable year: Dividends $16,000 Taxable interest 10,000 Exempt interest 10,000 Rent 4,000 Long-term capital gain allocable to corpus 6,000 Total income $46,000

The trust also had expenses as follows: Expenses allocable to rental income $2,000 Commissions allocable to taxable income 3,000 Commissions allocable to corpus 1,500 Depreciation of rental property 2,500 Total expenses $9,000

The trust instrument has no provision for the maintenance of a depreciation reserve, and requires the trustee to make the following distributions:

Income paid to charity $ 3,000 Income paid to Beneficiary A 15,000 Income paid to Beneficiary B 5,000 Total distributions $23,000

The allocation of depreciation is computed in the following manner: Total trust income $46,000 Less: long-term capital gains 6,000 Gross state law income $40,000 Less: Expenses and commissions

allocated to income

5,000 Net state law income $35,000 Less: Distributions 23,000 Income allocable to fiduciary $12,000 Fiduciary’s portion of depreciation: $12,000 / $35,000 x $2,500 = $ 857 Beneficiary A’s portion of depreciation: $15,000 / $35,000 x $2,500 = $1,071 Beneficiary B’s portion of depreciation: $ 5,000 / $35,000 x $2,500 = $ 357 Charitable beneficiary’s portion of depreciation: $ 3,000 / $35,000 x $2,500 = $ 215 Total depreciation deduction $2,500

D. Judicial interpretations

1. Apportionments to discretionary income beneficiaries As previously stated, depreciation and depletion are apportioned between the estate and its heirs, legatees, and devisees on the basis of income of the estate allocable to each party. Contradictory findings in two separate cases have left a cloud of uncertainty over the apportionment of deductions to discretionary beneficiaries of an estate.

Page 12: Advanced Selected Issues for Trusts, Estates, and Their ... · 28/07/1997  · 3. Availability of §179 expensing 2 C. Administrative stance 2 1. Estates 2 2. Trusts 3 D. Judicial

cpenow.com / [email protected] Copyright © 2016 Surgent McCoy CPE, LLC -- ITEB/16/01 1-6

a. The first opinion, rendered by the Fourth Circuit Court of Appeals, reversed the findings of the Tax Court,19 which held that the deductions must be apportioned under §167(g) of the Internal Revenue Code of 1954.20 The Appellate Court stated that “the words of statutes -- including revenue acts -- should be interpreted where possible in their ordinary, everyday senses.”21 In legal terminology and in ordinary, everyday acceptation, the words “heirs,” “legatees,” and “devisees” do not include discretionary distributees of estate income. Consequently, no part of the deduction for depreciation could properly be apportioned to them, and under §642(e) the estate was entitled to take the entire allowable deduction.22

b. In W.H. Lamkin, the Fifth Circuit Court of Appeals23 rejected the rationale of Nissen.24 Supposedly, the Fifth Circuit looked to the substance of the transaction over the form. As a consequence, the court viewed the distribution of income to a discretionary beneficiary as no different than a distribution to any other income beneficiary. By resolving the issued in this manner, the court adopted a uniform application of trust and estate-tax law so that income coming from an estate and being distributed to a discretionary beneficiary would be treated no differently than either of the two alternatives provided by §167(h). Therefore, in estates involving distributions of income to discretionary beneficiaries, one should scrutinize the intent of the estate agreement so that the more favorable ruling regarding apportionment of depreciation may be applied to accomplish the taxpayer’s desired goal.

2. Effect of reserve on apportionment a. The Upton Case -- Petitioners Anna L.S. Upton and Margaret St. Aubyn were income

beneficiaries of a testamentary trust created by the will of their father, William R. Sloan.25 Most of the income of the trust consisted of oil royalties. The petitioners contended that neither the will of the decedent, the decree of final distribution, nor the California Court Decree of May 19, 1953 made any provision for the apportionment of depletion allowances and, in accordance with the statute, “in the absence of such provisions” the allowable deduction “shall be apportioned on the basis of trust income allocable to each.”26 The Commissioner argued that if the trust instrument directs the trustee to make an allowance for keeping the trust corpus intact by retaining a reserve of current income for that purpose, the allowable deduction should be granted in full to the trustee.27 The court agreed with this reasoning and held that the permissible deduction for depletion was allowable in full solely to the trustees.

b. Spalding Trust Case28 -- The issue in the case was the depreciation deduction by the trustee where the trust instrument provided for distribution of the entire trust income from depreciable property held by the trust. The court examined the terms of the trust instrument itself to determine the intention of the trustor on this point. However, the trust instrument made no provision as to depreciation of trust assets and did not authorize or require the trustee to set aside an amount as a reserve for depreciation. The instrument

19 Nissen Estate v. Commissioner, 41 T.C. 522 (1965). 20 I.R.C. §167(g) was redesignated 167(h) by §13(c)(1) of the Revenue Act of 1962, P.L. 87-834. 21 See Hanover Bank v. Commissioner, 369 U.S. 672 (1962). 22 Nissen Estate v. Commissioner, 345 F.2d 230 (4th Cir. 1965), rev’g. 41 T.C. 522 (1965). 23 Lamkin v. United States, 75-1 U.S.T.C. ¶ 9479 (W.D. Tex. 1975). 24 Nissen Estate v. Commissioner, 345 F.2d 230 (4th Cir. 1965), rev’g and remd’g 41 T.C. 522 (1964). 25 Upton v. Commissioner, 32 T.C. 301 (1959), affd. 283 F.2d 716 (9th Cir., 1960). 26 I.R.C. §§167(h) and 611(b)(3). 27 Treas. Regs. §§1.167(h)-1(b)(2) and 1.611-1(b)(3). 28 Spalding Trust, 34 B.T.A. 762 (1936).

Page 13: Advanced Selected Issues for Trusts, Estates, and Their ... · 28/07/1997  · 3. Availability of §179 expensing 2 C. Administrative stance 2 1. Estates 2 2. Trusts 3 D. Judicial

cpenow.com / [email protected] Copyright © 2016 Surgent McCoy CPE, LLC -- ITEB/16/01 1-7

only contained provisions for the distribution of the “entire annual rents and profits” from the property after deducting related expenses. Additionally, the trust instrument specifically indicated that it was the purpose of the trust to provide income for the beneficiaries, and that the life beneficiaries otherwise were to have no interest in the corpus of the trust. In light of these facts, the court held that, in the absence of a provision in the trust instrument authorizing maintenance of a reserve, the beneficiaries who receive the entire net trust income are entitled to the entire deduction.

Note:

In such cases, the trustee is left without any deduction for depreciation, and the income beneficiary is entitled to the entire depreciation or depletion deduction.

3. Effect of charitable contributions when depreciation follows income

a. The Code and regulations require apportionment of depreciation and depletion deductions between the trustee and the income beneficiaries.29 One must avoid reading into Code §167(h) the word “taxable” before the words “income beneficiaries.” Such a reading has been rejected by the courts.30 The inherent unfairness of such an interpretation was pointed out in the Lambert case where it was observed:

If such a position (of excluding charitable beneficiaries from the term “income beneficiaries”) is correct, the settlor of a trust might convey 95 percent of the income to charities and five percent to an individual and such an individual, by deducting depreciation on the entire property, might escape taxes altogether. To justify such an inequitable conclusion would require a very clear mandate of the statute.31

In the court’s opinion, such an interpretation was a clear distortion of congressional intent:

It is the plaintiff’s position that the term “income beneficiaries” as here used32 refers to taxable income beneficiaries, and that since charitable beneficiaries are not taxable, it applies at bar only to the individual beneficiaries. A careful reading [of §167(h)] shows otherwise.

The term “income beneficiaries” in the first part [of the section] is carried forward with the same meaning into the last expression “on the basis of the trust income allocable to each.” [Thus], since the instrument did not provide for a [depreciation] reserve, the charitable organizations bore the loss that was occasioned by depreciation in the form of subsequently reduced income. It seems very clear from this provision that plaintiff was entitled only to the depreciation allocable to his interest, and not to his interest plus a corresponding part of the depreciation attributable to the interest of the charitable beneficiaries.33

29 I.R.C. §§167(h) and 611(b)(3); Treas. Regs. §1.167(h)-1(b). 30 Grey v. Commissioner, 41 B.T.A. 234 (1940), aff’d. 118 F2d 153 (7th Cir., 1941); Scott v. U.S. 78 F. Supp. 811 (Ct. Cl.

1948); Lambert Tree Trust Est. v. Commissioner, 38 T.C. 392 (1962). 31 See Scott, supra note 30, 78 F. Supp. at 815. 32 I.R.C. §167(h). 33 Scott, supra note 30, 78 F. Supp. at 815.

Page 14: Advanced Selected Issues for Trusts, Estates, and Their ... · 28/07/1997  · 3. Availability of §179 expensing 2 C. Administrative stance 2 1. Estates 2 2. Trusts 3 D. Judicial

cpenow.com / [email protected] Copyright © 2016 Surgent McCoy CPE, LLC -- ITEB/16/01 1-8

b. Clearly then, where the trust instrument makes no specific provision as to deduction for depreciation on the trust property, the deduction must follow the distribution of the income, whether the distribution is made to taxable or charitable income beneficiaries. The following example illustrates this point.

Example: A trustee is required to distribute one-half of the trust income to charitable

organizations and one-half to an individual beneficiary. The records of the fiduciary show the following items of income and deduction for the taxable year:

Nontaxable income (gross) $ 5,200 Taxable income (gross) 85,900 Depreciation 16,550 Deductible expenditures 11,600

The state law income of the trust for fiduciary accounting purposes is $79,500, computed as follows:

Nontaxable income (gross) $ 5,200 Taxable income (gross) 85,900 Total $ 91,100 Less: Deductible expenditures $ 11,600 Income as computed under §643(b) $ 79,500 Distributable to charitable organizations (50 percent) $ 39,750 Distributable to individual beneficiary (50 percent) $ 39,750

Since no provision was made in the trust instrument with regard to depreciation, the deduction is allocated as follows:

Deduction allocable to charitable organizations (50 percent)

$ 8,275

Deduction allocable to individual beneficiary (50 percent)

$ 8,275

It is obvious that 50 percent of the depreciation deduction was wasted in this scenario. As a result, it becomes essential to design the trust instrument to allocate the depreciation to either the trust or the taxable income beneficiaries.

E. Apportionment of depreciation and depletion during loss or no income years

1. In general

The allowable deductions34 for depreciation and depletion are computed by an estate or trust in its capacity as a separate taxable entity under §7701. As a result, the deductions for depreciation and/or depletion35 must first be computed by the estate or trust based on the properties it holds in its capacity as a separate taxable entity. Also, it is possible to allocate depreciation and depletion deductions between an estate and its heirs, legatees, and devisees or between a trust and its beneficiaries in amounts that are greater than their pro rata shares of the income of the estate or trust. This result is possible because

34 I.R.C. §§167(h) and 611(b)(3). 35 I.R.C. §§167(h) and 611(b)(3).

Page 15: Advanced Selected Issues for Trusts, Estates, and Their ... · 28/07/1997  · 3. Availability of §179 expensing 2 C. Administrative stance 2 1. Estates 2 2. Trusts 3 D. Judicial

cpenow.com / [email protected] Copyright © 2016 Surgent McCoy CPE, LLC -- ITEB/16/01 1-9

the depreciation and depletion deductions are apportioned on the basis of the income of the estate or trust allocable to each of the parties (without regard to any depreciation or depletion allocable to them). Consequently, the beneficiaries are not limited by the amount of such income.36

a. This concept was previously elaborated in the case of Sue Carol.37 The court interpreted the language of the section38 enumerating the various kinds of deductions from gross income as wholly general in character and employing the broadest terms in designating such deductions. The court felt that:

None of these allowable deductions are restricted to income from the property or obligations to which they relate. If the expenses of a trade or business exceed the receipts therefrom, the surplus amount may be included in the taxpayer’s return as a deduction from other income. Similarly, the indebtedness on which interest is allowed as a deduction is not required to be connected with income-producing property or a trade or business. “Taxes generally,” with specific exceptions, are deductible regardless of the nature of the assessment or its relation, if any, to income. Thus, while depreciation may emanate from certain property, the allowance of the deduction therefore is not limited to the income produced by that property.39

b. In light of these decisions, the heirs, legatees, and devisees of an estate or the

beneficiaries of a trust are entitled to the benefit of their pro rata share of the deduction, even in a year in which the property yields no income. Thus, the deductions may be used to offset income or profits derived from any other source.

2. Limiting criteria

One must avoid misinterpreting the concept of a beneficiary receiving depreciation and depletion in excess of their pro rata shares of income from the estate or trust. That is, it should be emphasized that depreciation and depletion deductions allocated to beneficiaries may not exceed their proportion of such deductions as determined by the estate or trust income allocable to each beneficiary.

a. This point of not allowing disproportionate allocations was dramatically illustrated in the Dusek40 case. The relevant facts were as follows:

Raymond Dusek contributed over $100,000 to a trust created in favor of his wife Velma. These funds were used for the purchase of rental property. During the tax years in question, the net income of the trust ranged from $5,100 to $15,800, and in the same period depreciation ranged from $4,500 to $11,000. In each year, the trustee distributed only $100 to Velma and, at the same time, allocated to her all of the federal income-tax deductions for depreciation.41

The Tenth Circuit repudiated this disproportionate allocation of deductions as contrary to the intent of Congress in enacting §23(R) of the Revenue Act of 1928. This intent is illustrated as follows:

36 Rev. Rul. 74-530, 1974-2 C.B. 188. 37 30 B.T.A. 443 (1934). 38 §23 of the Revenue Act of 1924; I.R.C. §62. 39 Sue Carol, 30 B.T.A. 448 (1934). 40 376 F. 410 (10th Cir., 1967), affd. 45 T.C. 355 (1961). 41 Id.

Page 16: Advanced Selected Issues for Trusts, Estates, and Their ... · 28/07/1997  · 3. Availability of §179 expensing 2 C. Administrative stance 2 1. Estates 2 2. Trusts 3 D. Judicial

cpenow.com / [email protected] Copyright © 2016 Surgent McCoy CPE, LLC -- ITEB/16/01 1-10

The purpose [of the section] was to recognize that the equitable interest of a trust beneficiary, as reflected in the trust income allocable to him, is sufficient to entitle him to claim the depreciation deduction to the extent of the income so allocable. It would be strange indeed to construe the section so as to permit, in the case of a trust beneficiary, a complete separation of the income and the deduction.42

b. It is quite clear that the depreciation and/or depletion deduction allocated to a beneficiary

may exceed the amount of that particular beneficiary’s share of trust income. On the other hand, it is equally clear that the depreciation and/or depletion deduction may not be apportioned in a percentage greater than his or her pro rata share of trust income.

F. Tax-planning implications

1. What is the intent?

To insure fulfillment of the manifest intentions of the decedent of an estate or the grantor of a trust, involving depreciation and depletion deductions, one should engage in careful fiduciary tax planning. The extent to which depreciation or depletion deductions are apportioned between an estate or trust and its beneficiaries depends primarily on the provisions of the instrument creating the estate or trust and, absent direction in the instrument, then state law. Accordingly, the pertinent terms of the instrument can be definitive in order to accomplish the desired apportionment.

2. Insuring proper allocation

Tax planners must insure that the deductions flow to the proper parties. If the primary purpose of a trust is to preserve corpus, provision for the requirement or authority of a trustee to maintain a reserve for depreciation or depletion must be specified. In contrast, a trust designed to benefit the income beneficiaries should specify apportionment of the deduction to those beneficiaries exclusively.

3. Charitable income beneficiaries

Special care should be taken if the trust instrument requires the distribution of income to charitable beneficiaries. Unless otherwise provided, the deductions will follow income so that a portion of the depreciation or depletion is wasted on an exempt entity. Avoidance of such waste can easily be accomplished by designing the instrument to allocate depreciation and depletion deductions to the trust in the form of a reserve.

4. Drafting Practitioners must also be aware of the implications of the Nissen43 and Lamkin44 cases as they apply to the wording contained in the governing instrument of an estate or trust. The Nissen decision, rendered by the Fourth Circuit Court of Appeals, held that the words of a legal document must be interpreted by their literal meaning. On the other hand, the Fifth Circuit Court, in Lamkin, rejected the rationale of literal interpretation and looked to the substance of a transaction over the form in determining the meaning of provisions within an estate or trust instrument. Although the reasoning in Nissen seems more logical, one should apply the decision most favorable to their client in accomplishing the inherent purpose of the estate or trust agreement. Obviously, tax planners within the Fourth and Fifth Circuits must abide by the decisions in Nissen and Lamkin, respectively.

42 Id. 43 Nissen Estate v. Commissioner, (4th Cir., 1965) 345 F. 2d 230. 44 Lamkin v. Commissioner, 75-1 U.S.T.C. ¶9479 (W.D. Tex. 1975).

Page 17: Advanced Selected Issues for Trusts, Estates, and Their ... · 28/07/1997  · 3. Availability of §179 expensing 2 C. Administrative stance 2 1. Estates 2 2. Trusts 3 D. Judicial

cpenow.com / [email protected] Copyright © 2016 Surgent McCoy CPE, LLC -- ITEB/16/01 1-11

Five-Step Approach to Understanding the Form 1041’s Depreciation Reserve Scenario (Based on Treas. Regs. §1.661(c)(2))

Under the terms of a complex trust, $10,000 per year is required to be paid out of income to a designated charity. The balance of the income may, in the trustee’s discretion, be accumulated or distributed to Beneficiary X. Expenses are allocable against income and the trust instrument requires a reserve for depreciation. During the taxable year, the trustee contributes $10,000 to charity and in his discretion distributes $15,000 of income to X. The trust has the following items of income and expenses for the taxable year.

Dividends $10,000 Interest 10,000 Fully tax-exempt interest 10,000 Rents 20,000 Rental expenses 2,000 Depreciation of rental property 3,000 Trustee’s commissions 5,000

Step I - State Law Income -- The state law income of the trust for fiduciary accounting purposes is $40,000, computed as follows:

Dividends $10,000 Interest 10,000 Fully tax-exempt interest 10,000 Rents 20,000 Total $50,000 Deductions: Rental expenses $2,000 Depreciation 3,000 Trustee’s commissions 5,000 10,000 Income as computed under §643(b) $40,000 (Line 8 of Sch. B)

Page 18: Advanced Selected Issues for Trusts, Estates, and Their ... · 28/07/1997  · 3. Availability of §179 expensing 2 C. Administrative stance 2 1. Estates 2 2. Trusts 3 D. Judicial

cpenow.com / [email protected] Copyright © 2016 Surgent McCoy CPE, LLC -- ITEB/16/01 1-12

Step II - §643(a) Ceiling -- The distributable net income of the trust as computed under §643(a) is $30,000, determined as follows: Rents $20,000 Dividends 10,000 Interest 10,000 Fully tax-exempt interest $10,000 Less: Expenses allocable thereto (10,000/50,000 x $5,000) $1,000 Charitable contributions allocable thereto (10,000/50,000 x $10,000) 2,000 3,000 7,000

Total $47,000 Deductions: Rental expenses $2,000 Depreciation of rental property 3,000 Trustees’ commissions ($5,000 less $1,000 allocated to tax exempt interest) 4,000 Charitable contributions ($10,000 less $2,000 allocated to tax-exempt interest) 8,000 17,000 Distributable Net Income (§643(a)) $30,000 (Line 7 of Sch. B) Step III - §661(a) Ceiling -- The deduction ceiling allowable to the trust under §661(a) for distributions to Beneficiary X is $11,500, computed as follows:

$15,000 Distribution to X Less: (3,500) Tax-exempt $11,500 Trust’s deduction for Beneficiary X (Line 15 of Sch. B)

Step IV - Trust’s Income -- The taxable income of the trust is computed as follows:

Rental income $20,000 Dividends 10,000 Interest 10,000 GROSS INCOME $40,000 Deductions: Rental expenses $2,000 Depreciation of rental property 3,000 Trustee’s commissions 4,000 Charitable contributions 8,000 Distributions to Beneficiary X 11,500 Personal exemption 100 $28,600 TAXABLE INCOME (Line 22) $11,400

In computing the taxable income of the trust, no deduction is allowable for the portions of the charitable contributions deduction ($2,000) and trustee’s commissions ($1,000), which are treated under §661(b) as attributable to the tax-exempt interest excludable from gross income.

Page 19: Advanced Selected Issues for Trusts, Estates, and Their ... · 28/07/1997  · 3. Availability of §179 expensing 2 C. Administrative stance 2 1. Estates 2 2. Trusts 3 D. Judicial

cpenow.com / [email protected] Copyright © 2016 Surgent McCoy CPE, LLC -- ITEB/16/01 1-13

Step V - Inclusion and Characterization to Beneficiary X -- It is assumed that the trustee elected to allocate the trustee’s commissions to rental income except for the amount required to be allocated to tax-exempt interest. In the absence of specific provisions in the trust instrument for the allocation of different classes of income, the charitable contribution is deemed to consist of a pro rata portion of the gross amount of each item of income of the trust; the trust is deemed to have distributed to X a pro rata portion (one-half) of each item of income included in distributable net income.

Rental income

Taxable Dividends

Interest Tax-Exempt Interest

TOTAL

Trust income $20,000 $10,000 $10,000 $10,000 $50,000 Less: Charitable contributions 4,000 2,000 2,000 2,000 10,000 Rental expenses 2,000 2,000 Depreciation 3,000 3,000 Trustee’s commissions 4,000 1,000 5,000 Total deductions 13,000 2,000 2,000 3,000 20,000 Distributable net income 7,000 8,000 8,000 7,000 30,000

Depreciation Follows Income and How to Always Get

Correct K-1 Amounts (Based on Treas. Regs. §1.662(c)-4) Under the terms of a testamentary trust, one-half of the trust income is to be distributed currently to W, the decedent’s wife, for life. The remaining trust income may, in the trustee’s discretion, either be paid to D, the grantor’s daughter, paid to designated charities, or accumulated. The trust is to terminate at the death of W and the principal will then be payable to D. No provision is made in the trust instrument with respect to depreciation of rental property. Capital gains are allocable to corpus under the applicable local law. The trust and both beneficiaries file returns on the calendar-year basis. The records of the fiduciary show the following items of income and deduction for the taxable year.

Rents $50,000 Dividends of domestic corporations 50,000 Tax-exempt interest 20,000 Taxable interest 10,000 Capital gains (long-term) 20,000 Depreciation of rental property 10,000 Expenses attributable to rental income 15,400 Trustee’s commissions allocable to income account 2,800 Trustee’s commissions allocable to principal account 1,100

Page 20: Advanced Selected Issues for Trusts, Estates, and Their ... · 28/07/1997  · 3. Availability of §179 expensing 2 C. Administrative stance 2 1. Estates 2 2. Trusts 3 D. Judicial

cpenow.com / [email protected] Copyright © 2016 Surgent McCoy CPE, LLC -- ITEB/16/01 1-14

Step I - State Law Income The state law income of the trust for fiduciary accounting purposes is $111,800, computed as follows.

Rents $50,000 Dividends 50,000 Tax-exempt interest 20,000 Taxable interest 10,000 TOTAL $130,000 Deductions: Rental expenses $15,400 Trustee’s commissions allocable to income account 2,800 18,200 Income as computed under §643(b) $111,800 (Line 8 of

Sch. B) (Note: depreciation and long-term capital gain are not included.)

The trustee distributed one-half of the state law income ($55,900) to W and, in his discretion, makes a contribution of one-quarter ($27,950) to Charity X and distributes the remaining one-quarter ($27,950) to D. The total of the distributions to beneficiaries is $83,850, consisting of: (i) income required to be distributed currently to W of $55,900; and (ii) other amounts properly paid or credited to D of $27,950. Step II - §643(a) Ceiling The distributable net income of the trust as computed under §643(a) is $82,750, determined as follows.

Rents $50,000 Dividends 50,000 Taxable interest 10,000 Tax-exempt interest 20,000 Less: Trustee’s commissions allocable to tax-exempt interest (20,000/130,000 of $3,900) $600 Charitable contributions allocable to tax-exempt interest (20,000/130,000 of $27,950) 4,300 4,900 15,100 TOTAL $125,100 Deductions: Rental expenses $15,400 Trustee’s commissions ($3,900 less $600 allocated to tax-exempt interest) 3,300 Charitable deduction ($27,950 less $4,300 attributable to tax-exempt interest) 23,650 42,350 DISTRIBUTABLE NET INCOME $82,750 (Line 7 of Sch. B)

Page 21: Advanced Selected Issues for Trusts, Estates, and Their ... · 28/07/1997  · 3. Availability of §179 expensing 2 C. Administrative stance 2 1. Estates 2 2. Trusts 3 D. Judicial

cpenow.com / [email protected] Copyright © 2016 Surgent McCoy CPE, LLC -- ITEB/16/01 1-15

Step III - §661(a) Ceiling The distributable net income of $82,750 as determined under §643(a) is less than the sum of the amounts distributed to W and D of $83,850. Therefore, the deduction allowable to the trust under §661(a) is such distributable net income as modified under §6618 to exclude therefrom the items of income not included in gross income of the trust. The following computation shows the effects:

Distributable net income $82,750 Less: Tax-exempt interest (as adjusted for expenses and the charitable contributions) 15,100 Deduction allowable under §661(a) $67,650 (Line 15 of Sch. B)

Step IV - Trust’s Income The taxable income of the trust is $19,900 determined as follows:

Rental income $50,000 Dividends 50,000 Taxable interest 10,000 Capital gains 20,000 GROSS INCOME $130,000 Deductions: Rental expenses $15,400 Trustee’s commissions 3,300 Charitable contributions 23,650 Distributions to beneficiaries

67,650

Personal exemption 100 $110,100 TAXABLE INCOME $19,900 (Line 22)

Page 22: Advanced Selected Issues for Trusts, Estates, and Their ... · 28/07/1997  · 3. Availability of §179 expensing 2 C. Administrative stance 2 1. Estates 2 2. Trusts 3 D. Judicial

cpenow.com / [email protected] Copyright © 2016 Surgent McCoy CPE, LLC -- ITEB/16/01 1-16

Step V - Inclusion and Characterization to Beneficiaries For the purpose of determining the character of the amounts deductible under §§642(c) and 661(a), the trustee elected to offset the trustee’s commissions (other than the portion required to be allocated to tax-exempt interest) against the rental income. The determination of the character of the amounts deemed distributed to beneficiaries and contributed to charity is made as follows:

Character of Amounts Distributed to Beneficiaries

and Contributed to Charity Under §661(a)

Rents Taxable Dividends

Tax-Exempt Interest

Taxable Interest

Total

Trust income $50,000 $50,000 $20,000 $10,000 $130,000 Less: Charitable contributions 10,750 10,750 4,300 2,150 27,950 Rental expenses 15,400 15,400 Trustee’s commissions 3,300 ______ 600 _____ 3,900 Total deductions 29,450 10,750 4,900 2,150 47,250 Amounts distributable to beneficiaries

20,550

39,250

15,100

7,850

82,750

The character of the charitable contribution is determined by multiplying the total charitable contribution ($27,950) by a fraction consisting of each item of trust income, respectively, over the total trust income, except that no part of the dividends excluded from gross income is deemed included in the charitable contribution. For example, the charitable contribution is deemed to consist of rents of $10,750 (50,000/130,000 x $27,950). Because the wife is a first-tier beneficiary and DNI is $82,750, the whole $55,900 is fully includable in her income. Thus, the character of the $55,900 is determined as follows:

Total each class total DNI

x Includable amount

= K-1 amounts

Rents $20,550 $82,750

x $55,900 = $13,882

Dividends $39,250 $82,750

x $55,900 = $26,515

Taxable interest

$7,850 $82,750

x $55,900 = $5,303

Tax-exempt interest

$15,100 $82,750

x $55,900 = $10,200

TOTAL = $55,900 Additionally, she may deduct a share of the depreciation deduction proportionate to the trust income allocable to her; that is, one-half of the total depreciation deduction, or $5,000. Since the sum of the amount of income required to be distributed currently to W ($55,900) and the other amounts properly paid, credited, or required to be distributed to D ($27,950) exceeds the distributable net income ($82,750) of the trust as determined under §643(a), D is deemed to have received $26,850

Page 23: Advanced Selected Issues for Trusts, Estates, and Their ... · 28/07/1997  · 3. Availability of §179 expensing 2 C. Administrative stance 2 1. Estates 2 2. Trusts 3 D. Judicial

cpenow.com / [email protected] Copyright © 2016 Surgent McCoy CPE, LLC -- ITEB/16/01 1-17

($82,750 less $55,900) for income-tax purposes. The character of the amounts deemed distributed to her is determined as follows:

Rents $20,550 $82,750

x $26,850 = $6,668

Dividends $39,250 $82,750

x $26,850 = $12,735

Taxable interest

$7,850 $82,750

x $26,850 = $2,547

Tax-exempt interest

$15,100 $82,750

x $26,850 = $4,900

TOTAL = $26,850 Also, D may deduct a share of the depreciation deduction proportionate to the trust income allocable to her; that is, one-fourth of the total depreciation deduction, or $2,500.

Caution:

In the above illustration, no provision was made in the trust instrument with respect to depreciation. Therefore, the remaining $2,500 of the depreciation deduction is allocated to the amount distributed to Charity X, and as a result, is nondeductible by the trust, W, or D.

Thus, it would behoove tax planners to make sure that depreciation does not follow income under like circumstances. This can be easily accomplished by a trust because the trust instrument can provide for a reserve for depreciation and/or depletion.

G. Impact of final regulations on the definition of trust income Final regulations have been issued to modify/redefine the accounting income of a trust.45 These new rules are necessary to reflect the changing state laws based on the Uniform Principal and Income Act of 1997. Specifically, Treas. Regs. §1.643(b)-1 delineates the following attributes of accounting income:

For purposes of subparts A through D, part I, subchapter J, chapter 1 of the Internal Revenue Code, income, when not preceded by the words “taxable,” “distributable net,” “undistributed net,” or “gross,” means the amount of income of an estate or trust for the taxable year determined under the terms of the governing instrument and applicable local law. Trust provisions that depart fundamentally from traditional principles of income and principal, that is, allocating ordinary income to income and capital gains to principal, will generally not be recognized. However, amounts allocated between income and principal pursuant to applicable local law will be respected if local law provides for a reasonable apportionment between the income and remainder capital gains, and appreciation. For example, a state law that provides for the income beneficiary to receive each year a unitrust amount of between three percent and five percent of the annual fair market value of the trust assets is a reasonable apportionment of the total return of the trust. Similarly, a state law that permits the trustee to make equitable adjustments between income and principal to fulfill the trustee’s duty of impartiality between the income and remainder beneficiaries is generally a reasonable apportionment of the total return of the trust. These adjustments are permitted when the trustee invests and manages the trust assets under the state’s prudent-investor standard, the trust describes the amount that shall or must be distributed to a beneficiary by referring to the trust’s income, and the trustee, after applying the state statutory rules regarding allocation of income and principal, is unable to administer the trust impartially. In addition, an allocation of capital gains to income will be respected if the allocation is made either pursuant to the terms of the governing instrument and local law, or pursuant to a

45 T.D. 91202.

Page 24: Advanced Selected Issues for Trusts, Estates, and Their ... · 28/07/1997  · 3. Availability of §179 expensing 2 C. Administrative stance 2 1. Estates 2 2. Trusts 3 D. Judicial

cpenow.com / [email protected] Copyright © 2016 Surgent McCoy CPE, LLC -- ITEB/16/01 1-18

reasonable and consistent exercise of a discretionary power granted to the fiduciary by local law or by the governing instrument, if not inconsistent with local law.

The gist of the above regulation is three-pronged. First, a unitrust interest of at least three percent per year satisfies the income requirement. Second, an adjustment power, such as that contained in §104 of the Uniform Principal and Income Act, will also satisfy that requirement. Third, capital gains are included in accounting income and DNI if the fiduciary, pursuant to a discretionary power granted by local law or by the governing instrument (when not inconsistent with local law), treats the capital gains as distributed to a beneficiary, provided the power is exercised in a reasonable and consistent manner. Thus, if a trustee exercises a discretionary power by treating any distribution in excess of ordinary income as being made from realized capital gains, any capital gain so distributed is included in DNI and accounting income.

DNI Computations Under the 1997 Act and/or the Instrument Allowing Capital Gains to be Distributed to the Current Income Beneficiaries

Pursuant to the terms of a simple trust all of the income is to be distributed equally to beneficiaries A and B and capital gains are allocated to the income beneficiaries pursuant to the trustee discretion under state law. A reserve is maintained for depreciation by the fiduciary. During 2015, the trust had the following items of income and expenses. Rents $50,000 Dividends of domestic corporations (Qualifying) $50,000 Tax-exempt interest on municipal bonds $50,000 Long-term capital gains $50,000 Taxes and expenses directly attributable to rents $10,000 Trustee's commissions allocable to income account $4,000 Trustee's commissions allocable to principal account $4,000 Depreciation $10,000 Step 1 : State Law Income / Trust Accounting Income Rents $50,000 Dividends of domestic corporations $50,000 Tax-exempt interest on municipal bonds $50,000 Long-term capital gains $50,000 $200,000 Deductions: Depreciation $10,000 Expenses directly attributable to rental Income $10,000 Trustee's commissions allocable to income account $4,000 $24,000 State law income computes under Code Sec. 643(b) Line 8….......... ……………….. $176,000 Step 2 : Code Sec. 643(a) Ceiling Rents $50,000 Dividends $50,000 Long-term capital gains $50,000 Tax-exempt $50,000

Page 25: Advanced Selected Issues for Trusts, Estates, and Their ... · 28/07/1997  · 3. Availability of §179 expensing 2 C. Administrative stance 2 1. Estates 2 2. Trusts 3 D. Judicial

cpenow.com / [email protected] Copyright © 2016 Surgent McCoy CPE, LLC -- ITEB/16/01 1-19

Less: Expenses allocable thereto (50,000/200,000 * 8000) $2,000 $48,000 $198,000 Deductions: Depreciation $10,000 Expenses directly attributable to rental income $10,000 Trustee's commissions (8,000-2,000 allocable to tax-exempt interest) $6,000 $26,000 Line 7….......... ……………….. $172,000 Step 3 : Code Sec. 651(a) Ceiling Distributable net income computed under Code Sec. 643 (a) $172,000 Less: Tax-exempt interest as adjusted $48,000 Distributable net income (deduction ceiling) Line 15.............. ……………….. $124,000 Step 4 : Trust's Taxable Income Rents $50,000 Dividends $50,000 Long-term capital gains $50,000 Gross income $150,000 Deductions

Depreciation $10,000 Rental Expenses $10,000 Trustee's commissions $6,000 Distributions to beneficiaries $124,000 Personal exemption for a simple trust $300 $150,300

Taxable income Line 22…........... ……………….. $0 Step 5 : Inclusion and Characterization of Beneficiaries

Rents Dividends

Long-term Capital

gain

Tax-exempt Interest Total

Income for trust acctg purposes $50,000 $50,000 $50,000 $50,000 $200,000 Less:

Depreciation $10,000 $10,000 Rental Expenses $10,000 $10,000 Trustee's Commission $2,000 $2,000 $2,000 $2,000 $8,000

Total Deductions $22,000 $2,000 $2,000 $2,000 $28,000 Character of amount in the hands of the beneficiaries $28,000 $48,000 $48,000 $48,000 $172,000 K-1 Amounts for X and Y 14,000 – Line 7 – Rents 24,000 – Line 2b – Qualifying Dividends 24,000 – Line 4a – Long-term Capital Gains 24,000 – Line 14A – Tax-exempt

Page 26: Advanced Selected Issues for Trusts, Estates, and Their ... · 28/07/1997  · 3. Availability of §179 expensing 2 C. Administrative stance 2 1. Estates 2 2. Trusts 3 D. Judicial

cpenow.com / [email protected] Copyright © 2016 Surgent McCoy CPE, LLC -- ITEB/16/01 1-20

H. Supplement: The 3.8-percent net investment income tax There’s a lot for taxpayers to know when it comes to the 3.8-percent net investment income tax (“3.8% NIIT”). This new tax is imposed on income from several sources and its effects are far reaching. Analyzing its impact can get complicated fast. Originating as a component of 2010 health care legislation and first effective in 2013, the 3.8% NIIT is assessed on the lesser of net investment income (NII) or modified adjusted gross income (MAGI) above specified thresholds. MAGI is adjusted gross income plus any excluded net foreign earned income. The MAGI thresholds are $200,000 for single individuals; $250,000 for joint filers and surviving spouses; and $125,000 for married taxpayers filing separate returns. Only individuals with some amount of NII, and MAGI above the applicable threshold amount, will be subject to the 3.8% NIIT. For example, if a married couple has $200,000 of wage income and $100,000 of interest and dividend income (i.e., MAGI totaling $300,000), the 3.8% NIIT applies to the $50,000 that is over the $250,000 MAGI threshold. Trust and estates can also be hit with the 3.8% NIIT. But for them, the tax applies to the lesser of their undistributed net investment income or AGI in excess of the threshold for the top trust federal income tax bracket. For 2015, that threshold is only $12,300 (up to $12,400 in 2016), so many trusts and estates will no doubt be affected. The components of NII generally include gross income from interest, dividends, royalties, and rents; gross income from a trade or business involving passive activities; and net gain from the disposition of property (other than property held in a trade or business in which the owner materially participates). All of these components are reduced by any allocable deductions. This may sound simple, but as always, the devil is in the details.

1. Details

For purposes of the 3.8-percent net investment income tax on the undistributed net investment income of estates and trusts (§1411(c)), net investment income (NII) means the excess, if any, of:

a. The sum of: (i) Gross income from interest, dividends, annuities, royalties, and rents (collectively

“Sec. 1411(c)(1)(A)(i) income”), unless those items are derived in the ordinary course of a trade or business to which the NIIT doesn’t apply;

(ii) Other gross income derived from a trade or business to which the NIIT applies (“other passive or trading income”);46 and

(iii) Net gain (to the extent taken into account in computing taxable income) attributable to the disposition of property other than property held in a trade or business to which the NIIT doesn’t apply.47

b. The allowable deductions that are properly allocable to that gross income or net gain.48 The NIIT applies to a trade or business if it is:

a. A passive activity of the taxpayer, within the meaning of Code §469;49 or

46 Sec. 1411(c)(1)(A)(ii). 47 Sec. 1411(c)(1)(A)(iii). 48 Sec. 1411(c)(1)(B). 49 Sec. 1411(c)(2)(A).

Page 27: Advanced Selected Issues for Trusts, Estates, and Their ... · 28/07/1997  · 3. Availability of §179 expensing 2 C. Administrative stance 2 1. Estates 2 2. Trusts 3 D. Judicial

cpenow.com / [email protected] Copyright © 2016 Surgent McCoy CPE, LLC -- ITEB/16/01 1-21

b. A trade or business of trading in financial instruments or commodities (trading activity), as defined in Code §475(e)(2).50

Practice Note:

Therefore, when an estate or trust doesn’t engage in a passive activity or a financial instrument or commodities trading business, “net investment income” will include only non-business income from interest, dividends, annuities, royalties, rents, and capital gains, minus the allocable deductions. Business income isn’t included. If a trust or estate does engage in a passive activity or a financial instrument or commodities trading business, “net investment income” will include the above items, plus the gross income (minus allocable deductions) from the passive activity or trading business. In general, rental activities are treated as passive activities. However, rental real estate activities of certain “real estate professionals” aren’t automatically passive. That is, a taxpayer is a “real estate professional” for a particular tax year if: (1) more than half of the personal services the taxpayer performs during that year are performed in real property trades or businesses in which the taxpayer materially participates; and (2) the taxpayer performs more than 750 hours of services during that year in real property trades or businesses in which he/she materially participates.

2. Adjusted gross income Code §67(e) defines AGI for an estate or trust as the AGI of an individual with three differences. The first difference is the deduction allowed for expenses which would not have been incurred but for the existence of the trust (e.g., trustee’s fees). The second difference is distributions to beneficiaries of simple and complex trusts. The third difference is the deduction allowed in lieu of a personal exemption. As a result, the AGI of an estate or trust is equal to total income less: fiduciary fees; attorney fees; accountant fees; return preparer fees; other deductions not subject to the two-percent AGI floor; the distribution deduction (DD); and the exemption.

3. Exceptions The following trusts are not subject to the net investment income tax:51

a. A trust all of the unexpired interests in which are devoted to one or more charitable purposes.

b. A trust exempt from tax under §501 (tax-exempt trusts including qualified plans and IRAs).

c. A charitable remainder trust subject to special rules regarding the treatment of annuity or unitrust distributions from such trust to individuals subject to the net investment income tax.

d. Any other trust, fund, or account that is statutorily exempt from income taxes such as MSAs, HSAs, §529 plans, and Coverdell trusts.

e. A trust, or a portion thereof, that is treated as a grantor trust. However, in the case of any such trust or portion thereof, each item of income or deduction that is included in computing taxable income of a grantor or another person shall be treated as if it had been received by, or paid directly to, the grantor or other person for purposes of calculating such person’s net investment income.

50 Sec. 1411(c)(2)(B). 51 Prop. Regs. §1.1411-3(b).

Page 28: Advanced Selected Issues for Trusts, Estates, and Their ... · 28/07/1997  · 3. Availability of §179 expensing 2 C. Administrative stance 2 1. Estates 2 2. Trusts 3 D. Judicial

cpenow.com / [email protected] Copyright © 2016 Surgent McCoy CPE, LLC -- ITEB/16/01 1-22

f. Generally, a foreign trust.

4. Traditional five steps

Under the terms of a testamentary trust, one-half of the trust income is to be distributed currently to W, the decedent’s wife, for life. The remaining trust income may, in the trustee’s discretion, either be paid to D, the grantor’s daughter, paid to designated charities, or accumulated. The trust is to terminate at the death of W and the principal will then be payable to D. A provision for a depreciation reserve is made in the trust instrument with respect to depreciation of rental property. Capital gains are allocable to corpus under the instrument. The trust and both beneficiaries file returns on the calendar year basis.

The records of the fiduciary show the following items of income and deduction for the taxable year:

Rents $50,000 Dividends of domestic corporations 50,000 Taxable interest 10,000 Capital gains (long-term) 20,000 Depreciation of rental property 10,000 Expenses attributable to rental income 15,000 Trustee’s commissions allocable to income account 2,000 Trustee’s commissions allocable to principal account 2,000

Step I – State Law Income or Trust Acc. Inc. The state law income of the trust for fiduciary accounting purposes is $83,000, computed as follows:

Rents $50,000 Dividends 50,000 Taxable interest 10,000 TOTAL $110,000

Deductions:

Rental expenses $15,000 Reserve for depreciation 10,000 Trustee’s commissions allocable to income account 2,000 27,000 Income as computed under §643(b) $83,000 (Line 8 of Sch. B) The trustee distributed one-half of the state law income ($41,500) to W and, in his discretion, makes a contribution of one-quarter ($20,750) to Charity X and distributes the remaining one-quarter ($20,750) to D. The total of the distributions to beneficiaries is $62,250, consisting of: (1) income required to be distributed currently to W of $41,500; and (2) other amounts properly paid or credited to D of $20,750.

Page 29: Advanced Selected Issues for Trusts, Estates, and Their ... · 28/07/1997  · 3. Availability of §179 expensing 2 C. Administrative stance 2 1. Estates 2 2. Trusts 3 D. Judicial

cpenow.com / [email protected] Copyright © 2016 Surgent McCoy CPE, LLC -- ITEB/16/01 1-23

Step II – Sec. 643(a) Income Ceiling The distributable net income of the trust as computed under §643(a) is $60,250, determined as follows:

Rents $50,000 Dividends 50,000 Taxable interest 10,000 TOTAL $110,000 Deductions: Rental expenses $15,000 Trustee’s commissions 4,000 Reserve for depreciation 10,000 Charitable deduction 20,750 49,750 Distributable Net Income $60,250 (Line 7 of Sch. B)

Step III – Sec. 661(a) Deduction Ceiling The distributable net income of $60,250 as determined under §643(a) is less than the sum of the amounts distributed to W and D of $62,250. Therefore, the deduction allowable to the trust under §661(a) is such distributable net income as determined under §643(a) of $60,250. This amount goes on line 15 of Sch. B. Of course, if there had been tax-exempt income, then additional adjustments (as adjusted for expenses and the charitable contributions) would have been made to the DNI of $60,250.

Step IV – Trust’s Income The taxable income of the trust is $19,900, determined as follows:

Rental income $50,000 Dividends 50,000 Taxable interest 10,000 Capital gains 20,000 GROSS INCOME $130,000

Deductions: Rental expenses $15,000 Reserve for depreciation 10,000 Trustee’s commissions 4,000 Charitable contributions 20,750 Distributions to beneficiaries 60,250 Personal exemption 100 $110,100 TAXABLE INCOME $19,900

(Line 22)

Page 30: Advanced Selected Issues for Trusts, Estates, and Their ... · 28/07/1997  · 3. Availability of §179 expensing 2 C. Administrative stance 2 1. Estates 2 2. Trusts 3 D. Judicial

cpenow.com / [email protected] Copyright © 2016 Surgent McCoy CPE, LLC -- ITEB/16/01 1-24

Step V – Inclusion and Characterization to Beneficiaries For the purpose of determining the character of the amounts deductible under §642(c) and §661(a), the trustee elected to offset the trustee’s commissions against the rental income. The determination of the character of the amounts deemed distributed to beneficiaries and contributed to charity is made as follows:

Character of Amounts Distributed to Beneficiaries and Contributed to Charity under §661(A)

Rents Taxable Dividends Taxable Interest Total

Trust income $50,000 $50,000 $10,000 $110,000 Less:

Charitable contributions 9,431 9,431 1,888 20,750 Rental expenses 15,000

$15,000

Trustee’s commissions 4,000

$4,000 Depreciation 10,000

$10,000

Total Deductions 38,431 9,431 1,888 49,750

Amounts distributable

to beneficiaries $11,569 40,569 8,112 60,250

The character of the charitable contribution is determined by multiplying the total charitable contribution ($20,750) by a fraction consisting of each item of trust income, respectively, over the total trust income. For example, the charitable contribution is deemed to consist of rents of $9,431 (50,000/110,000 X $20,750). Because W is a first-tier beneficiary and DNI is $60,250, the whole $41,500 is fully includable in her income. Thus, the character of the $41,500 is determined as follows:

Total each class X Includable = K-1 Total DNI amount Amounts Rents $11,569 X $41,500 = $7,969 $60,250 Dividends $40,569 X $41,500 = $27,944 $60,250 Taxable interest $8,112 X $41,500 = $5,588 $60,250 TOTAL = $41,500

Since the sum of the amount of income required to be distributed currently to W ($41,500) and the other amounts properly paid, credited, or required to be distributed to D ($20,750) exceeds the distributable net income ($60,250) of the trust as determined under §643(a), D is deemed to have received $18,750

Page 31: Advanced Selected Issues for Trusts, Estates, and Their ... · 28/07/1997  · 3. Availability of §179 expensing 2 C. Administrative stance 2 1. Estates 2 2. Trusts 3 D. Judicial

cpenow.com / [email protected] Copyright © 2016 Surgent McCoy CPE, LLC -- ITEB/16/01 1-25

($60,250-41,500) for income tax purposes. The character of the amounts deemed distributed to her is determined as follows:

Rents $11,569 X $18,750 = $3,600 60,250 Dividends $40,569 X $18,750 = $12,625 60,250 Taxable interest $8,112 X $18,750 = $2,525 $60,250 TOTAL = $18,750

Net Investment Income Tax for Estates and Trusts

The previous scenario and the facts thereunder will be used for our illustrations. The trust has the following items for 2015: Rental income $50,000 Dividends 50,000 Taxable interest 10,000 Capital gains (LT) 20,000 Depreciation reserve 10,000 Trustee commissions 4,000

(allocated 50% to income and 50% to principal) Rental expenses 15,000 Charitable contributions 20,750 Step I – Compute traditional DNI as an income ceiling Rents $50,000 Dividends 50,000 Taxable interest 10,000 Less: Rent expenses (15,000) Depreciation (10,000) Charitable contributions (20,750) Trustee commissions (4,000) DNI $60,250

Page 32: Advanced Selected Issues for Trusts, Estates, and Their ... · 28/07/1997  · 3. Availability of §179 expensing 2 C. Administrative stance 2 1. Estates 2 2. Trusts 3 D. Judicial

cpenow.com / [email protected] Copyright © 2016 Surgent McCoy CPE, LLC -- ITEB/16/01 1-26

Step II – Calculate UNII

Rental income $50,000 Dividends 50,000 Taxable interest 10,000 Capital gains (LT) 20,000 Rental expenses (15,000) Depreciation (10,000) Trustee fees (4,000) Charitable deduction (20,750) Distribution deduction (limited to DNI) (60,250) UNII $20,000

Step III – Calculate actual NII tax

Net rental income $25,000 (50,000-15,000-10,000) Dividends 50,000 Taxable interest 10,000 Capital gains (LT) 20,000 Trustee fees (4,000) Distribution deduction (60,250) Exemption (100) AGI 40,650 Less: Highest bracket amount – 2015 12,300 Excess 28,350

NII tax–3.8% X 20,000……………………………………………………………$760 (3.8% X 20,000, which is the lesser of UNII or excess of AGI over highest bracket amount)

For an illustration of the filled-in net investment income tax form for estates and trusts, see the accompanying Form 8960.

Page 33: Advanced Selected Issues for Trusts, Estates, and Their ... · 28/07/1997  · 3. Availability of §179 expensing 2 C. Administrative stance 2 1. Estates 2 2. Trusts 3 D. Judicial

cpenow.com / [email protected] Copyright © 2016 Surgent McCoy CPE, LLC -- ITEB/16/01 1-27

Page 34: Advanced Selected Issues for Trusts, Estates, and Their ... · 28/07/1997  · 3. Availability of §179 expensing 2 C. Administrative stance 2 1. Estates 2 2. Trusts 3 D. Judicial
Page 35: Advanced Selected Issues for Trusts, Estates, and Their ... · 28/07/1997  · 3. Availability of §179 expensing 2 C. Administrative stance 2 1. Estates 2 2. Trusts 3 D. Judicial

cpenow.com / [email protected] Copyright © 2016 Surgent McCoy CPE, LLC -- ITEB/16/01 2-i

Charitable Remainder Trusts

Learning objectives 1 I. Introduction 1

A. Background 1 1. Uses 1 2. Planning requirements 1

B. Impact of 1997 Act on charitable remainder trusts 1 1. Maximum payout rate 1 2. Minimum remainder value 1

II. Operation of the trust 2 A. Features and requirements 2

1. Taxation of the trust 2 2. Distributions 2 3. Tax character of distributions 2 4. Governing instrument 4 5. Annuity or unitrust requirement 5 6. Permissible income recipients 5 7. Exclusive function 6 8. Trustee 6 9. Charitable deduction 6 10. Computation of the present value of the remainder interest for an annuity trust 7 11. Computation of the present value of the remainder interest for a unitrust 8 12. Limitations 8 13. Tax consequences for the grantor 9 14. Charitable remainder trusts: pro rata division of trusts by State Court 9

B. Planning with charitable remainder trusts 10 1. Avoiding capital-gains tax on appreciated investment property 10 2. Funding for a college education 11 3. Pension alternative 11 4. Sale of an interest in a CRAT or CRUT 11

III. Conclusion 12

Page 36: Advanced Selected Issues for Trusts, Estates, and Their ... · 28/07/1997  · 3. Availability of §179 expensing 2 C. Administrative stance 2 1. Estates 2 2. Trusts 3 D. Judicial
Page 37: Advanced Selected Issues for Trusts, Estates, and Their ... · 28/07/1997  · 3. Availability of §179 expensing 2 C. Administrative stance 2 1. Estates 2 2. Trusts 3 D. Judicial

cpenow.com / [email protected] Copyright © 2016 Surgent McCoy CPE, LLC -- ITEB/16/01 2-1

Charitable Remainder Trusts

Learning objectives

After completing this chapter, you will be able to: • Explain the charitable remainder trust (CRT) qualifying requirements; • Determine the charitable deduction for gift and income-tax purposes; • Explain how to avoid capital-gains tax on the sale of appreciated investment property;

and • Describe how to reduce estate taxes with CRTs.

I. Introduction

A. Background

1. Uses

An excellent income and estate-tax planning technique, the charitable remainder trust can: • Provide a current charitable deduction; • Avoid capital-gains tax on the sale of appreciated investment property, thereby increasing

the amount available for investment; • Increase cash flow; • Reduce estate taxes; • Maintain (or possibly increase) the amount of assets passing to one’s heirs; and • Achieve all of the above while satisfying the donor’s charitable desires.

2. Planning requirements

To realize these benefits, however, careful planning is required to comply with the highly technical rules that govern charitable remainder trusts. This module will review the technical requirements and general features of charitable remainder trusts and illustrate planning opportunities.

B. Impact of 1997 Act on charitable remainder trusts

1. Maximum payout rate

Two restrictions have been imposed in connection with charitable remainder trusts. First, for transfers in trust after June 18, 1997, the trust will not qualify as a charitable remainder trust if the required annual payout to the income beneficiary is greater than 50 percent of the initial fair market value of the trust assets, in the case of a charitable remainder annuity trust, or 50 percent of the annual value of the trust assets, in the case of a charitable remainder unitrust.

2. Minimum remainder value

The second restriction is a requirement that the value of the remainder interest of a charitable remainder trust must be at least 10 percent of the value of the property transferred to the trust. This requirement is effective for transfers made after July 28, 1997.

Page 38: Advanced Selected Issues for Trusts, Estates, and Their ... · 28/07/1997  · 3. Availability of §179 expensing 2 C. Administrative stance 2 1. Estates 2 2. Trusts 3 D. Judicial

cpenow.com / [email protected] Copyright © 2016 Surgent McCoy CPE, LLC -- ITEB/16/01 2-2

II. Operation of the trust

A. Features and requirements

1. Taxation of the trust

A qualified charitable remainder trust is exempt from income tax except in any year that it has unrelated business taxable income1 or fails to make a required payment to a noncharitable beneficiary prior to the due date for the trust’s Form 5227, including extensions.2

2. Distributions

The trust must provide for annual (or more frequent) payments to one or more noncharitable beneficiaries. The payment period must be either for the lives of the individual beneficiaries (all of whom must be living at the time the trust is created) or for a term of years, not in excess of 20 years. After termination of the noncharitable interests, the remaining trust assets must be either held in trust for, or paid over to, charity.3 Payments to the income beneficiaries are specified in one of two ways, which determines the type of charitable remainder trust.

a. In a charitable remainder annuity trust, the payments must be a sum certain,4 which is not less than five percent of the initial net fair market value of all property placed in trust.5

b. In a charitable remainder unitrust, the payments must equal a fixed percentage (which cannot be less than five percent) of the net fair market value of the trust, valued annually.6 As an option, the unitrust’s governing instrument may provide that the income beneficiary receive only trust income not in excess of the stated percentage (net income unitrust).7 With this option, the trust may provide that any prior deficiency will be made up when trust income exceeds the stated percentage (net income with makeup unitrust).8 A deficiency arises in years in which the trust income is less than the amount determined by application of the stated percentage.

c. No amount other than the annuity or unitrust amount can be paid to a person other than a charitable organization.9 Such payments, or the possibility of such payments, can disqualify the trust.10 This does not prohibit the trust, however, from making a transfer to a noncharitable person in exchange for full and adequate consideration.11

3. Tax character of distributions

The tax character of the annuity or unitrust payments received by the noncharitable beneficiary is determined by the rules of §664(b) and the regulations thereunder. These rules treat the payments as coming from one or more “tiers” of trust income or corpus. Payments are treated as being made first from

1 I.R.C. §664(c); Treas. Regs. §1.664-l(a)(l)(i). See Newhall Unitrust v. Commissioner, 104 T.C. No. 10 (1995). 2 Treas. Regs. §§1.664-2(a)(1)(I) and 1.664-3(a)(1)(i)(a). 3 I.R.C. §§664(d)(1) and (2). 4 Treas. Regs. §§1.664-2(a)(1)(ii) and (iii) provide a stated dollar amount expressed either as an absolute amount or as a

fraction or percentage of the initial fair market value of the trust property. 5 I.R.C. §664(d)(l)(A). 6 I.R.C. §664(d)(2)(A). 7 I.R.C. §664(d)(3)(A). 8 I.R.C. §664(d)(3)(B). 9 I.R.C. §§664(d)(1)(B) and (2)(B); Treas. Regs. §§1.664-2(a)(4) and 1.664-3(a)(4). 10 Treas. Regs. §1.664-l(a)(6), Exs. (3), (4), and (5). In Rev. Rul. 82-128, 1982-2 C.B. 71, the trust was not qualified if it was

possible that federal estate and state death taxes would be paid from trust assets. In PLR 7802106, the Service noted that the trust would not be qualified if the donor continued to live rent-free on the real estate that was to be transferred to the trust because that right would be a payment of other than the annuity or unitrust amount to a noncharitable organization.

11 Treas. Regs. §§1.664-2(a)(4) and 1.664-3(a)(4).

Page 39: Advanced Selected Issues for Trusts, Estates, and Their ... · 28/07/1997  · 3. Availability of §179 expensing 2 C. Administrative stance 2 1. Estates 2 2. Trusts 3 D. Judicial

cpenow.com / [email protected] Copyright © 2016 Surgent McCoy CPE, LLC -- ITEB/16/01 2-3

ordinary income, second from capital gains, third from other income (including tax-exempt income), and finally from trust corpus. Current and previously undistributed income from each tier must be exhausted in the above order before income earned in the next tier is considered distributed.12

a. The Service has issued proposed regulations to revise the rules for characterizing a distribution to take into account differences in the federal income-tax rates applicable to items of income that are assigned to the same category. As before, the trust's income is assigned, in the year it is required to be taken into account by the trust, to one of three categories: the ordinary-income category, the capital-gains category, or the “other” income category; in addition, within the ordinary-income and capital-gains categories, items are also assigned to different classes based on the federal income-tax rate applicable to each type of income in the category. Within the ordinary-income and capital-gains categories, income is treated as distributed from the classes of income in that category beginning with the class subject to the highest federal income-tax rate and ending with the class subject to the lowest federal income tax rate. The proposed regulations also provide rules for netting different classes of capital gains and losses based on prior guidance.13 In general, the provisions are applicable for taxable years ending after November 20, 2003.

b. A trust's income, including income includable in gross income and other income, is assigned to one of three categories in the year in which it is required to be taken into account by the trust. These categories are as follows: • Gross income, other than gains and amounts treated as gains from the sale or

other disposition of capital assets (referred to as the ordinary-income category); • Gains and amounts treated as gains from the sale or other disposition of capital

assets (referred to as the capital-gains category); and • Other income.14

c. Items within the ordinary-income and capital-gains categories are assigned to different classes based on the federal income-tax rate applicable to each type of income in that category in the year the items are required to be taken into account by the trust. For example, the ordinary-income category may include a class of qualified dividend income and a class of all other ordinary income. In addition, the capital-gains category may include separate classes for short-term capital gains and losses, for 28-percent rate gain, for unrecaptured §1250 gain, and for all other long-term capital gains and losses.15

d. After items are assigned to a class, the tax rates may change so that items in two or more classes would be taxed at the same rate if distributed during a particular year. If the changes to the tax rates are permanent, the undistributed items in those classes are combined into one class. If, however, the changes to the tax rates are only temporary (for example, the new rate for one class will sunset in a future year), the classes are kept separate.

e. The categories and classes of income are used to determine the character of an annuity or unitrust distribution from the trust in the hands of the recipient regardless of whether the trust is exempt from taxation under §664(c) for the year of the distribution. The determination of the character of amounts distributed shall be made as of the end of the taxable year of the trust. The recipient is taxed on the distribution based on the tax rates applicable in the year of the distribution to the classes of income that are deemed

12 I.R.C. §664(b); Treas. Regs. §1.664-l(d)(l). 13 Notice 97-59, 1997-2 C.B. 309. 14 Prop. Regs. §1.664-1(d)(1)(i)(a). 15 Prop. Regs. §1.664-1(d)(1)(i)(b).

Page 40: Advanced Selected Issues for Trusts, Estates, and Their ... · 28/07/1997  · 3. Availability of §179 expensing 2 C. Administrative stance 2 1. Estates 2 2. Trusts 3 D. Judicial

cpenow.com / [email protected] Copyright © 2016 Surgent McCoy CPE, LLC -- ITEB/16/01 2-4

distributed from the trust. The character of the distribution in the hands of the annuity or unitrust recipient is determined by treating the distribution as being made from each category in the following order:16 • First, from ordinary income to the extent of the sum of the trust's ordinary income

for the taxable year and its undistributed ordinary income for prior years; • Second, from capital gain to the extent of the trust's capital gains determined

below; • Third, from other income to the extent of the sum of the trust's other income for

the taxable year and its undistributed other income for prior years; and • Finally, from trust corpus (with corpus defined for this purpose as the net fair

market value of the trust assets less the total undistributed income (but not loss) in paragraphs (d)(1)(i)(a)(1) through (3) of Prop. Regs. §1.664-1)).

f. If the trust has different classes of income in the ordinary-income category, the distribution from that category is treated as being made from each class, in turn, until exhaustion of the class, beginning with the class subject to the highest federal income-tax rate and ending with the class subject to the lowest federal income-tax rate.17 (i) If the trust has different classes of net gain in the capital-gains category, the

distribution from that category is treated as being made first from the short-term capital-gain class and then from each class of long-term capital gain, in turn, until exhaustion of the class, beginning with the class subject to the highest federal income-tax rate and ending with the class subject to the lowest rate.18

(ii) If two or more classes within the same category are subject to the same current tax rate, but at least one of those classes will be subject to a different tax rate in a future year (for example, if the current rate sunsets), the order of that class in relation to other classes in the category with the same current tax rate is determined based on the future rate or rates applicable to those classes.

(iii) Within each category, if there is more than one type of income in a class, amounts treated as distributed from that class are to be treated as consisting of the same proportion of each type of income as the total of the current and undistributed income of that type bears to the total of the current and undistributed income of all types of income included in that class. For example, if rental income and interest income are subject to the same current and future federal income-tax rate and therefore are in the same class, a distribution from that class will be treated as consisting of a proportional amount of rental income and interest income.

4. Governing instrument

A written governing instrument must create a valid irrevocable trust under applicable local law,19 incorporate any applicable requirements of local law relating to charitable trusts,20 and comply with §664

16 Prop. Regs. §1.664-1(d)(1)(ii)(a). 17 Prop. Regs. §1.664-1(d)(1)(ii)(b). 18 These rules require long-term capital gains to be distributed in the following order: first, 28-percent-rate gain; second,

unrecaptured §1250 gain; and then, all other long-term capital gains are applicable for taxable years ending on or after December 31, 1998.

19 Rev. Proc. 89-20, 1989-1 C.B. 841, §3. 20 However, provisions of state law that are inconsistent with the requirements set forth by I.R.C. §664 and the regulations

thereunder should be expressly negated by the governing instruments. See, e.g., Rev. Rul. 77-58, 1977-1 C.B. 175 that disqualified a trust because of its failure to negate state law that allowed the trustee in certain circumstances to invade the trust for the benefit of the grantor and the income beneficiary.

Page 41: Advanced Selected Issues for Trusts, Estates, and Their ... · 28/07/1997  · 3. Availability of §179 expensing 2 C. Administrative stance 2 1. Estates 2 2. Trusts 3 D. Judicial

cpenow.com / [email protected] Copyright © 2016 Surgent McCoy CPE, LLC -- ITEB/16/01 2-5

and the regulations thereunder.21 As a result, it must provide that, upon the termination of the last income interest, an irrevocable transfer of the trust corpus be made to charity (either paid to or held in trust).22

a. The governing instrument cannot restrict the trustee from investing the trust assets in a manner that could result in the annual realization of a reasonable amount of income or gain from the sale or disposition of trust assets.23

b. Because charitable remainder trusts are also subject to many private foundation provisions of §§4940-4948,24 the governing instrument should include the required language from §508(e). Examples of such provisions are included in the sample documents written by the IRS.25 The governing instrument of a charitable remainder annuity trust must expressly forbid any additional contributions to the trust after the initial funding.26

c. The IRS has published a series of sample documents that can be used as a guideline in drafting the trust’s governing instrument.27

5. Annuity or unitrust requirement

The trust must be either a charitable remainder annuity trust or a charitable remainder unitrust.28 The payment requirements for the two types of trust cannot be combined. A combination will result in the disqualification of the trust.

Example: An irrevocable trust created by G provides that he will receive annually for the remainder of his life the greater of $5,000 or five percent of the annually determined value of the trust assets with any remainder to be paid to charity.

Result: The trust will fail to qualify as a charitable remainder annuity trust because the payment in some years is not a “sum certain.”29 It will also fail to qualify as a charitable remainder unitrust because the payment for a particular year may be a sum certain instead of the “fixed percentage” of the annually determined value of the trust assets, as required for charitable remainder unitrusts.30

6. Permissible income recipients

Payments from the trust must be made to at least one named noncharitable person.31 Included within the Code’s definition of “person” are individuals, trusts, estates, associations, companies, corporations, and partnerships.32 Other permissible recipients include members of a named class,33 noncharitable trusts,34 or charitable organizations.35 Pets, however, are not permissible income recipients.36

21 See, e.g., the provisions in the sample instrument in Rev. Proc. 89-20, 1989-1 C.B. 841. 22 Treas. Regs. §§1.664-2(a)(6)(i) and 1.664-3(a)(6)(i). 23 Treas. Regs. §1.664-l(a)(3). 24 I.R.C. §4947. 25 See, e.g., Rev. Proc. 89-20, 1989-1 C.B. 841, §4.06. 26 Treas. Regs. §1.664-2(b). For sample language, see Rev. Proc. 89-21, 1989-1 C.B. 842, §4.05. 27 See e.g., Rev. Proc. 2005-52, 2005-34 I.R.B. 326 (inter vivos one life charitable remainder unitrust), Rev. Proc. 2005-53,

2005-34 I.R.B. 339 (inter vivos charitable remainder unitrust for a term certain); Rev. Proc. 2005-54, 2005-34 I.R.B. 353 (inter vivos charitable remainder unitrusts with consecutive interests for two-lives), Rev. Proc. 2005-55, 2005-34 I.R.B. 367 (inter vivos charitable remainder unitrusts with concurrent and consecutive interests for two-lives); Rev. Proc. 2005-56, 2005-34 I.R.B. 383 (testamentary one-life charitable remainder unitrusts); Rev. Proc. 2005-57, 2005-34 I.R.B. 392 (testamentary charitable remainder unitrusts for a term certain); Rev. Proc. 2005-58, 2005-34 I.R.B. 402 (testamentary charitable remainder unitrusts with consecutive interests for two-lives); and Rev. Proc. 2005-59, 2005-34 I.R.B. 412 (testamentary charitable remainder unitrusts with concurrent and consecutive interests for two-lives).

28 Treas. Regs. §1.664-l(a)(2). 29 I.R.C. §664(d)(1). 30 I.R.C. §664(d)(2). 31 I.R.C. §664(d)(1)(A), (2)(A); Treas. Regs. §§1.664-2(a)(3)(i) and 1.664-3(a)(3)(i). 32 I.R.C. §7701(a)(l). 33 Treas. Regs. §§1.664-2(a)(3)(i) and 1.664-3(a)(3)(i). However, if the class includes any individuals, and the payment term

is for life, all individuals must be alive and ascertainable at the time the trust is created.

Page 42: Advanced Selected Issues for Trusts, Estates, and Their ... · 28/07/1997  · 3. Availability of §179 expensing 2 C. Administrative stance 2 1. Estates 2 2. Trusts 3 D. Judicial

cpenow.com / [email protected] Copyright © 2016 Surgent McCoy CPE, LLC -- ITEB/16/01 2-6

7. Exclusive function

The trust must meet the definition of and function exclusively as a charitable remainder trust from the date of its creation.37 An inter vivos trust is created on the earliest date on which the trust contains property38 and no person is considered to be the owner of the entire trust corpus under §§671-678 (grantor-trust rules).39 Neither the grantor nor the grantor’s spouse is treated as an owner of the trust corpus merely because the spouse is named as a beneficiary of the trust.40 For a testamentary trust, the date of creation will generally be the date of death even though the funding of the trust may be deferred.41

8. Trustee

Either the grantor or an independent party can serve as trustee.42 If the grantor serves as trustee, the governing instrument must be carefully drafted to avoid the grantor trust rules of §§671-678. For example, the grantor may not have the power as trustee to affect or alter the income distributions.43 If the grantor retains such power, the grantor will be treated as the owner of the trust, resulting in the disqualification of the trust.44

9. Charitable deduction

The creation of a qualified charitable remainder trust entitles the grantor, in the case of an inter vivos trust, or the testator’s estate, in the case of a testamentary trust, to an immediate charitable deduction.45 The amount of the deduction is the current fair market value of the remainder interest passing to charity. Calculation of the remainder interest’s current value is governed by §7520. Section 7520 requires a fluctuating monthly discount rate equal to 120 percent of the applicable federal midterm rate under §1274(d), compounded semiannually and rounded to the nearest two-tenths of one percent. In addition, §7520 prescribes the use of actuarial tables, which reflect more recent mortality data. The appropriate discount rate is determined for the month in which the valuation of the interest is to be made. For an inter vivos transfer, the appropriate valuation date is generally the date on which the grantor transfers the property to the trust.46 For a testamentary transfer, the appropriate valuation date is the date of death, unless the alternate valuation date is elected under §2032.47 In either case, a taxpayer may elect to use the rate in effect for the appropriate valuation month or the rate in effect for either of the two preceding months.48 Once the appropriate discount rate has been determined, comprehensive tables prepared by the IRS can generally be used to calculate the current value of the remainder interest.49 Where the tables do not provide the appropriate factor, a taxpayer may request that the IRS furnish the needed valuation factor.50

34 Rev. Rul. 76-270, 1976-1 C.B. 194. 35 Treas. Regs. §§1.664-2(a) (3)(i) and 1.664-3(a) (3)(i). 36 Rev. Rul. 78-105, 1978-1 C.B. 295. 37 Treas. Regs. 1.664-l(a)(4). 38 Id. 39 Id. 40 Id. 41 Treas. Regs. §1.664-1(a)(5)(i). 42 PLR 7730015. 43 I.R.C. §674. 44 Treas. Regs. §1.664-l(a)(4). In addition to the disqualification, application of the grantor-trust rules would cause the

grantor to be taxed directly on any income earned or any gains realized by the trust. 45 I.R.C. §§170(f)(2)(A), 2522(c)(2)(A), and 2055(e)(2)(A). 46 Treas. Regs. §§1.664-2(c) and 1.664-4(b)(3). 47 Id. 48 I.R.C. §7520(a). 49 IRS Publications 1457, Actuarial Values Aleph Volume, and 1458, Actuarial Values Beth Volume. Publication 1457 should

be used in calculations regarding annuity trusts while 1458 should be used for unitrusts. 50 Treas. Regs. §1.664-4(a)(4). Situations where this may occur include payments for more than two lives or payment

frequencies not provided for in the tables.

Page 43: Advanced Selected Issues for Trusts, Estates, and Their ... · 28/07/1997  · 3. Availability of §179 expensing 2 C. Administrative stance 2 1. Estates 2 2. Trusts 3 D. Judicial

cpenow.com / [email protected] Copyright © 2016 Surgent McCoy CPE, LLC -- ITEB/16/01 2-7

10. Computation of the present value of the remainder interest for an annuity trust

The present value of the remainder interest in an annuity trust is the net fair market value of the property transferred to the trust less the present value of the annuity (income) interest.51

Example 1: Annuity for a term of years -- Assume on January 1, 2016, G transfers property with a net fair market value of $200,000 to a charitable remainder annuity trust from which he will receive an annuity of $10,000, payable annually on December 31 for the next 10 years. Also, assume G decides to use a §7520 rate of 6.4 percent for January 2016 for valuation purposes. The present value of the remainder interest is:

1.000000 Less: Remainder factor52 - .537754 Income factor .462246 Applicable §7520 rate ÷ .064 Annuity factor 7.222594 Annuity amount x 10,000 Value of annuity 72,226

======= Value of remainder interest (200,000 - 72,226)

127,774 =======

Example 2: Annuity for single life -- G, in this example, is to receive a lifetime annuity and

is nearest to age 65. The present value of the remainder interest is:

1.000000 Less: Remainder factor53 - .414640 Income factor .585360 Applicable §7520 rate ÷ .064 Annuity factor 9.146250 Annuity amount x 10,000 Value of annuity 91,463

======= Value of remainder interest (200,000 - 91,463)

108,537 =======

51 Treas. Regs. §l.664-2(c). 52 From IRS Publication 1457, Actuarial Values Aleph Volume, Table B. The factor may also be calculated in accordance

with the instructions in Notice 89-24, 1989-1 C.B. 660. 53 From IRS Publication 1457, Actuarial Values Aleph Volume, Table S (6.4).

Page 44: Advanced Selected Issues for Trusts, Estates, and Their ... · 28/07/1997  · 3. Availability of §179 expensing 2 C. Administrative stance 2 1. Estates 2 2. Trusts 3 D. Judicial

cpenow.com / [email protected] Copyright © 2016 Surgent McCoy CPE, LLC -- ITEB/16/01 2-8

11. Computation of the present value of the remainder interest for a unitrust

Determination of the present value of the remainder interest is more complex for a unitrust. a. First, an “adjusted payout rate” is computed by multiplying the fixed percentage provided

in the governing instrument by a “payout factor” obtained from the F Tables in Publication 1458, or Prop Regs. §1.664-4.

b. Once the adjusted payout rate is determined, a valuation factor is obtained from the appropriate table.54

c. Finally, with the appropriate valuation factor, the present value of the remainder interest is determined by multiplying the factor by the net fair market value of the assets placed in trust.55

Example: Unitrust for single life -- Our old friend G, instead of transferring the property to

an annuity trust, transfers the property to a unitrust that is required to pay 10 percent of the fair market value of the trust assets determined at the beginning of each year. The present value of the remainder interest is:

Payout rate 10 percent Payout factor from Table F (6.4) x .939850 Adjusted payout rate 9.398500 % Factor at 9.2% at age 65 .284440 Factor at 9.4% at age 65 .278320 Difference (0.09398500 - 0.092) / 0.002 = X / 0.00612

x = .006074

Factor at 9.2% at age 65 .284440 Less: X - .006074 Interpolated remainder factor .278366 Fair market value of transferred assets x 200,000 Present value of remainder interest 55,673

12. Limitations

The ability of the grantor to utilize the income-tax charitable deduction is generally determined by §170(b). Among other rules, the deduction for the contribution of appreciated capital-gain property to a public charity is generally limited to 30 percent of the grantor’s contribution base, while the contribution cash is limited to 50 percent of the grantor’s contribution base. The contribution base is the grantor’s adjusted gross income. To the extent the contribution is so limited, a carryover is permitted to the five succeeding taxable years.56

Note:

For gift and estate taxes, the charitable deduction is not limited to a certain percentage. However, the IRS has taken the position that a charitable deduction is not available for gift- or estate-tax purposes where the probability that the charitable remaindermen will not receive any trust corpus exceeds five percent. The basis for this position is found in both the estate- and gift-tax regulations.57

54 Either Table D from Treas. Regs. §1.664-4(d)(5) if the unitrust amount is payable for a term of years, Table U(1) in IRS

Publication 1458 if the unitrust amount is payable for the life of one individual, or Table U(2) (found only in Publication 1458) if the amount is payable for over two lives. Prop. Regs. §1.664-4 contains selected tables as well. If the specific adjusted payout rate is not given in the table, the valuation factor must be found by interpolation.

55 Treas. Regs. §1.664-4(b)(3). 56 I.R.C. §170(d). 57 Treas. Regs. §§20.2055-2(b)(1) and 25.2522(c)-3(b)(1).

Page 45: Advanced Selected Issues for Trusts, Estates, and Their ... · 28/07/1997  · 3. Availability of §179 expensing 2 C. Administrative stance 2 1. Estates 2 2. Trusts 3 D. Judicial

cpenow.com / [email protected] Copyright © 2016 Surgent McCoy CPE, LLC -- ITEB/16/01 2-9

13. Tax consequences for the grantor

a. Transfers of property, including appreciated property, to charitable remainder trusts are generally free of income tax. However, the bargain-sale rules will apply to transfers of mortgaged property and transfers in exchange for any property other than an interest in the trust.58

Note:

The Revenue Reconciliation Act of 1993 repealed §57(a)(6), which eliminated contributions of appreciated property as tax-preference items.59

b. The creation of an inter vivos charitable remainder trust will not have any gift

consequences as long as the grantor retains the income interest and is the only noncharitable beneficiary, as the gift of the remainder interest is generally offset by the charitable deduction.60 If any other noncharitable beneficiaries exist, the present value of their interest in the annuity or unitrust is generally a gift. If the grantor’s spouse is the only other noncharitable beneficiary, a gift-tax marital deduction is available.61 For other noncharitable beneficiaries, the $14,000 annual exclusion per donee is available for gifts of a present interest.62 The annuity or unitrust interest will qualify as a present interest where the trustee is required to make annual distributions.63 However, the exclusion is not available for gifts that will commence at some future occurrence or date.64

c. The grantor’s retention of the right to receive payment of the annuity or unitrust amount or the right to terminate or revoke the interest of a noncharitable beneficiary will generally cause the entire value of the trust be included in the grantor’s estate.65 The estate, however, will be entitled to a charitable deduction for the present value of the remainder interest passing to charity.66 The amount of the deduction will depend on the number of successive noncharitable beneficiaries provided for by the trust. In addition to the charitable deduction, a special marital deduction is allowed the grantor’s estate if the grantor’s spouse is the sole secondary beneficiary.67 However, if any other noncharitable beneficiary besides the surviving spouse exists, the marital deduction is not allowed and the value of the interests passing to all noncharitable beneficiaries will be subject to the estate tax.

14. Charitable remainder trusts: pro rata division of trusts by State Court

In situations where a state court has approved a pro rata division of a CRAT or CRUT into one such separate trust for each recipient living at time of division, with each separate trust being intended to qualify as same type of CRT, division won’t cause trust or separate trusts to fail to qualify as CRT under Code §664(d). Basis of each separate trust’s share of each asset immediately after division of trust is the same share of basis of that asset in hands of trust immediately before division. Excise tax imposed under

58 Treas. Regs. §1011-2. Additionally, if the trust holds any mortgaged property, it may lose its tax-exempt status if this

generates debt-financed income, which is considered unrelated business taxable income. See I.R.C. §514(c)(9); more exceptions that apply to mortgaged real estate.

59 P.L. 103-66, 103rd Cong., 1st Sess. 60 I.R.C. §2522(a). 61 I.R.C. §2523(g). 62 I.R.C. §2503(b). 63 Treas. Regs. §25.2503-3(b). 64 Treas. Regs. §25.2503-3(c). 65 I.R.C. §2036(a). 66 I.R.C. §2055. 67 I.R.C. §2056(b)(8).

Page 46: Advanced Selected Issues for Trusts, Estates, and Their ... · 28/07/1997  · 3. Availability of §179 expensing 2 C. Administrative stance 2 1. Estates 2 2. Trusts 3 D. Judicial

cpenow.com / [email protected] Copyright © 2016 Surgent McCoy CPE, LLC -- ITEB/16/01 2-10

Code §507(c) wouldn’t apply: trust didn’t terminate private foundation status by its division. Since remainder interest of trust remains preserved exclusively for charitable interests, and there is no increase in annuity or unitrust amount at charitable interest’s expense, division of assets isn’t a sale or exchange between private foundation and disqualified person, so Code §4941, excise tax for self-dealing, doesn’t apply. Finally, division of trust wouldn’t create taxable expenditure within the meaning of Code §4945.68

B. Planning with charitable remainder trusts

1. Avoiding capital-gains tax on appreciated investment property

Holding appreciated property is the classic charitable remainder trust opportunity. An individual holds investment property that has greatly appreciated in value, such as securities or land, which in many cases is not producing a satisfactory cash flow or the individual desires to diversify investments. A sale of the investment would result in the appreciated portion being taxed, reducing the amount available for other investments. A charitable remainder trust, however, would save the amount paid in taxes and increase cash flow.

Example: Suppose Mr. Will, age 67, and Mrs. Will, age 63, have a parcel of unimproved real property worth approximately $1,250,000, which has a basis (because they acquired it over 25 years ago) of $50,000. The property does not produce any income, but costs an increasing amount in taxes each year. If they sell the property and reinvest the proceeds without a charitable remainder trust, Mr. and Mrs. Will would incur a taxable gain of $1,200,000 ($1,250,000 - $50,000), producing $180,000 in federal income taxes at the maximum capital gain rate of 15 percent. If the balance of the proceeds, $1,070,000, is invested at an eight-percent rate of return, the proceeds produce an annual income of $85,600.

However, if Mr. and Mrs. Will establish a unitrust, contribute the appreciated property to it, and allow it to sell the real estate, different financial results occur. Suppose the unitrust is required to pay each year eight percent of the fair market value of the trust assets, valued annually, to Mr. Will for his life and then to Mrs. Will for the time that she survives him. The Wills will also be entitled to a charitable income-tax deduction equal to the present value of the remainder interest. As illustrated previously, the present value is determined for a unitrust by multiplying the appropriate valuation factor by the net fair market value of the assets placed in trust. If the appropriate valuation factor for Mr. and Mrs. Will is 0.178776, then the present value of the remainder interest, and the charitable deduction, is $223,470 ($1,250,000 x 0.178776). Because the unitrust is not taxed on the gain from the disposition of the real estate, the entire $1,250,000 can be invested. The unitrust would then provide the eight-percent payout based on the fair market value of $1,250,000 or $100,000 (an increase of $14,400 in cash flow not including the possible tax savings from the charitable deduction).

In addition to the income-tax savings, Mr. and Mrs. Will have been able to remove a sizeable amount from their estates as well.69 Because the asset is being irrevocably given to the trust and will not be available to pass to one’s heirs, an individual may be reluctant to use the charitable remainder trust. However, a variant of the above would take some of the additional cash flow and use it to fund an irrevocable life insurance trust. This would maintain (possibly even increase) the assets passing to their heirs and still remove the assets from their estate.

68 Rev. Rul. 2008-41, 2008-2 CB 170. 69 As noted previously, I.R.C. §2036(a) will cause the fair market value of the trust assets to be included in Mr. Will’s estate.

However, that amount will be offset by both the charitable deduction for the remainder interest and the marital deduction permitted by I.R.C. §2056(b)(8).

Page 47: Advanced Selected Issues for Trusts, Estates, and Their ... · 28/07/1997  · 3. Availability of §179 expensing 2 C. Administrative stance 2 1. Estates 2 2. Trusts 3 D. Judicial

cpenow.com / [email protected] Copyright © 2016 Surgent McCoy CPE, LLC -- ITEB/16/01 2-11

2. Funding for a college education

This method involves using a charitable remainder trust to fund a child’s college education. Because of the high cost of such education, many parents begin to save and invest early. In many cases, the investments are in high-growth, low-yielding securities. Again the situation involves appreciated investment property where charitable remainder trusts have a significant advantage. If securities are sold or transferred to a noncharitable irrevocable trust that then sells the securities, the gain will be taxed at the trust’s tax bracket.70 The charitable remainder trust can sell the appreciated securities tax-free resulting in the full fair market value being available to fund the child’s college education. In addition, the parents will get a charitable income-tax deduction. Because the child is the beneficiary of either trust (the irrevocable trust or the charitable remainder trust), the parents will be making a gift to their child. However, their transfer to the charitable remainder trust will result in a lower taxable amount because of the charitable-gift deduction.71

3. Pension alternative

The complex rules governing pension plans, the benefit and contribution limitations on highly compensated employees, and the expense of maintaining qualified plans have caused many employers to seek alternatives. One alternative is the charitable remainder net income only with make-up unitrust. Contributions can be made to the unitrust each year in an amount that is not subject to the pension plan limitations. However, such contributions would be additional compensation to the employee and would only be somewhat offset by the charitable income-tax deduction. The trustee would then invest the contributions in low-yield, high-growth securities. When the donor reaches retirement age, the securities are sold tax-free and the proceeds invested in tax-exempt bonds. Because the investments were in low-yielding securities, the unitrust should be able to pay a higher amount to the donor after retirement than the required percentage because of the trust’s make-up provision.72

4. Sale of an interest in a CRAT or CRUT

a. Charitable remainder trusts new developments. Charitable remainder trusts (CRTs) take advantage of the fact that lifetime gifts to charity are almost always superior from a tax standpoint to testamentary charitable transfers. A bequest by will is deductible for estate-tax charitable deduction purposes. A lifetime gift has the same estate tax effect as a bequest because at the donor’s death the property has been removed from the donor’s estate, but in addition a portion of the lifetime gift is recaptured through the charitable income-tax deduction.

Example: A testator in a 40-percent estate-tax bracket who bequeaths $1,000,000 to charity

recovers most of it through the estate-tax deduction. If the property had been given to charity during lifetime, not only would the estate tax have been saved (because the property would not have been in the donor’s estate at the date of death) but a portion of the gift would have been recovered through the income-tax deduction.

A charitable remainder trust is a basic concept. Donor transfers property to trust

retaining an income interest for life or lives, with remainder passing to charity at the last beneficiary’s death. Donor receives an immediate income-tax deduction for the actuarial

70 This assumes the trust sells the securities within two years of transfer bring the sale under I.R.C. §644. 71 See, for example, Bettigole, “Charitable Giving for the Uncharitable to Fund College Education,” 67 TAXES 736 (Nov.

1989). 72 See, for example, Breen, Gertz, and Nyberg, “Charitable Trusts Are an Alternative to Qualified Plans,” 47 Taxation for

Accountants 20 (July 1991).

Page 48: Advanced Selected Issues for Trusts, Estates, and Their ... · 28/07/1997  · 3. Availability of §179 expensing 2 C. Administrative stance 2 1. Estates 2 2. Trusts 3 D. Judicial

cpenow.com / [email protected] Copyright © 2016 Surgent McCoy CPE, LLC -- ITEB/16/01 2-12

value of the remainder. The life beneficiaries may (but are not required to) include the donor. CRTs are quite useful for buying and selling assets without immediate capital-gains tax because the CRTs are exempt from income tax. Moreover, the trustee’s ability to invest and reinvest on a tax-deferred basis is particularly important where the property transferred to the CRT is highly appreciated.

b. New considerations. In comparison to traded real estate investment trusts and mutual

funds, loans, or early withdrawals from retirement funds, and living trusts that are revocable, the CRT has not been considered a liquid asset. However, CRT grantors, fiduciaries, attorneys, and CPAs should be aware that governance of charitable remainder trust is possible. This is very important consideration for CRTs. That is, no longer does a client have to make a commitment to a CRT strategy for life or a term of years. Specifically, clients can convert a long and uncertain stream of payment into an immediate and certain lump sum.73

III. Conclusion

Given the proper planning and right situation, charitable remainder trusts can provide creative solutions to minimizing taxes for many clients.

73 See PLRs 200314021 and 200127023. Also, see Michael Jones, “A Brass Ring for IRD,” February 2006 in Trusts &

Estates and Silk and Lintott, “Selling CRT Lead interests,” August 2005 in Trusts & Estates for an excellent discussion on selling CRT interests.

Page 49: Advanced Selected Issues for Trusts, Estates, and Their ... · 28/07/1997  · 3. Availability of §179 expensing 2 C. Administrative stance 2 1. Estates 2 2. Trusts 3 D. Judicial

cpenow.com / [email protected] Copyright © 2016 Surgent McCoy CPE, LLC -- ITEB/16/01 3-i

The Alternative Minimum Tax for Estates and Trusts

Learning objectives 1 I. The AMT 1

A. Background 1 1. Application 1 2. Need for strategies 1

B. AMT calculation 1 1. Concept of AGI 1 2. Alternative tax net-operating-loss deduction 2 3. Alternative tax itemized deductions 2 4. Tax-preference items and adjustments 3 5. Calculating the alternative minimum tax 5 6. Minimum tax credit 6

C. DNI for minimum tax purposes 6 1. In general 6 2. DNAMTI 7 3. Differences 7

D. Tax-planning opportunities -- AMT 7 1. Accelerate ordinary income 7 2. Defer payment of non-AMT deductions 8 3. Capital-gain property -- Estates only 8 4. Impact of the tax adjustment 8 5. Potential AMT credit problem 9

Page 50: Advanced Selected Issues for Trusts, Estates, and Their ... · 28/07/1997  · 3. Availability of §179 expensing 2 C. Administrative stance 2 1. Estates 2 2. Trusts 3 D. Judicial
Page 51: Advanced Selected Issues for Trusts, Estates, and Their ... · 28/07/1997  · 3. Availability of §179 expensing 2 C. Administrative stance 2 1. Estates 2 2. Trusts 3 D. Judicial

cpenow.com / [email protected] Copyright © 2016 Surgent McCoy CPE, LLC -- ITEB/16/01 3-1

The Alternative Minimum Tax for Estates and Trusts

Learning objectives

After completing this chapter, you will be able to: • Identify criteria that triggers the alternative minimum tax (AMT) on estates and trusts; • Determine the AMT calculation for an estate or trust; and • List tax-planning strategies to mitigate or avoid the AMT.

I. The AMT

A. Background

1. Application

While much has been written about the alternative minimum tax (AMT) for individual taxpayers, little attention has been focused on its application for estate and trust taxpayers. Generally, estates and trusts are treated in the same manner as individuals; however, there are significant areas of difference in the application and focus of the AMT to fiduciary taxpayers.

2. Need for strategies

The purposes of this module are to review and illustrate the AMT as it applies to estates and trusts and to develop tax-planning strategies to mitigate and/or circumvent the imposition of the AMT.

B. AMT calculation

1. Concept of AGI

Prior to the Tax Reform Act of 1986 (TRA ’86),1 adjusted gross income (AGI) of a taxpayer was the cornerstone on which the AMT was built. However, this concept of AGI did not exist for estates and trusts. As a consequence, this explains why Form 1041 did not provide a line caption for AGI as did Form 1040. Nonetheless, Form 6251’s instructions did state that the AGI of an estate or trust was to be figured in the same way as for an individual except that the costs of administration of an estate or trust were allowed in deriving AGI. Additionally, the Service ruled that interest on estate tax deferred under §6166 did not constitute an administration expense when calculating the AGI of an estate.2

Example 1: John Henry’s estate has income of $200,000 for the year. The only expenses for the year (prior to 1986) are administration expenses of $150,000 including $40,000 for interest on estate tax deferred under §6166. As a result, for the year, the AGI of the estate is $90,000 ($200,000 - (150,000-40,000)).

Now, under TRA ’86, taxable income will serve as the focal point for AMT purposes.3

1 PLR 99-514. 2 PLR 8409008. 3 I.R.C. §55(b).

Page 52: Advanced Selected Issues for Trusts, Estates, and Their ... · 28/07/1997  · 3. Availability of §179 expensing 2 C. Administrative stance 2 1. Estates 2 2. Trusts 3 D. Judicial

cpenow.com / [email protected] Copyright © 2016 Surgent McCoy CPE, LLC -- ITEB/16/01 3-2

Note:

Because §6166 interest is no longer deductible as an administrative expense, it will not enter into the calculation of AGI or taxable income.

2. Alternative tax net-operating-loss deduction

After the alternative minimum taxable income (AMTI) has been determined, but for the adjustment for NOLs it is reduced by the amount of the alternative tax net-operating-loss deduction (AMTNOL), if any. The AMTNOL is calculated by starting with the §172 NOL and reducing it by the amount of the AMTI tax-preference items and increasing and reducing by tax adjustments, in the year that was taken into account in computing the NOL. The 1986 Act limits the amount of the deduction to 90 percent of AMTI as determined without considering the AMTNOL deduction.4 Thus, it is no longer possible to completely eliminate the AMT solely through an AMTNOL carryover.

Note:

In computing gross income and deductions for §172, a trust must exclude any portion of income and deductions attributable to the grantor or another person under §§671-678.5 Additionally, an estate or trust is not allowed to consider the charitable-contribution deduction or §§651 and 661 deductions for distributions in determining the NOL.6

Example 2: The John Smith Trust has a loss of $40,000, including a $10,000 charitable

contribution. For purposes of the net operating loss, the trust has a loss of $30,000.

3. Alternative tax itemized deductions

a. For noncorporate taxpayers, taxable income must be adjusted by the excess of itemized deductions for regular purposes over the itemized deductions allowed for alternative tax purposes. Fewer itemized deductions are allowed in calculating AMTI than are allowed in computing the regular taxable income of an estate or trust. Several items (e.g., state and local income taxes, real property taxes) are allowed as itemized deductions for computation of regular taxable income, but not allowed for computing AMTI. Some of the more frequently encountered itemized deductions allowed in computing AMTI for an estate or trust include: (i) Estate tax attributable to income in respect of a decedent under §691(c); (ii) The deductions allowable for distributions under §§651(a) and 661(a); (iii) The charitable deduction under §642(c); (iv) The interest deduction, to the extent it does not exceed qualified net investment

income of the taxpayer, except for qualified housing interest, which is handled separately; and

(v) Qualified housing interest. b. An estate does not appear eligible for a deduction for qualified housing interest, since it is

not a principal residence of the taxpayer, the estate, or a qualified dwelling used by the taxpayer, against the estate.7 On the other hand, this interest may qualify as an alternative tax itemized deduction.

4 I.R.C. §56(d)(1)(A). 5 Treas. Regs. §1.642(d)-1(a). 6 Treas. Regs. §1.642(d)-1(b). 7 Rev. Rul. 54-583, 1954 -2 C.B. 158, and see Frank MacBoyle Lewis Testamentary Trust B dated 8/28/78, 83 T.C. 246

(1984).

Page 53: Advanced Selected Issues for Trusts, Estates, and Their ... · 28/07/1997  · 3. Availability of §179 expensing 2 C. Administrative stance 2 1. Estates 2 2. Trusts 3 D. Judicial

cpenow.com / [email protected] Copyright © 2016 Surgent McCoy CPE, LLC -- ITEB/16/01 3-3

4. Tax-preference items and adjustments

For estates and trusts, the alternative minimum tax (AMT) may apply as an addition to the regular tax for a taxable year. That is,8 the alternative minimum taxable income (AMTI) of the entity and any beneficiary is determined by applying Subchapter J with modifications. Specifically, these modifications are termed adjustments and preferences.

a. TRA ’86 uses two terms, adjustments and preferences, in reference to certain differences between the regular tax and the AMT, which prior law simply considered preferences. Adjustments differ from preferences only in that adjustments involve a substitution of a special AMT treatment of an item for the regular tax treatment, while a preference involves the addition of the difference between the special AMT treatment and the regular tax treatment. Typically, adjustments can be negative, i.e., they may result in an AMTI that is less than taxable income; conversely, tax preferences cannot be negative amounts.

b. The final step in computing the AMTI is to consider items of tax preference and adjustments. The code lists many tax-preference and adjustment items including: (i) Accelerated depreciation or accelerated cost recovery deductions on real

property -- The amount by which the deduction taken exceeds that which would have been allowable for the tax year had the applicable straight-line method been used constitutes a tax-preference item for property placed in service under pre-TRA ’86 law. For property covered by the 1986 Act, the alternative depreciation system, basically straight line with a 40-year life, must be used in the calculation of depreciation of real property for purposes of the AMT.9

(ii) Accelerated depreciation or accelerated cost recovery deductions on leased personal property -- Again, the excess over straight-line amounts to the tax preference for property placed in service under pre-TRA ’86 law. For property covered by the 1986 Act, AMT depreciation is computed under the alternative depreciation system, using the 150-percent declining balance method and switching to straight line in the year necessary to maximize the allowance; this is a substitute for straight-line depreciation.10

(iii) Mining, exploration, and development costs -- Under pre-TRA ’86 law, in respect of each mine or other natural deposit, excepting oil and gas wells, the excess of the deductions claimed for mining, exploration, and development costs over the amount that would have been taken if the costs were amortized ratably over 10 years is a tax-preference item. The 1986 Act provides an adjustment with respect to mine exploration and development costs. This adjustment has AMTI computed by capitalizing mine exploration and development expenditures incurred after December 31, 1986 and amortizing them on a straight-line basis over 10 years, beginning in the taxable year in which the expenditures are made.11 The new provision considers all mines together, i.e., no differentiation is made regarding those mines with a regular tax deduction exceeding its alternative tax minimum from the others. In effect, this permits mines with negative preferences to enter the calculation.

8 I.R.C. §59(c). 9 I.R.C. §56(a)(1)(A)(i). 10 I.R.C. §56(a)(1)(A)(ii). 11 I.R.C. §56(a)(2)(A).

Page 54: Advanced Selected Issues for Trusts, Estates, and Their ... · 28/07/1997  · 3. Availability of §179 expensing 2 C. Administrative stance 2 1. Estates 2 2. Trusts 3 D. Judicial

cpenow.com / [email protected] Copyright © 2016 Surgent McCoy CPE, LLC -- ITEB/16/01 3-4

(iv) Circulation, and research and experimental expenses -- Under pre-TRA ’86 law, the amount of tax preference is the excess of the deduction taken over the amount that would have been allowable had the circulation expenditures been amortized ratably over three years and the research and experimental expenses been amortized over 10 years. Under TRA ’86 for AMT purposes, capitalizing circulation and research and experimental expenditures is required, and a resulting adjustment for AMTI is computed by taking as a deduction for circulation expenditures not the amount taken for regular tax purposes but an amount that amortizes the expenditures over a three-year period. Also, for research and experimental expenditures, a similar adjustment is required using a 10-year amortization.12 This appears to create no significant change from prior law.

(v) Depletion -- The excess of the depletion deduction for the year over the adjusted basis of the property at year-end is a preference item.13 This preference was retained without change by the 1986 Act.

(vi) Incentive stock options -- The excess of the fair market value of the stock at the time of exercise over the exercise price is a preference item, unless there is an early disposition under §422A. This provision has been retained intact by the 1986 law.14 Additionally, the 1986 Act provides that, for AMT purposes, stock acquired by exercise of incentive stock options has a basis that considers the ISO preference. This rule will only apply in determining gain or loss on disposition of the stock.15 Thus, gain will generally be less for AMT purposes then for regular tax purposes.

(vii) Intangible drilling costs -- For all oil, gas, and geothermal properties, the excess of the intangible drilling costs over 65 percent of the taxpayer’s net income from the properties for the year was a tax preference.16

(viii) Interest on “specified private activity bonds,” which are exempt from regular tax, are included in AMTI (§57(a)(5)) -- On the other hand, when a deduction is denied under regular tax rules and would have been allowable if such interest were taxable, that amount reduces the private activity bond interest included to AMTI.17

(ix) For AMT purposes, no loss is permitted from any “tax shelter farm activity” of any noncorporate taxpayer,18 except to the extent of the amount by which the taxpayer is insolvent at the close of the taxable year19 -- A tax shelter farm activity is defined as either: (a) a farming syndicate; or (b) any other activity of farming unless the taxpayer materially participates in the activity.20 In determining material participation, the rules of §469(d) for determination of the limitation on deductions of passive-activity losses apply.

12 I.R.C. §§56(b)(2)(A)(i) and (ii). 13 I.R.C. §57(a)(1). 14 I.R.C. §57(a)(3)(A). 15 I.R.C. §57(a)(3)(B). 16 I.R.C. §57(a)(2)(A). 17 I.R.C. §57(a)(5)(A). 18 I.R.C. §58(a)(1)(A). 19 I.R.C. §58(c)(1)(A). 20 I.R.C. §58(a)(2).

Page 55: Advanced Selected Issues for Trusts, Estates, and Their ... · 28/07/1997  · 3. Availability of §179 expensing 2 C. Administrative stance 2 1. Estates 2 2. Trusts 3 D. Judicial

cpenow.com / [email protected] Copyright © 2016 Surgent McCoy CPE, LLC -- ITEB/16/01 3-5

c. While this is not intended as an exhaustive list, it does contain most of the tax-preference/adjustment items. Additionally, the most frequently encountered preference/adjustments items by an estate or trust would be the private activity bond interest, the excess depreciation, intangible drilling costs, and the passive farm loss provisions.

5. Calculating the alternative minimum tax

After calculating the alternative minimum taxable income, there still remains one final adjustment. That is, the Code provides an exemption of $23,800 for estates and trusts in 2015 ($23,900 in 2016).21 The exemption will be reduced 25 cents per $1.00 of AMTI exceeding $79,450 in 2015, resulting in the exemption being phased out at $174,650 of minimum taxable income.22 Estates and trusts are now subject to a two-tiered graduated tax schedule. That is, a 26-percent rate applies to the first $185,400 of AMTI in 2015 in excess of the exemption amount and the 28-percent rate applies to the AMTI that exceeds the sum of $185,400 in 2015 and the exemption amount.

Example 3: The income tax return of John Henry’s estate showed no regular tax liability. However, the estate did generate AMTI of $20,000. The use of the exemption will prevent the AMT from generating a tax liability.

Example 4: Assume John Henry’s fiduciary return has AMTI of $101,550 in 2015. Because

the AMTI exceeds $79,450, the exemption is reduced to $18,275 [$23,800- (($101,550-$79,450) x .25)].

The estate or trust must pay the higher of the AMT or the regular tax. The following illustration describes in detail an application of the AMT.

Example 5: The John Henry estate, a calendar-year taxpayer, received in 2016 dividend income of $200,000, $40,000 of bonus income related to his lifetime services, and a distributive share loss from a limited partnership in the amount of $300,000.

The estate incurred fiduciary fees of $18,000 and state fiduciary income taxes of $25,000. Also, the estate had a §691(c) deduction of $7,000.

The executor distributed $28,000 to the income beneficiary, Vivian Elmore, in 2016.

The estate had a MACRS depreciation deduction of $24,000 that, using straight-line depreciation for AMT purposes, was $12,000.

21 I.R.C. §55(d)(1)(C)(ii). 22 I.R.C. §55(d)(3)(C).

Page 56: Advanced Selected Issues for Trusts, Estates, and Their ... · 28/07/1997  · 3. Availability of §179 expensing 2 C. Administrative stance 2 1. Estates 2 2. Trusts 3 D. Judicial

cpenow.com / [email protected] Copyright © 2016 Surgent McCoy CPE, LLC -- ITEB/16/01 3-6

For 2016, the estate’s federal tax liability is calculated as follows:

Regular Tax AMT Dividends $ 200,000 $ 200,000 Bonus IRD 40,000 40,000 Partnership loss* -0- -0- Fiduciary fees -18,000 -18,000 State income tax -25,000 -0- Section 691(c) deduction -7,000 -7,000 Income distribution -28,000 -28,000 Depreciation -24,000 -12,000 Tax exemption -600 -0- Regular taxable income $ 137,400 AMTI $ 175,000 AMT Exemption: $23,900-[(175,000-$79,850)] x .25 -0- Taxable amount 137,400 175,000 Tax liability $52,719 $ 45,500 * The estate was unable to currently deduct the loss based

on the passive-loss limitations.

The estate must pay $52,719 in taxes, which is the greater of the regular tax or the AMT.

6. Minimum tax credit

The minimum tax credit23 (MTC) is a new element for AMT consideration, introduced by the 1986 Act. Its purpose is to make the AMT more equitable after 1986. Basically, the imposition of the AMT will generate a credit for all or a portion of the tax paid against the regular tax liability of the taxpayer in future years.24 It should be noted that the MTC cannot be used to reduce the regular tax liability in the carryforward year below the level of the AMT for that year.25 Under this provision, no carrybacks are permitted, but the carryover period is not limited.26

a. The MTC does not apply to the exclusion preferences. These preferences applicable to estates and trusts are: • Deduction adjustments; • Depletion; and • Tax-exempt interest.

b. The other preferences and adjustments are of a deferral nature, where the timing, not the amount of the deduction creates tax-preference treatment.27 It is these deferral preferences that produce the MTC carryover. This adjustment prevents the taxpayer from losing the benefits of certain deductions altogether.

C. DNI for minimum tax purposes

1. In general

Estates and trusts determine AMTI by applying Subchapter J with the adjustments provided in the minimum tax rules. 28 Moreover, the Conference Report on the TRA ’86 provides that, “In the case of an 23 I.R.C. §53. 24 I.R.C. §53(a). 25 I.R.C. §53(c). 26 I.R.C. §53(b). 27 S. Rep. No. 99-313, at 521. 28 I.R.C. §59(c).

Page 57: Advanced Selected Issues for Trusts, Estates, and Their ... · 28/07/1997  · 3. Availability of §179 expensing 2 C. Administrative stance 2 1. Estates 2 2. Trusts 3 D. Judicial

cpenow.com / [email protected] Copyright © 2016 Surgent McCoy CPE, LLC -- ITEB/16/01 3-7

estate or trust, instead of allocating items of tax preference between the estate and trust or its beneficiaries … it is provided that the minimum tax will apply by determining DNI on a minimum-tax basis.”29 As a result, fiduciaries of estates and trusts will have to compute DNI twice, that is, once under the regular tax rules and again under the AMT rules.

2. DNAMTI

The Service has issued a Form 1041 Schedule I -- Alternative Minimum Tax, for use by fiduciaries. This form specifies line 44 as “distributable net alternative minimum taxable income.” From this line, the fiduciary allocates to each beneficiary his or her shares of distributable net alternative minimum taxable income (DNAMTI). Next, the fiduciary reflects the beneficiary’s share of DNAMTI on the K-1. Note that Schedule K-1 contains Line 12A (DNAMTI - DNI) that provides the net AMT adjustment beneficiaries need on Line 15 of their Form 6251 in preparing their own AMT calculations for items included in DNI. Also, note that Line 12G and 12H on the K-1 breaks out depreciation and depletion preferences that are being passed out to the beneficiaries.

3. Differences

The difference between DNI and DNAMTI can be traced to several non-AMT preference and/or adjustment items. Specifically, a beneficiary’s share of DNAMTI can vary from his/her amount of DNI because of state and local income taxes, real property taxes, and “miscellaneous itemized deductions”30 in excess of two-percent of AGI. Naturally, any AMT items of adjustments and preferences incurred by the fiduciary would make DNI and DNAMTI differ as to amount.

D. Tax-planning opportunities -- AMT

1. Accelerate ordinary income

When income fluctuates from year to year, a trustee or executor may be subject to the AMT in one year, but not in another. In a case such as this, the fiduciary should consider accelerating ordinary income into a year in which the AMT applies. Since the additional income will be taxed at the AMT rate of 28 percent, substantial tax savings can result. Care must be exercised in this income-shifting technique because at some point an additional dollar of income will result in regular tax liability to the alternative tax.

Example 6: Assume a trust is active in oil and gas exploration. During the latter part of the taxable year, several wells have hit and are commercially viable. Because of the trust’s AMT situation (largely due to intangible drilling expenses), it is prudent to bring the wells on line as soon as possible because the income will be taxed at 28 percent rather than 39.6 percent.

29 Report 99-841, 9/18/86, page II-263. 30 I.R.C. §67(e).

Page 58: Advanced Selected Issues for Trusts, Estates, and Their ... · 28/07/1997  · 3. Availability of §179 expensing 2 C. Administrative stance 2 1. Estates 2 2. Trusts 3 D. Judicial

cpenow.com / [email protected] Copyright © 2016 Surgent McCoy CPE, LLC -- ITEB/16/01 3-8

2. Defer payment of non-AMT deductions

Due to the limited number of alternative-tax itemized deductions, many of the typical deductions yield no tax benefits in an AMT year. Deferring expenses is a difficult task where specific due dates are present and cannot be changed. Careful analysis of the value of deferring into a non-AMT year versus the penalty that may be incurred for late payment must be undertaken. Naturally, when the benefits outweigh the costs, consideration to deferral should be given.

3. Capital-gain property -- Estates only

The impact of income in respect of a decedent (IRD) on an estate offers a particularly significant planning opportunity, both for AMT and regular tax calculations. Careful planning prior to death can yield excellent results. Specifically, where a contract entered into prior to death has only the element of time involved prior to collection, IRD will result.31 Such is the case of the sale of a capital asset prior to death with either installment or full payment being made after death. The result is that the asset will not be eligible for a stepped up basis to fair market value at the date of death.32 Therefore, the IRD is subject to either regular tax or the AMT. The courts have determined that if significant acts remained to be performed or substantial contingencies remain at the date of death, the step-up in basis is available.33 Therefore, in drafting a document of sale, provisions should be made to cancel the contract in the event of the seller’s death. While in certain cases this may not be possible, in most cases, if the buyer knows that the buyer will still receive the asset, under similar terms, no objection should be made. This allows the successor to become the “architect” of a new deal, and a stepped-up basis is allowable. The regulations give good tax-planning advice in the IRD arena.34 That is, a taxpayer can enter into a sales contract providing that the sale becomes final only after the seller’s death. Proceeds received under such a contract will not be IRD.

4. Impact of the tax adjustment

State and local income taxes, foreign income taxes, and property taxes (not associated with a trade or businesses) are not deductible for AMT purposes but are deductible for regular tax purposes. As a result, state taxes can, in many instances, create an AMT problem for an estate or trust. Consider the following scenario:

Example 7: A trust is required by its instrument to distribute all of its income currently, and capital gains are allocable to corpus. In 2016, the trust has $200,000 of income and $50,000 of state taxes that arose in 2015 but were paid in 2016 and allocated to principal.

31 I.R.C. §691(a). 32 I.R.C. §1014(c). 33 See Estate of Charley W. Peterson v. Commissioner, 667 F. 2d. 675 (8th Cir., 1983). 34 Treas. Regs. §1.691(a) - 2(b), Example 4.

Page 59: Advanced Selected Issues for Trusts, Estates, and Their ... · 28/07/1997  · 3. Availability of §179 expensing 2 C. Administrative stance 2 1. Estates 2 2. Trusts 3 D. Judicial

cpenow.com / [email protected] Copyright © 2016 Surgent McCoy CPE, LLC -- ITEB/16/01 3-9

Regular tax computation: Income $200,000 State taxes on capital gain (50,000) DNI deduction* (150,000) Trust’s taxable income 0

AMT computation: Regular taxable income $150,000 Plus taxes 50,000 AMTI 200,000 DNI deduction (AMT) (150,000) Exemption (23,900) Taxable amount 26,100 Tax rate 26% AMT $6,786

* The lesser of net income ($200,000) and DNI ($150,000). State taxes on capital gains are allocated to principal.

Planning point:

Pay state income taxes in the tax year that the gain is generated in order to match income and distribution for AMT calculations.

5. Potential AMT credit problem

In the event that an AMT credit has not been utilized when an estate or trust terminates, the credit will be lost. That is, no tax credits pass out on termination. Under §642(h), only capital losses, net operating losses, and excess deductions will carry over to beneficiaries succeeding to the property. These attributes are discussed in detail in Chapter 4.

Planning point:

Estates and trusts should plan to withhold discretionary distributions before termination. As a result, the retained taxable income could absorb the AMT credit and prevent wastage.

Depiction of the flow-out of preferences and adjustments to beneficiaries of estates

and trusts for alternative-minimum-tax purposes During 2015, the XYZ Trust had the following items of income and expenses: Taxable interest $175,000 Tax-exempt interest on private activity bonds 50,000 Rents 200,000 Dividends (nonqualifying from high-yield Trust preferreds) 450,000 Taxes and expenses attributable to rents 10,000 Trustee’s commissions allocable to income account 3,500 Trustee’s commissions allocable to principal account 1,500 Depreciation 18,182

Pursuant to the terms of the trust, all the income is to be distributed to the beneficiary, Steve Colburn, and the trust instrument requires a reserve for depreciation. Under I.R.C. §167(d), the deduction is allocated between the trust and income beneficiary in accordance with the provisions of the trust, or in their absence, on the basis of the income allocable to each. In an estate, it is based on the income allocable to each. No distribution of principal is made.

Page 60: Advanced Selected Issues for Trusts, Estates, and Their ... · 28/07/1997  · 3. Availability of §179 expensing 2 C. Administrative stance 2 1. Estates 2 2. Trusts 3 D. Judicial

cpenow.com / [email protected] Copyright © 2016 Surgent McCoy CPE, LLC -- ITEB/16/01 3-10

STEP 1: State law income Taxable interest $175,000 Tax-exempt interest 50,000 Rents 200,000 Dividends 450,000 TOTAL 875,000 Less: Rental expenses $10,000 Depreciation* 18,182 Trustee’s commissions allocable to income account 3,500 31,682 State law income computed under §643(b) $843,318

* Because the trust provides for a reserve, the trustee allocates the depreciation against current income.

STEP 2: Section 643(a) distributable net income ceiling Taxable interest $175,000 Rents 200,000 Dividends 450,000 Tax-exempt interest on private activity bonds $50,000 Less: Expenses ($50,000/$875,000 x $5,000)* 286 49,714 TOTAL $874,714 Less: Rental expenses $10,000 Depreciation on rental property** 18,182 Trustee’s commissions ($5,000 less $286 allocated to tax-exempt interest) 4,714 32,896 DNI as income ceiling $841,818

* Total trustee commissions. ** Since depreciation is an expense against accounting income.

STEP 3: Section 651(a) deduction ceiling DNI under §643(a) $841,818 Less: Tax-exempt interest as adjusted 49,714 DNTI under §651(b)* $792,104 STEP 4: Trust’s taxable income Interest $175,000 Rents 200,000 Dividends 450,000 Gross Income $825,000 Less: Rental expenses $10,000 Depreciation 18,182 Trustee’s commissions 4,714 Dist. to beneficiaries** 792,104 Personal exemption 300 $825,300 Trust’s income 0

* Distributable net taxable income. ** Lesser of income and distributable net taxable income.

Page 61: Advanced Selected Issues for Trusts, Estates, and Their ... · 28/07/1997  · 3. Availability of §179 expensing 2 C. Administrative stance 2 1. Estates 2 2. Trusts 3 D. Judicial

cpenow.com / [email protected] Copyright © 2016 Surgent McCoy CPE, LLC -- ITEB/16/01 3-11

STEP 5: Inclusion and characterization to beneficiaries

Rents Nonqualifying Dividends

Tax-exempt interest

Taxable interest

Total

Income for trust accounting purposes

200,000

450,000

50,000

175,000

875,000

Less: Rental expenses 10,000 10,000 Depreciation 18,182 18,182 Trustee’s commissions 4,714 286 5,000 32,896 0 286 0 33,182 Character of amounts in the hands of the beneficiaries

$167,104

$450,000

$49,714

$175,000

$841,818

Attachment to Schedule I (Year ended 12/31/15)

XYZ Trust - E. I. #36-7464321 Rental property placed in service on 1/1/93 MACRS - Straight line 27.5 Years (residential rental prop.)

Basis = $500,000 / 27.5 years = $18,182 (Regular tax purposes)

For purposes of alternative minimum tax:

Basis - $500,000 / 40 years = $12,500 Adjustment $5,682

Page 62: Advanced Selected Issues for Trusts, Estates, and Their ... · 28/07/1997  · 3. Availability of §179 expensing 2 C. Administrative stance 2 1. Estates 2 2. Trusts 3 D. Judicial

cpenow.com / [email protected] Copyright © 2016 Surgent McCoy CPE, LLC -- ITEB/16/01 3-12

Page 63: Advanced Selected Issues for Trusts, Estates, and Their ... · 28/07/1997  · 3. Availability of §179 expensing 2 C. Administrative stance 2 1. Estates 2 2. Trusts 3 D. Judicial

cpenow.com / [email protected] Copyright © 2016 Surgent McCoy CPE, LLC -- ITEB/16/01 3-13

Page 64: Advanced Selected Issues for Trusts, Estates, and Their ... · 28/07/1997  · 3. Availability of §179 expensing 2 C. Administrative stance 2 1. Estates 2 2. Trusts 3 D. Judicial

cpenow.com / [email protected] Copyright © 2016 Surgent McCoy CPE, LLC -- ITEB/16/01 3-14

Page 65: Advanced Selected Issues for Trusts, Estates, and Their ... · 28/07/1997  · 3. Availability of §179 expensing 2 C. Administrative stance 2 1. Estates 2 2. Trusts 3 D. Judicial

cpenow.com / [email protected] Copyright © 2016 Surgent McCoy CPE, LLC -- ITEB/16/01 3-15

Page 66: Advanced Selected Issues for Trusts, Estates, and Their ... · 28/07/1997  · 3. Availability of §179 expensing 2 C. Administrative stance 2 1. Estates 2 2. Trusts 3 D. Judicial

cpenow.com / [email protected] Copyright © 2016 Surgent McCoy CPE, LLC -- ITEB/16/01 3-16

Page 67: Advanced Selected Issues for Trusts, Estates, and Their ... · 28/07/1997  · 3. Availability of §179 expensing 2 C. Administrative stance 2 1. Estates 2 2. Trusts 3 D. Judicial

cpenow.com / [email protected] Copyright © 2016 Surgent McCoy CPE, LLC -- ITEB/16/01 4-i

Terminating an Estate or Trust

Learning objectives 1 I. Termination 1

A. Questions involved 1 B. Timing 1

1. Termination of an estate 1 2. Termination of a trust 2

C. Beneficiaries succeeding to the property of the estate or trust 2 1. In general 2 2. Intestate estate 2 3. Testate estate 2 4. Trusts 3

D. Tax effects of termination 3 1. In general 3 2. Excess deductions 3 3. Net operating losses and capital losses 4 4. Comprehensive illustration of allocations to beneficiaries on termination 4

Page 68: Advanced Selected Issues for Trusts, Estates, and Their ... · 28/07/1997  · 3. Availability of §179 expensing 2 C. Administrative stance 2 1. Estates 2 2. Trusts 3 D. Judicial
Page 69: Advanced Selected Issues for Trusts, Estates, and Their ... · 28/07/1997  · 3. Availability of §179 expensing 2 C. Administrative stance 2 1. Estates 2 2. Trusts 3 D. Judicial

cpenow.com / [email protected] Copyright © 2016 Surgent McCoy CPE, LLC -- ITEB/16/01 4-1

Terminating an Estate or Trust

Learning objectives

After completing this chapter, you will be able to: • List general guidelines for determining when an estate or trust is to be terminated; • Determine who are beneficiaries succeeding to the property of an estate or trust; • Identify the tax attributes that flow out to beneficiaries succeeding to the property on

termination; and • Explain the allocation of excess deductions, net operating losses, and capital losses

among the beneficiaries.

I. Termination

A. Questions involved

In previous chapters, we were concerned with the taxation of estates and trusts as viable on-going tax-paying entities. However, in this chapter, we will examine the tax consequences of termination of an estate or trust. Specifically, this chapter addresses the following questions:

• When does an estate terminate? • When does a trust terminate? • Who are succeeding beneficiaries? • What happens to excess deductions? • What happens to loss carryovers? • How are allocations made among beneficiaries? • What does a 1041 return look like for a terminating trust with excess deductions?

B. Timing

1. Termination of an estate

There is no statutory, administrative, or judicial authority that identifies the exact point at which an estate terminates. However, the regulations provide the following general guidance:

The period of administration or settlement is the period actually required by the administrator or executor to perform the ordinary duties of administration, such as the collection of assets and the payment of debts, taxes, legacies, or bequests, whether the period required is longer or shorter than the period specified under the applicable local law for the settlement of estates.1

Clarification as to several ramifications of estate termination is provided in the following illustration.

Example 1: Where an executor who is also named as trustee under a will fails to obtain his discharge as executor, the period of administration continues only until the duties of administration are complete and he actually assumes his duties as trustee, whether or not pursuant to a court order. However, the period of administration of an estate cannot be unduly prolonged. If the administration of an estate is unreasonably prolonged, the estate is considered terminated for federal income-tax purposes after the expiration of a reasonable period for the performance by the executor of all duties of administration. Further, an estate will be considered terminated when all the assets have been distributed except for a reasonable

1 Treas. Regs. §1.641(b)-(3).

Page 70: Advanced Selected Issues for Trusts, Estates, and Their ... · 28/07/1997  · 3. Availability of §179 expensing 2 C. Administrative stance 2 1. Estates 2 2. Trusts 3 D. Judicial

cpenow.com / [email protected] Copyright © 2016 Surgent McCoy CPE, LLC -- ITEB/16/01 4-2

amount that is set aside in good faith for the payment of unascertained or contingent liabilities and expenses (not including a claim by a beneficiary in the capacity of beneficiary).2

2. Termination of a trust

The termination of a trust turns on very different criteria than an estate. That is, the importance of the estate’s final accounting is not relevant in trust terminations. Moreover, a trust often terminates upon the happening of a particular event, such as the death of the life tenant. Even though, pursuant to its governing instrument, the trust is to terminate on the death of the widow (life tenant) and the corpus is to be distributed to a son and daughter (the remaindermen), the trust will continue for a reasonable time after the death of the life tenant in order to finish up the business affairs of the trust. Also, the regulations3 provide that a trust will be considered terminated when all of the assets have been distributed except for the retention of a reasonable amount for the payment of contingent or unascertained liabilities.

C. Beneficiaries succeeding to the property of the estate or trust

1. In general

Beneficiaries succeeding to the property of an estate or trust identifies those beneficiaries who are entitled to receive a deduction for a net-operating-loss carryover, a capital-loss carryover, and from any excess deductions of the terminating estate or trust. Who these beneficiaries are depends on whether the decedent died testate or intestate or if the situation involved a trust.

2. Intestate estate

In an intestate estate, the succeeding beneficiaries are the heirs and next of kin to whom the estate is to be distributed, or if the estate is insolvent, then to the party to whom it would have been distributed had the estate not been insolvent.4 For example, a decedent’s spouse is entitled to a specific dollar amount, and if the estate is less than that amount, then the spouse becomes a beneficiary succeeding to the property to the extent of that deficiency.5

3. Testate estate

In the case of a testate estate, the beneficiaries succeeding to the property are typically the residuary beneficiaries (including a residuary trust).6 But it should be noted that a pecuniary legatee may also be considered a beneficiary succeeding to the property. That is, whenever a pecuniary legatee is not satisfied in full, then legatee will be a beneficiary succeeding to the property to the extent of the deficiency.7 In order to appreciate this sequence of succession and priorities relating to a pecuniary legatee, consider carefully the following illustration.

Example 2: Assume a decedent’s will leaves $200,000 to X, with the residue passing to Y and Z equally. The estate has assets only sufficient to pay $150,000, which it pays to X. No payment is made to Y or Z.

2 Treas. Regs. §1.641(b)-3(a). 3 Treas. Regs. §1.641(b)-3(b). 4 Treas. Regs. §1.642(h)-3(b). 5 Id. 6 Treas. Regs. §1.642(h)-3(c). 7 Treas. Regs. §1.642(h)-3(c).

Page 71: Advanced Selected Issues for Trusts, Estates, and Their ... · 28/07/1997  · 3. Availability of §179 expensing 2 C. Administrative stance 2 1. Estates 2 2. Trusts 3 D. Judicial

cpenow.com / [email protected] Copyright © 2016 Surgent McCoy CPE, LLC -- ITEB/16/01 4-3

4. Trusts

In the case of a trust, a remainderman, who receives all or a portion of the property of a trust on final termination, is considered to be a beneficiary succeeding to the property of the trust.8

Example 3: If property is transferred into trust to pay the income to X for life and then to pay $30,000 to Y and distribute the balance of the trust corpus to Z, then Z and not Y is considered to be the beneficiary succeeding to the property except to the extent that the trust corpus is insufficient to pay Y $30,000.9

D. Tax effects of termination

1. In general

Under §642(h), the beneficiaries succeeding to the property of an estate or trust on termination may take advantage of:

• A net-operating-loss carryover under §172 or a capital-loss carryover under §1212; or • For the last taxable year of the estate or trust, deductions (other than the deductions

allowed under personal exemption or charitable deduction) in excess of gross income for such year.

2. Excess deductions

a. In the event that a terminating estate or trust has deductions (other than personal exemptions and charitable contributions) in excess of its income for it last taxable year, then these excess deductions are allowed as a deduction to the beneficiaries succeeding to the property.

Example 4: In Example 2, assume that there is an excess of deductions over gross income

for the last taxable year of the estate of $60,000, to which §642(h) applies. X is the beneficiary succeeding to the property of the estate to the extent of $50,000, and since the total of the excess deductions is $60,000, X is entitled to the benefit of $50,000, and the remaining $10,000 is divided equally between Y and Z. If there had been only $140,000 left to distribute to X, he would have been entitled to all of the excess deductions.10

b. The excess deductions are treated by the beneficiaries as itemized deductions and these

excess deductions must be used by the beneficiaries in their taxable years during the year the estate or trust terminates.11 As a result of the Tax Reform Act of 1986, excess deductions will be subject to the two-percent-of-adjusted-gross-income floor. Additionally, there is no carryover associated with excess deductions.

Example 5: Assume that an estate distributes all its assets to X and terminates on December

31, 2016. As of that date, the estate had excess deductions of $20,000 from executor’s commissions. X, who reported on the calendar-year basis, can claim $20,000 as an itemized deduction for the taxable year 2016. As nonbusiness deductions, they cannot create a net operating loss.12 If X’s adjusted gross income was only $15,000, then the excess of $5,000 deductions could not be carried to subsequent years. Also, it should be noted that the beneficiary’s excess deductions would be subject to the two-percent-of-adjusted-gross-income rule in 2016.13

8 Treas. Regs. §1.642(h)-3(d). 9 Based on Treas. Regs. §1.642(h)-3(d). 10 Based on Treas. Regs. §1.642(h)-4. 11 Treas. Regs. §1.642(h)-2(a). 12 See I.R.C. §172(d). 13 I.R.C. §68.

Page 72: Advanced Selected Issues for Trusts, Estates, and Their ... · 28/07/1997  · 3. Availability of §179 expensing 2 C. Administrative stance 2 1. Estates 2 2. Trusts 3 D. Judicial

cpenow.com / [email protected] Copyright © 2016 Surgent McCoy CPE, LLC -- ITEB/16/01 4-4

3. Net operating losses and capital losses

A beneficiary succeeding to the property of a terminating estate or trust is allowed to use an estate or trust’s net-operating-loss carryforward and/or a capital-loss carryforward.14 As a result, even an insolvent trust or estate would provide its beneficiaries with important tax benefits.

Note:

The net operating loss on termination is only available to the beneficiary on a pure carryforward basis; it cannot be carried back. Also, capital losses are available to beneficiaries, who are individuals, estates, and trusts on an unlimited carryforward basis.15

4. Comprehensive illustration of allocations to beneficiaries on termination

In order to appreciate the proportionate allocation of carryovers and excess deductions to beneficiaries who bear the burden of the underlying losses or expenses, examine carefully the following illustration.

Example 6: A decedent dies in May 2014. A May 31 fiscal tax year was selected by the estate to minimize the estate taxes and to defer 11 months into the next tax year. His will provided that the estate was to be distributed to X and to a trust for the benefit of Y and Z in equal shares. The estate was terminated at the end of its fiscal year of May 31, 2016, and the property distributed equally to X and the trust. X and the trust are calendar-year taxpayers.

During the period of administration the estate had the following items of income and deductions:

Business income $50,000 Taxable interest 10,000 Total income $60,000 Business expenses $55,000 Administrative expenses and corpus commissions not allowable to business income

20,000

Total deductions $75,000

The estate also had a net long-term capital loss of $10,000. The net operating loss of the estate is computed as follows:

Deductions of the estate for the fiscal year ending May 31, 2016

$75,000

Less adjustment under §172(d)(4) (deductions not attributable to a trade or business ($20,000) are allowable only to the extent of gross income not derived from that trade or business ($10,000))

10,000

Adjusted deductions $65,000 Gross income of the estate 60,000 Net operating loss of the estate for the fiscal year ending May 31, 2016.

$5,000

The capital loss is not includable for net-operating-loss purposes (§172(d)(2)). Neither X nor the trust will be allowed to carryback any part of the net operating loss. The $5,000 net operating loss will be allocated between X on the one hand and the trust for the benefit of Y and Z on the other equally: to X in the amount of a $2,500 net operating loss for his calendar year ended December 31, 2016, and the next 19 years, and to the trust in the amount of a $2,500 net operating loss for its calendar year ended December 31, 2016 and its next 19 taxable years.

14 Treas. Regs. §1.642(h)-1(a). 15 I.R.C. §1212(b).

Page 73: Advanced Selected Issues for Trusts, Estates, and Their ... · 28/07/1997  · 3. Availability of §179 expensing 2 C. Administrative stance 2 1. Estates 2 2. Trusts 3 D. Judicial

cpenow.com / [email protected] Copyright © 2016 Surgent McCoy CPE, LLC -- ITEB/16/01 4-5

The excess deductions of $10,000 will be allowed as an itemized deduction to X to the extent of $5,000 for his calendar year 2016 and to the trust to the extent of $5,000 for the calendar year ended December 31, 2016.

X is allowed a long-term capital-loss carryover of $5,000 for his taxable year 2016 with an indefinite carryover. The trust is allowed a long-term capital-loss carryover of $5,000 for its taxable year ended December 31, 2016 with an indefinite carryover. It should be noted that the trust receives all of the deductions and none is allowable directly to Y and Z, the trust beneficiaries. Y and Z may receive indirect benefit to the extent that the distributable net income of the trust is reduced by the carryovers and excess deductions.16

Illustration of Excess Deductions in Year of Complete Termination of a Complex Trust

A complex trust was terminated on September 30, 2015. The trust had the following income and expenses:

Domestic dividends $15,000 Tax-exempt municipal interest 10,000 Trustee’s commissions: Income 4,000 Corpus 14,000 Attorney’s fees 12,000 Accountant’s fees 10,000

The trustee allocates the expenses on the basis of income. As a result, the following allocation was made:

Income Expenses

Taxable $15,000 $24,000 Tax-exempt 10,000 16,000 $25,000 $40,000

Excess deductions are computed as follows:

Domestic dividends $15,000 Less: Commissions and fees allocated to taxable income (24,000) Excess deductions for the remaindermen ($9,000)

A completed 2015 Form 1041 based on this fact scenario is presented next.

16 Based on Treas. Regs. §1.642(h)-5.

Page 74: Advanced Selected Issues for Trusts, Estates, and Their ... · 28/07/1997  · 3. Availability of §179 expensing 2 C. Administrative stance 2 1. Estates 2 2. Trusts 3 D. Judicial

cpenow.com / [email protected] Copyright © 2016 Surgent McCoy CPE, LLC -- ITEB/16/01 4-6

Page 75: Advanced Selected Issues for Trusts, Estates, and Their ... · 28/07/1997  · 3. Availability of §179 expensing 2 C. Administrative stance 2 1. Estates 2 2. Trusts 3 D. Judicial

cpenow.com / [email protected] Copyright © 2016 Surgent McCoy CPE, LLC -- ITEB/16/01 4-7

Page 76: Advanced Selected Issues for Trusts, Estates, and Their ... · 28/07/1997  · 3. Availability of §179 expensing 2 C. Administrative stance 2 1. Estates 2 2. Trusts 3 D. Judicial
Page 77: Advanced Selected Issues for Trusts, Estates, and Their ... · 28/07/1997  · 3. Availability of §179 expensing 2 C. Administrative stance 2 1. Estates 2 2. Trusts 3 D. Judicial

cpenow.com / [email protected] Copyright © 2016 Surgent McCoy CPE, LLC -- ITEB/16/01 5-i

Generation-Skipping Transfers

Learning objectives 1 I. The generation-skipping-transfer tax system 1

A. Background 1 1. Estate and gift taxes 1 2. Generation-skipping-transfer tax 3

B. Definitions 3 1. Generation assignment 3 2. Skip person 4 3. Transferor 4 4. Generation-skipping transfer 5 5. Exemption 8 6. Inclusion ratio 8

C. The GST tax 10 1. In general 10 2. Whose tax? 11 3. Applicable rate 11 4. Taxable amount 11 5. Allocation of exemption 13 6. Nontaxable trust transfers 17

D. Calculating the tax 17 1. Taxable terminations 17 2. Taxable distributions 18 3. Direct skips 19

II. Planning to reduce the impact of the GST tax 19 A. Using the GST exemption 19

1. Maximizing the GST exemption for estates under $5,430,000 19 2. Maximizing the GST exemption for estates over $5,430,000 20 3. Allocation of exemption 22 4. Life insurance trusts 24 5. Multiple skips 25 6. Layering 26 7. Retroactive allocation of the GST tax exemption 26 8. Relief from late elections 26 9. Substantial compliance 26

B. Segregation of assets 27 1. In general 27 2. Separate trusts 27 3. Severing of trusts holding property having an inclusion ratio of greater than zero 28 4. Inter vivos trusts 29 5. Transfers at death 29

C. Making choices 30 1. Other transfer taxes 30 2. Using the other tax systems 30 3. Using direct skips 30

D. Handling “grandfathered” trusts 30 1. In general 30 2. Additions 31 3. Effect of an addition 31 4. Planning 31

Page 78: Advanced Selected Issues for Trusts, Estates, and Their ... · 28/07/1997  · 3. Availability of §179 expensing 2 C. Administrative stance 2 1. Estates 2 2. Trusts 3 D. Judicial
Page 79: Advanced Selected Issues for Trusts, Estates, and Their ... · 28/07/1997  · 3. Availability of §179 expensing 2 C. Administrative stance 2 1. Estates 2 2. Trusts 3 D. Judicial

cpenow.com / [email protected] Copyright © 2016 Surgent McCoy CPE, LLC -- ITEB/16/01 5-1

Generation-Skipping Transfers Learning objectives

Upon studying this chapter, you will be able to: • Explain the generation-skipping-transfer tax system; • Calculate the generation-skipping-transfer (GST) tax; • Allocate the generation-skipping-transfer (GST) tax exemption; • Plan to reduce the impact of the generation-skipping-transfer (GST) tax; • Maximize the generation-skipping-transfer (GST) tax exemption; • Use separate trusts or sever trusts to maximize GST savings; and • Make additions to grandfathered trusts.

Note:

The 2001 tax law repealed the estate and generation-skipping taxes effective for decedents dying (or transfers made) after December 31, 2009, and before January 1, 2011. In addition, both estate and generation-skipping transfers benefited from increased exemptions through 2009; the generation-skipping exemption increases to $1.5 million in 2004, $2 million in 2006, and $3.5 million in 2009. Both of these taxes further benefit (from a taxpayer’s point of view) by rate reductions; the highest rate for such transfers in 2002 went from 55 percent to 50 percent and this highest rate was further reduced in succeeding years to 45 percent. The Tax Relief, Unemployment Insurance Reauthorization, and Job Creation Act of 2010 modified both the estate tax and generation-skipping transfers both for 2010 and at least 2011 and 2012, but, as predicted, these changes became permanent with one exception. The American Taxpayer Relief Act of 2012 extended the law permanently with respect to the gift, estate, and generation-skipping-transfer taxes, but modified the tax rates so that the highest effective rate is 40 percent, beginning with taxable transfers in excess of $1,000,000.

I. The generation-skipping-transfer tax system

A. Background

1. Estate and gift taxes

Estate and gift taxes are imposed on transfers from the donor-decedent to the beneficiary. The beneficiary is then subject to estate and gift taxes when the beneficiary transfers the property to someone else. The original donor-decedent could save taxes by reducing the number of transfers subject to these taxes.

Example: Suppose Dad wills $5,450,000 after taxes to his son, who invests the property at 10 percent. Son dies 36 years later with an estate of $168,474,109. Son’s estate will pay estate taxes of $67,389,643.56, leaving Granddaughter $101,084,465. Granddaughter dies 36 years later when her estate is worth $3,124,791,784. Granddaughter’s estate will pay estate taxes of $1,244,466,714, leaving Great-grandson $1,880,325,070. Dad could have saved estate taxes on Son and Granddaughter’s death by leaving the property directly to his great-grandson. By reducing the number of transfers subject to the estate tax, Dad would have saved estate taxes of $1,311,856,357, but the future value is much larger.

Page 80: Advanced Selected Issues for Trusts, Estates, and Their ... · 28/07/1997  · 3. Availability of §179 expensing 2 C. Administrative stance 2 1. Estates 2 2. Trusts 3 D. Judicial

cpenow.com / [email protected] Copyright © 2016 Surgent McCoy CPE, LLC -- ITEB/16/01 5-2

The problem with the above example is that Son and Granddaughter would receive nothing. Alternatively, Dad could have left $5,450,000 in trust to Son where Son would receive the income for life. At Son’s death, nothing would be included in his gross estate, as an income interest in a trust is not an includable asset. Granddaughter would also only receive an income interest in trust. Likewise, nothing would be included in her estate.

Thus, there is an overall savings of $3,327,661,236 ($5,207,986,307 - $1,880,325,070, the future value of the estate-tax savings to the great-grandson). If Dad had not done GST planning, 63.90 percent of his estate would have been lost to taxes as it passed from him to his great-grandson.

Page 81: Advanced Selected Issues for Trusts, Estates, and Their ... · 28/07/1997  · 3. Availability of §179 expensing 2 C. Administrative stance 2 1. Estates 2 2. Trusts 3 D. Judicial

cpenow.com / [email protected] Copyright © 2016 Surgent McCoy CPE, LLC -- ITEB/16/01 5-3

2. Generation-skipping-transfer tax

The generation-skipping-transfer tax (GST) is designed to tax those transfers where no gift or estate tax has been imposed and the transfer either splits the benefits of property between generations or skips over a younger generation to an even younger generation. The generation-skipping-transfer tax applies to gifts, bequests, and distributions from certain trusts.

B. Definitions

1. Generation assignment

Every person other than the transferor must be assigned a generation.1 a. Marriage to transferor -- An individual who has been married at any time to the

transferor is assigned to the transferor’s generation, regardless of the application of any other rule.2

b. Lineal descendants -- An individual who is a lineal descendant of a grandparent of the transferor is assigned to that generation, which results from comparing the number of generations between the grandparent and that individual with the number of generations between the grandparent and the transferor.3 Similarly, an individual who is a lineal descendant of a grandparent of a spouse (or former spouse) of the transferor (other than such spouse) is assigned to that generation resulting from comparing the number of generations between such grandparent and such individual with the number of generations between such grandparent and such spouse.4 • Adoption -- For these purposes, a relationship by adoption or by half-blood is

treated as a relationship by blood.5 • Marriage to lineal descendant -- An individual who has been married at any

time to a lineal descendant is assigned to the generation of the lineal descendant.6

c. Other individuals -- Any other individual is assigned to a generation on the basis of the date of such individual’s birth. • An individual born not more than 12-1/2 years after the date of the birth of the

transferor assigned to the transferor’s generation; • An individual born more than 12-1/2 years but not more than 37-1/2 years after

the date of the birth of the transferor assigned to the first generation younger than the transferor; and

• Similar rules for a new generation every 25 years.7

1 I.R.C. §2651(a). An individual who would be assigned to more than one generation is assigned to the youngest such

generation. 2 I.R.C. §2651(c)(1). 3 I.R.C. §2651(b)(1). 4 I.R.C. §2651(b)(2). 5 I.R.C. §2651(b)(3). 6 I.R.C. §2651(c)(2). 7 I.R.C. §2651(d).

Page 82: Advanced Selected Issues for Trusts, Estates, and Their ... · 28/07/1997  · 3. Availability of §179 expensing 2 C. Administrative stance 2 1. Estates 2 2. Trusts 3 D. Judicial

cpenow.com / [email protected] Copyright © 2016 Surgent McCoy CPE, LLC -- ITEB/16/01 5-4

0 Transferor's date of birth

12-1/2 years

Same generation

37-1/2 years

One generation down

Two generations down

62-1/2 years

2. Skip person

A non-skip person is any person who is not a skip person.8 a. Individuals -- A skip person is a natural person who is at least two generations below the

transferor.9 b. Trusts -- A trust may be a skip person if:

• The trust has no beneficiaries or ultimate distributees that are non-skip persons;10 or

• There is no person holding an interest in such trust, and at no time after the transfer may a distribution (including distributions on termination) be made from such trust to a non-skip person.11

3. Transferor

A transferor is that person who is the donor for gift-tax purposes or the decedent for estate-tax purposes if the transfer is subject to the gift or estate tax.12

Example 1: H creates a marital-deduction trust under will with income to Spouse W for life with the remainder to whomever W shall appoint by will. Since the trust will be subject to tax in W’s estate more recently than in H’s estate, W, and not H, is the transferor with respect to any generation-skipping transfer, which occurs under the power of appointment.

Example 2: H creates a credit-shelter trust under will, which provides income to the spouse

for life with remainder to H’s issue. Since on the death of the spouse the trust is not subject to estate tax, any generation-skipping transfer at that time will be attributed to the transfer under will subject to estate tax (even though any tax is offset by the applicable credit amount) in H’s estate. H is the transferor.

8 I.R.C. §2613(b). 9 I.R.C. §2613(a)(1); Treas. Regs. §26.2612-1(d)(i). 10 I.R.C. §2613(a)(2)(A); Treas. Regs. §26.2612-1(d)(2)(i). 11 I.R.C. §2613(a)(2)(B). The regulations add that no distributions, other than those the probability of which can be

ascertained by actuarial standards to be less than five percent, may be made to any person other than a skip person. Treas. Regs. §26.2612-1(d)(2)(ii).

12 I.R.C. §2652(a)(1); Treas. Regs. §26.2652-1(a)(1).

Page 83: Advanced Selected Issues for Trusts, Estates, and Their ... · 28/07/1997  · 3. Availability of §179 expensing 2 C. Administrative stance 2 1. Estates 2 2. Trusts 3 D. Judicial

cpenow.com / [email protected] Copyright © 2016 Surgent McCoy CPE, LLC -- ITEB/16/01 5-5

a. Split gifts -- Under §2513(a), a husband and wife are permitted to treat a gift as though it was made equally by each of them. For these split-gifts, each spouse is treated for generation-skipping tax purposes as the transferor of one-half of the property.13

Planning point:

Gift-splitting may be an advantageous way of utilizing the generation-skipping exemption as well as the applicable credit amount (formerly known as the unified credit) of the donor’s spouse in cases in which the spouse has few assets. Alternatively, since a transfer to a spouse is not generally subject to gift tax, such transfers can be used once the donor has utilized the full generation-skipping exemption; this permits the spouse to utilize the spouse’s separate exemption.

Example: H transfers $100,000 to a trust for the sole benefit of H’s grandchild. This is a

completed gift and H is the transferor of $100,000. If H elects to gift-split with Spouse W, H and W are the transferors of $50,000 each.

b. QTIP trusts – See discussion later in this module.

4. Generation-skipping transfer

a. General rule -- A “generation-skipping transfer” includes the following: • A direct skip; • A taxable termination; and • A taxable distribution.14

b. Exception -- A generation-skipping transfer does not include: (i) any transfer that, if made inter vivos (during life) by an individual, would not be treated as a taxable gift by reason of §2503(e) (relating to exclusion of certain transfers for educational or medical expenses); or (ii) any transfer to the extent the property transferred was subject to a prior generation-skipping tax and the transferee in the prior transfer was assigned to the same generation as (or a lower generation than) the generation assignment of the transferee in this transfer.15

c. A direct skip is a transfer to a skip person taxable under the estate- or gift-tax rules.16 (i) Orphan step-up rule -- However, if a transfer is made to an individual who is a

grandchild of the transferor (or of the transferor’s spouse or former spouse), and as of the time of the transfer, the parent of such individual who is a lineal descendant of the transferor (or of the transferor’s spouse or former spouse) is dead, such individual shall be treated as if such individual were a child of the transferor and all of that grandchild’s children shall be treated as if they were grandchildren of the transferor. In the case of lineal descendants below a grandchild, the preceding sentence may be reapplied.17 This rule has been extended to collateral heirs. Thus, a transfer to a grandniece or grandnephew of the transferor whose parent predeceased the transfer is treated as a transfer to the niece or nephew, and consequently is not a direct skip or any other generation-skipping transfer provided the transferor had no living lineal descendants at the time of the transfer.

13 I.R.C. §2652(a)(2); Treas. Regs. §26.2652-1(a)(5). 14 I.R.C. §2611(a); Treas. Regs. §26.2611-1. 15 I.R.C. §2611(b). 16 I.R.C. §2612(c)(1); Treas. Regs. §26.2612-1(a)(1). 17 I.R.C. §2612(c)(2); Treas. Regs. §26.2612-1(a)(2)(i).

Page 84: Advanced Selected Issues for Trusts, Estates, and Their ... · 28/07/1997  · 3. Availability of §179 expensing 2 C. Administrative stance 2 1. Estates 2 2. Trusts 3 D. Judicial

cpenow.com / [email protected] Copyright © 2016 Surgent McCoy CPE, LLC -- ITEB/16/01 5-6

Example: G gives grandchild, GC, $100,000. GC’s parent, S, the son of G, is alive. The transfer to GC skips S’s generation. Since it is subject to the gift tax (even if no tax is currently payable by reason of the applicable credit amount), the transfer is a direct skip.

Not direct skip:

Transfer one generation

down

Donor

Donee's Parents

(deceased)

Donee

Orphan Step-up Rule

$

(ii) Skip-person trusts -- If property is transferred to a trust, the transfer is a direct skip only if the trust is a skip person.18 A trust is a skip person only if all of its beneficiaries are skip persons with respect to the transferor.

Skip Person

Trust

Non-Skip Person

Trust

Grandchildrenor other skip persons

Childrenor other non-skip persons

Grandchildrenor other skip persons

(iii) The generation assignment of a lineal descendant of the transferor, spouse, or former spouse of the transferor is determined by disregarding the generation assignment of a predeceased individual who was both an ancestor of the descendant and the lineal descendant of the transferor, spouse, or former spouse of the transferor solely for the purposes of determining whether a transfer is a direct skip.19

18 Treas. Regs. §26.2612-1(a)(1). 19 Treas. Regs. §26.2612-1(a)(2)(i).

Page 85: Advanced Selected Issues for Trusts, Estates, and Their ... · 28/07/1997  · 3. Availability of §179 expensing 2 C. Administrative stance 2 1. Estates 2 2. Trusts 3 D. Judicial

cpenow.com / [email protected] Copyright © 2016 Surgent McCoy CPE, LLC -- ITEB/16/01 5-7

(iv) A trust will be a skip person if all interests in the trust are held by skip persons.20 A trust is also a skip person if: (a) no person holds an interest in the trust; and (b) at no time after the transfer may a distribution (including distributions on termination) be made to a non-skip person.21

Example 1: If T conveys Blackacre to grandchild, GC, the transfer is generally a direct skip.

If, however, GC’s parent, C, T’s child, died prior to the transfer, the transfer is not a direct skip.

Example 2: If T conveyed Blackacre to GGC, T’s great-grandchild, and GC’s parent died

prior to the transfer, the transfer would skip two generations, but for purposes of the GST tax, it is treated as a single direct-skip transfer.

Example 3: Suppose T transfers property to a testamentary trust under which income is

payable to Spouse S for life, with remainder to T’s grandchildren. The transfer is not a direct skip because S has an interest in the trust and S is a non-skip person. If T’s executor makes the QTIP election, S is the transferor and the distribution at S’s death will be a direct skip, subject to the estate tax.

Example 4: T establishes an irrevocable trust that pays income to GC for five years and then

distributes the principal to GC. If child, C, is alive at the time the trust is created, GC is a skip person. In that case, because only a skip person is a beneficiary of the trust, the trust is a skip person. Because the transfer to the trust is a direct transfer to a skip person, the initial transfer would be a direct skip.

However, if, at the time the trust was created, C had predeceased GC, GC is treated as the child of T. For purposes of determining whether the transfer was a direct skip, GC is a non-skip person. Therefore, the trust is not a skip person and the initial transfer is not a direct skip. Distributions during the term and at the final termination of the trust are not generation-skipping transfers.

Now suppose that the trust provided for the payment of income to T’s spouse for a period of two years. T’s spouse’s interest prevents the trust from being a skip person, so the transfer into trust is not a direct skip; when T’s spouse’s interest expires, there is a taxable termination because GC is not treated as T’s child.

d. Taxable termination means a termination by death, lapse of time, release, or otherwise

of an interest in a trust unless: (i) there is gift or estate tax due at the time of the termination for the distributed property at the generation one level above the beneficiary;22 or (ii) immediately after the termination, a non-skip beneficiary has an interest in the trust; or (iii) at no time after the termination may a distribution be made from the trust to a skip person.23 A partial termination is possible if a specified portion is payable to skip beneficiaries after a termination.24 The orphan step-up rule has been expanded to provide that the taxable termination will not be subject to GST with respect to a beneficiary whose parent (the lineal descendant of the transferor) was deceased at the time the transfer to the trust was subject to estate or gift taxes; generally, the transfer would have been subject to estate or gift tax when the trust was funded.25

20 I.R.C. §2613(a)(2)(A). 21 I.R.C. §2613(a)(2)(B). 22 I.R.C. §2612(a)(1); Treas. Regs. §26.2612-1(b)(1)(i). 23 Treas. Regs. §26-2612-1(b)(1)(ii). This is satisfied if no distributions, other than those the probability of which can be

ascertained by actuarial standards to be less than five percent, may be made to any person other than a skip person. Treas. Regs. §26-2612-1(b)(1)(iii).

24 I.R.C. §2612(a)(2); Treas. Regs. §26.2612-1(b)(2). 25 I.R.C. §2651(e).

Page 86: Advanced Selected Issues for Trusts, Estates, and Their ... · 28/07/1997  · 3. Availability of §179 expensing 2 C. Administrative stance 2 1. Estates 2 2. Trusts 3 D. Judicial

cpenow.com / [email protected] Copyright © 2016 Surgent McCoy CPE, LLC -- ITEB/16/01 5-8

Example 1: T establishes an irrevocable trust under which income is paid to T’s child, C, with remainder to grandchild GC. The trust is not a skip person by reason of C’s interest therein. When C dies, a taxable termination occurs because C’s interest terminates and the trust is distributed to a skip person.

Example 2: T’s will establishes an irrevocable trust under which income is paid to T’s living

children until the death of the survivor of T’s children, with the remainder to T’s grandchildren in equal shares. The termination of the trust is a taxable termination subject to GST except to the extent of the distribution to any of T’s grandchildren whose parent (T’s lineal descendant) predeceased T. The trust became subject to estate taxes at T’s death and the orphan step-up rule is tested at that time.

e. Taxable distribution is any distribution (other than a taxable termination or a direct skip)

from a trust to a skip person.26 The payment by the trust of any generation-skipping tax imposed on the distributee is treated as an additional taxable distribution as of the last day of the calendar year for which the original taxable distribution was made.27 The orphan step-up rule has been expanded to provide that the taxable distribution will not be subject to GST with respect to a beneficiary whose parent (the lineal descendant of the transferor) was deceased at the time the transfer to the trust was subject to estate or gift taxes; generally, the transfer would have been subject to estate or gift tax when the trust was funded.28

5. Exemption

For purposes of determining the inclusion ratio, every individual is allowed an exemption of $5,450,000, which may be allocated by the individual (or the individual’s executor) to any property with respect to which the individual is the transferor for GST tax purposes.29 After 1998, the exemption is indexed to inflation in increments of $10,000. The allocation of the exemption is explained below.

6. Inclusion ratio

Generally, the inclusion ratio with respect to any property transferred in a generation-skipping transfer is the excess (if any) of one over the applicable fraction determined for the trust from which such transfer is made or, in the case of a direct skip, one over the applicable fraction determined for such skip.30 The determination is made to the nearest one-thousandth, rounding up.31

Inclusion ratio = 1 - Applicable fraction

a. The numerator of the applicable fraction is the amount of the GST exemption allocated to the trust (or in the case of a direct skip, allocated to the property transferred in such skip), and the denominator is the value of the property transferred to the trust (or involved in the case of a direct skip) reduced by the sum of: (i) any federal estate tax or state death tax actually recovered from the trust attributable to such property; and (ii) any charitable deduction allowed under §§2055 or 2522 with respect to such property; and (iii) in the case of a direct skip, the value that is a nontaxable gift.32 A nontaxable gift is

26 I.R.C. §2612(b). 27 Treas. Regs. §26.2612-1(c)(1). 28 I.R.C. §2651(e). 29 I.R.C. §2631. 30 I.R.C. §2642(a)(1). 31 Treas. Regs. §26.2642-1(a). 32 I.R.C. §2642(a)(2); Treas. Regs. §26.2642-1(c)(1).

Page 87: Advanced Selected Issues for Trusts, Estates, and Their ... · 28/07/1997  · 3. Availability of §179 expensing 2 C. Administrative stance 2 1. Estates 2 2. Trusts 3 D. Judicial

cpenow.com / [email protected] Copyright © 2016 Surgent McCoy CPE, LLC -- ITEB/16/01 5-9

that portion of a transfer not subject to tax by reason of the annual exclusion (§2503(b)) after application of gift-splitting (§2513) and/or by reason of the exclusion for a qualified transfer to an educational institution or medical-care provider (§2503(e)). (i) The amount of the numerator is generally the exemption allocated to the trust (or

to the property in the case of a direct skip).33 (ii) In the case of a taxable termination or taxable distribution made during the

estate-tax inclusion period, the numerator is the sum of: (i) the exemption previously allocated to the trust reduced by the nontaxed portion of any prior generation-skipping transfers; and (ii) any exemption allocated to the trust on a timely return filed after the estate-tax inclusion period.34 • The nontaxed portion of a prior generation-skipping transfer is the

amount of that transfer multiplied by the applicable fraction.35 • The estate-tax inclusion period is the period during which, should death

occur, the value of transferred property would be includable in the gross estate of the transferor or the transferor’s spouse other than by reason of the three-year rule.

Inclusion ratio = 1 - TE VT - (ET CD) TE Transferor’s exemption allocated to this transfer. VT Value of the transferred property. ET Federal estate tax and state death tax recovered from the trust attributable to the

transferred property. CD Any charitable deduction allowed for such property.

Example: T transfers $10,900,000 to a trust for the benefit of T’s children and

grandchildren. T allocates the entire $5,450,000 exemption to the trust. Because T’s children are beneficiaries, the transfer to the trust is not a direct skip. The applicable fraction is $5,450,000/$10,900,000 or one half. The inclusion ratio is 1/2. When the children die, a taxable termination takes place and a generation-skipping-transfer tax would be due. The GST tax equals 40 percent x 1/2 x the taxable amount at the time of termination of the children’s interest.

b. If the denominator of the applicable fraction is zero,36 the inclusion ratio is zero. c. Charitable lead annuity trusts -- For purposes of determining the inclusion ratio for any

charitable lead annuity trust, the applicable fraction is a fraction: • The numerator of which is the adjusted GST exemption; and • The denominator of which is the value of all the property in the trust immediately

after the termination of the charitable lead annuity interest.37 d. The adjusted GST exemption is an amount equal to the GST exemption allocated to the

trust increased by interest determined: • At the interest rate used in determining the amount of the deduction under §2055

or §2522 (as the case may be) for the charitable lead annuity; and • For the actual period of the charitable lead annuity.38

33 Treas. Regs. §26.2642-1(b)(1). 34 Treas. Regs. §26.2642-1(b)(2)(i). 35 Treas. Regs. §26.2642-1(b)(2)(ii). 36 Treas. Regs. §26.2642-1(c)(2). 37 I.R.C. §2642(e)(1); Treas. Regs. §26.2642-3(a). 38 I.R.C. §2642(e)(2); Treas. Regs. §26-2642-3(a).

Page 88: Advanced Selected Issues for Trusts, Estates, and Their ... · 28/07/1997  · 3. Availability of §179 expensing 2 C. Administrative stance 2 1. Estates 2 2. Trusts 3 D. Judicial

cpenow.com / [email protected] Copyright © 2016 Surgent McCoy CPE, LLC -- ITEB/16/01 5-10

e. Generally, for purposes of determining the inclusion ratio, if an individual makes an inter vivos transfer of property, and the value of such property would be includable in the gross estate of such individual if such individual died immediately after making such transfer (other than by reason of §2035), any allocation of GST exemption to such property cannot be made before the close of the estate-tax inclusion period. The term “estate-tax inclusion period” means any period after the transfer during which the value of the property involved in such transfer would be includable in the gross estate of the transferor or the transferor’s spouse if the transferor or the transferor’s spouse died. If the transfer is a direct skip, the skip is treated as occurring as of the close of the estate-tax inclusion period.39 In these cases, the value of such property is its value for purposes of the estate tax if such property is includable in the gross estate of the transferor (other than by reason of §2035); otherwise, the value of the property is its value as of the close of the estate-tax inclusion period (or, if any allocation of GST exemption to such property is not made on a timely filed gift tax return for the calendar year in which such period ends, its value as of the time such allocation is filed with the Secretary).40 The estate-tax inclusion period in no event extends beyond the earlier of: (i) the date on which there is a generation-skipping transfer with respect to such property; or (ii) the date of the transferor’s death.41

Example 1: H creates a $5,450,000 trust with income for H for nine years or H’s earlier death,

with remainder to H’s grandchild, GC. H survives the nine-year term at which time the trust is worth $10,900,000. H allocates the $5,450,000 exemption to the trust on the Form 709 reporting the gift. Because the trust would be included in H’s gross estate (as retained income interest under §2036) if H had died during the nine-year term, the $5,450,000 allocation is ineffective until the expiration of the estate-tax inclusion period. Consequently, the inclusion ratio for the distribution of the trust to GC is half (1 - $5,450,000/$10,900,000) rather than zero (1 - $5,450,000/$5,450,000).

Example 2: H instead creates a $5,450,000 trust with income to his son S for life with

remainder to H’s grandchild, GC. Nine years later when S dies, the trust is worth $10,900,000. Because the trust would not have been included in H’s gross estate at any time after the creation of the trust, the estate-tax inclusion period ends on the date of the initial transfer, and H’s allocation of $5,450,000 exemption to the transfer is effective immediately. Consequently, the inclusion ratio for the distribution of the trust to GC is zero (1 - $5,450,000/$5,450,000).

C. The GST tax

1. In general

A tax is imposed on a generation-skipping transfer made after October 22, 1986.42 The amount of the tax is the product of the taxable amount and the applicable rate.43

39 I.R.C. §2642(f)(1). 40 I.R.C. §2642(f)(2). 41 I.R.C. §2642(f)(3). 42 I.R.C. §2601; Treas. Regs. §26.2601-1(a)(1). 43 I.R.C. §2602.

Page 89: Advanced Selected Issues for Trusts, Estates, and Their ... · 28/07/1997  · 3. Availability of §179 expensing 2 C. Administrative stance 2 1. Estates 2 2. Trusts 3 D. Judicial

cpenow.com / [email protected] Copyright © 2016 Surgent McCoy CPE, LLC -- ITEB/16/01 5-11

2. Whose tax?

a. The person subject to tax depends on the type of generation-skipping transfer.44 (i) Taxable distribution -- The GST tax is paid by the transferee; hence the

payment of the GST tax by the trust is itself a distribution for the benefit of the transferee and an additional taxable distribution.

(ii) Taxable termination or direct-skip trust -- The GST tax is paid by the trustee. (iii) All other direct skips -- The GST tax is paid by the transferor.

b. Apportionment -- Unless otherwise directed pursuant to the governing instrument by specific reference to the generation-skipping tax, the tax imposed on a generation-skipping transfer is charged to the property constituting such transfer.45

3. Applicable rate

The term “applicable rate” means, with respect to any generation-skipping transfer, the product of the maximum federal estate-tax rate and the inclusion ratio with respect to the transfer.46 For these purposes, the term “maximum federal estate-tax rate” means the maximum rate imposed by §2001 on the estates of decedents dying in the year the taxable distribution, taxable termination, or direct skip, as the case may be takes place.47 At present, this means 40 percent even if the transferor is in a lower estate-tax bracket.48

Applicable rate = Inclusion ratio x 40 percent Thus, property with an inclusion ratio of 1.0 would have an applicable rate of 40 percent. Property with an inclusion ratio of 0.0 would have an applicable rate of zero percent.

4. Taxable amount

The amount of property subject to the GST tax is referred to as the “taxable amount.” a. The taxable amount depends on the type of generation-skipping transfer.

(i) Taxable distribution -- The taxable amount is: • The value of the property received by the transferee;49 reduced by • Any expense incurred by the transferee in connection with the

determination, collection, or refund of the tax imposed with respect to such distribution.50

Note:

The taxable distribution will be reported to the beneficiary using Form 706-GS(D-1) and the beneficiary will pay the tax with Form 706-GS(D) and deduct it as a tax not subject to the two-percent-of-AGI limitation.

44 I.R.C. §2603(a). 45 I.R.C. §2603(b). 46 I.R.C. §2641(a). 47 I.R.C. §2641(b). This does not include the surtax on certain estates that phases out the estate-tax brackets lower than

the highest bracket. Treas. Regs. §26.2641-1. 48 Treas. Regs. §26.2641-1(a). 49 For these purposes, if any of the generation-skipping tax imposed with respect to any taxable distribution is paid out of the

trust, an amount equal to the portion so paid is also treated as a taxable distribution. I.R.C. §2621(b). 50 I.R.C. §2621(a).

Page 90: Advanced Selected Issues for Trusts, Estates, and Their ... · 28/07/1997  · 3. Availability of §179 expensing 2 C. Administrative stance 2 1. Estates 2 2. Trusts 3 D. Judicial

cpenow.com / [email protected] Copyright © 2016 Surgent McCoy CPE, LLC -- ITEB/16/01 5-12

(ii) Taxable termination -- The taxable amount is: • The value of all property with respect to which the taxable termination

has occurred;51 reduced by • A deduction similar to the deduction allowed by §2053 (relating to

expenses, indebtedness, and taxes) for amounts attributable to the property with respect to which the taxable termination has occurred.52

Note:

The taxable termination is reported and the trustee pays the tax using Form 706-GS(T) and deducts it as a tax not subject to the two-percent-of-AGI limitation.

(iii) Direct skip -- The taxable amount is the value of the property received by the

transferee.53 b. Generally, property is valued as of the time of the generation-skipping transfer.54

(i) In the case of any direct skip of property included in the transferor’s gross estate, the value of such property is the same as its value for purposes of the estate tax.55

(ii) Simultaneous terminations -- If one or more taxable terminations with respect to the same trust occur at the same time as a result of an individual’s death, an election may be made to value all of the property included in such terminations in accordance with the alternate valuation date under §2032.56

(iii) Consideration furnished by others -- The value of the property transferred is reduced by the amount of any consideration provided by the transferee.57

c. The amount of property necessary to leave a grandchild with $1 million is often bewildering and is different based on whether the property is gifted during life or bequeathed at death, as illustrated by the following.

Direct-skip gifts to grandchildren Amount gifted $1,000,000 GSTT on the gift (assume inclusion ratio is one) $400,000 Gift tax imposed on gift (assume highest bracket) $400,000 Gift tax on GSTT paid (40% x $400,000) $160,000 Amount required for gift $1,960,000

Testamentary bequest (direct skip) to grandchildren

Amount bequeathed $1,000,000 GSTT on bequest (assume inclusion ratio is one) $400,000 Estate tax on bequest (40% x $2,333,333) $933,333 Funds needed for bequest $2,333,333

51 I.R.C. §2622(a). 52 I.R.C. §2622(b). 53 I.R.C. §2623. 54 I.R.C. §2624(a). 55 I.R.C. §2624(b). 56 I.R.C. §2624(c). 57 I.R.C. §2624(d).

Page 91: Advanced Selected Issues for Trusts, Estates, and Their ... · 28/07/1997  · 3. Availability of §179 expensing 2 C. Administrative stance 2 1. Estates 2 2. Trusts 3 D. Judicial

cpenow.com / [email protected] Copyright © 2016 Surgent McCoy CPE, LLC -- ITEB/16/01 5-13

5. Allocation of exemption

a. General rule -- Any allocation by an individual of the individual’s exemption may be made at any time on or before the date prescribed for filing the estate tax return (determined with regard to extensions), regardless of whether such a return is required to be filed.58 If the property is held in trust, the exemption attaches to the entire trust and not to specific assets. If the transfer is a direct skip to a trust, the allocation to the property is treated as an allocation to the trust.59

b. Automatic allocation to lifetime direct skips -- If any individual makes a direct skip during the individual’s lifetime, any unused portion of such individual’s GST exemption is allocated to the property transferred to the extent necessary to make the inclusion ratio for such property zero. If the amount of the direct skip exceeds the unused portion of the transferor’s GST exemption, the entire unused portion is allocated to the property transferred (but not in excess of the fair market value of the property on the date of the transfer).60 (i) Electing out of automatic allocation -- The transferor may prevent the

automatic allocation of GST exemption by describing on a timely filed United

States Gift (and Generation-Skipping Transfer) Tax Return (Form 709) the transfer and the extent to which the automatic allocation is not to apply. In addition, a timely filed Form 709, accompanied by payment of the GST tax (as shown on the return with respect to the direct skip), is sufficient to prevent an automatic allocation of GST exemption with respect to the transferred property.61

(ii) Automatic allocation irrevocable after due date -- The automatic allocation of GST exemption (or the election to prevent the allocation, if made) is irrevocable after the due date. An automatic allocation of exemption is effective as of the date of the transfer to which it relates. Except as provided above, a Form 709 need not be filed to report an automatic allocation. A Form 709 is timely filed if it is filed on or before the date required for reporting the transfer if it were a taxable gift including any extensions to file actually granted.62

(iii) In the case of property transferred to a skip person subject to an estate-tax inclusion period, the direct skip is treated as occurring at the termination of an estate-tax inclusion period.63

c. Allocation for gifts (other than direct and certain indirect skips) -- An allocation of GST exemption to property transferred during the transferor’s lifetime, other than in a direct skip, is made on Form 709. The allocation must clearly identify the trust to which the allocation is being made, the amount of GST exemption allocated to it, and the value of the trust assets at the effective date of the allocation if the allocation is late or if an inclusion ratio is greater than zero. The allocation should also state the inclusion ratio of the trust after the allocation. An allocation of GST exemption may generally be made by a formula.

58 I.R.C. §2632(a); Treas. Regs. §26.2632-1(a). 59 Treas. Regs. §26.2632-1(a). 60 Treas. Regs. §26.2632-1(b)(1)(i). 61 Treas. Regs. §26.2632-1(b)(1)(i). 62 Treas. Regs. §26.2632-1(b)(1)(ii). 63 Treas. Regs. §26.2632-1(c)(4).

Page 92: Advanced Selected Issues for Trusts, Estates, and Their ... · 28/07/1997  · 3. Availability of §179 expensing 2 C. Administrative stance 2 1. Estates 2 2. Trusts 3 D. Judicial

cpenow.com / [email protected] Copyright © 2016 Surgent McCoy CPE, LLC -- ITEB/16/01 5-14

Planning point:

Many estate planners use formula GST allocations. This type of formula allocation prevents the IRS from revaluing the transfer at a later date and claiming an inclusion ratio of other than zero. The allocation is often expressed in terms of the amount necessary to produce an inclusion ratio of zero. However, formula allocations made with respect to charitable lead annuity trusts are not valid except to the extent they are dependent on values as finally determined for federal estate- or gift-tax purposes.64

(i) With respect to a timely allocation, an allocation of GST exemption becomes

irrevocable after the due date of the return. Except for charitable lead annuity trusts, an allocation of GST exemption to a trust is void to the extent the amount allocated exceeds the amount necessary to obtain an inclusion ratio of zero with respect to the trust.

(ii) An allocation is also void if the allocation is made with respect to a trust that has no GST potential with respect to the transferor making the allocation, at the time of the allocation. For this purpose, a trust has GST potential even if the possibility of a GST is so remote as to be negligible.65

d. Effective date of allocations -- Generally, an allocation of exemption is effective as of the date of any transfer as to which the Form 709 on which it is made is a timely filed return (a timely allocation). Generally, an allocation to a trust made on a Form 709 filed after the due date for reporting a transfer to the trust (a late allocation) is effective on the date the Form 709 is filed and deemed to precede in point of time any taxable event occurring on such date. The Form 709 is deemed filed on the date it is postmarked. If it is unclear whether an allocation of exemption on a Form 709 is a late or a timely allocation to a trust, the allocation is effective in the following order:66 • To any transfer to the trust disclosed on the return as to which the return is a

timely return; • As a late allocation; and • To any transfer to the trust not disclosed on the return as to which the return

would be a timely return. If other transfers exist with respect to which the exemption could be allocated, any exemption allocated to a trust transfer disclosed on a timely return is allocated in an amount equal to the value of the transferred property as reported on the Form 709. Thus, if the GST exemption allocated on the Form 709 exceeds the value of the transfers reported on that return that have generation-skipping potential, the initial allocation is the amount of the value of those transfers as reported on that return. Any remaining amount of exemption allocated on that return is allocated to the late allocation and a transfer to the trust not disclosed on the return as to which the return would be a timely return, notwithstanding any subsequent upward adjustment in value of the transfers reported on the return.67

Example 1: T transfers $100,000 to an irrevocable generation-skipping trust on December 1,

2016. The transfer to the trust is not a direct skip. The date prescribed for filing the gift tax return reporting the taxable gift is April 15, 2017. On February 10, 2017, T files a Form 709 allocating $50,000 of GST exemption to the trust. On

64 Treas. Regs. §26.2632-1(b)(2)(i). 65 Treas. Regs. §26.2632-1(b)(2)(i). 66 Treas. Regs. §26.2632-1(b)(2)(ii)(A). 67 Treas. Regs. §26.2632-1(b)(2)(ii)(B).

Page 93: Advanced Selected Issues for Trusts, Estates, and Their ... · 28/07/1997  · 3. Availability of §179 expensing 2 C. Administrative stance 2 1. Estates 2 2. Trusts 3 D. Judicial

cpenow.com / [email protected] Copyright © 2016 Surgent McCoy CPE, LLC -- ITEB/16/01 5-15

April 10 of the same year, T files an amended Form 709 allocating $100,000 of GST exemption to the trust in a manner that clearly indicates the intention to modify and supersede the prior allocation with respect to the 2016 transfer. The allocation made on the April 10 return supersedes the prior allocation because it is made on a timely filed Form 709 that clearly identifies the trust and the nature and extent of the modification of GST exemption allocation. The allocation of $100,000 of GST exemption to the trust is effective as of December 1, 2016. The result would be the same if the amended Form 709 decreased the amount of the GST exemption allocated to the trust.

Example 2: The facts are the same as in Example 1, except on July 10, 2017, T files a Form

709 attempting to reduce the earlier allocation. The return is not a timely filed return. The $100,000 GST exemption allocated to the trust, as amended on April 10, 2017, remains in effect because an allocation, once made, is irrevocable and may not be modified after the last date on which a timely filed Form 709 can be filed.

Example 3: T transfers $100,000 to an irrevocable generation-skipping trust on December 1,

2016. The transfer to the trust is not a direct skip. The date prescribed for filing the gift tax return reporting the taxable gift is April 15, 2017. On December 1, 2017, T files a Form 709 and allocates $50,000 to the trust. The allocation is effective as of December 1, 2017.

Example 4: T transfers $100,000 to a generation-skipping trust on December 1, 2016, in a

transfer that is not a direct skip. T does not make an allocation of GST exemption on a timely filed Form 709. On July 1, 2017, the trustee makes a taxable distribution from the trust to T’s grandchild in the amount of $30,000. Immediately prior to the distribution, the value of the trust assets was $150,000. On the same date, T allocates GST exemption to the trust in the amount of $50,000. The allocation of GST exemption on the date of the transfer is treated as preceding in point of time the taxable distribution. At the time of the GST, the trust has an inclusion ratio of 0.6667 (1 - (50,000/150,000)).

Example 5: On May 15, 2016, T transfers $50,000 to a trust in a direct skip. T does not file a

timely gift tax return electing out of the automatic allocation. On April 30, 2017, T and T’s spouse, S, file an initial gift tax return for 2016 on which they consent, pursuant to §2513, to have the gift treated as if one-half had been made by each. As a result of the election under §2513, which is retroactive to the date of T’s transfer, T and S are each treated as the transferor of one-half of the property transferred in the direct skip. Thus, $25,000 of T’s unused GST exemption and $25,000 of S’s unused GST exemption is automatically allocated to the trust. Both allocations are effective on and after the date that T made the transfer.

e. The unused portion of an individual’s GST exemption is that portion of the exemption that

had not previously been allocated by such individual (or treated as allocated with respect to a prior direct skip).68

f. Deemed allocation -- Any portion of an individual’s GST exemption that had not been allocated within the time for the filing of the gift-tax return is deemed to be allocated as follows:69 • To property that is the subject of a direct skip occurring at such individual’s

death; then • To trusts with respect to which such individual is the transferor and from which a

taxable distribution or a taxable termination might occur at or after such individual’s death.

68 I.R.C. §2632(b)(2). 69 I.R.C. §2632(c).

Page 94: Advanced Selected Issues for Trusts, Estates, and Their ... · 28/07/1997  · 3. Availability of §179 expensing 2 C. Administrative stance 2 1. Estates 2 2. Trusts 3 D. Judicial

cpenow.com / [email protected] Copyright © 2016 Surgent McCoy CPE, LLC -- ITEB/16/01 5-16

The allocation is made among the properties and the trusts, as the case may be, in proportion to the respective amounts (at the time of allocation) of the nonexempt portions of such properties or trusts. The term “nonexempt portion” means the value (at the time of allocation) of the property or trust, multiplied by the inclusion ratio with respect to such property or trust.

g. Generally, an allocation of a decedent’s unused exemption by the executor of the decedent’s estate is made on the appropriate, United States Estate (and Generation-

Skipping Transfer) Tax Return (Form 706 or Form 706NA), filed on or before the date prescribed for filing the return (including any extensions actually granted [the due date]). An allocation of GST exemption with respect to property included in the gross estate of a decedent is effective as of the date of death. A timely allocation of GST exemption by an executor with respect to a lifetime transfer of property that is not included in the transferor’s gross estate is made on a Form 709. A late allocation of GST exemption by an executor, other than an allocation that is deemed to be made under §2632(b)(1), with respect to a lifetime transfer of property, is made on Form 706 or Form 706NA and is effective as of the date the allocation is filed. An allocation of GST exemption to a trust (whether or not funded at the time the Form 706 or Form 706NA is filed) is effective if the notice of allocation clearly identifies the trust and the amount of the decedent’s GST exemption allocated to the trust. An executor may allocate the decedent’s GST exemption by use of a formula. For purposes of this section, an allocation is void if the allocation is made to a trust that has no GST potential with respect to the transferor for whom the allocation is being made, as of the date of the transferor’s death. For this purpose, a trust has GST potential even if the possibility of a GST is so remote as to be negligible.70 If no allocation is made, an automatic allocation occurs pro rata based on the value of the property as finally determined for estate-tax purposes.71

Note:

The 2001 tax law made most indirect skips subject to automatic allocation, which has now been extended permanently. An indirect skip is any transfer of property (other than a direct skip) subject to the gift tax (or subject to the estate-tax inclusion period) made to a GST trust.72 The definition of a GST trust is broadly defined as a trust that could have a generation-skipping transfer with respect to its transferor. There are exceptions, but in most cases they will not apply. And, to make matters worse, the effect is to allocate exemption in many of the more common instances where, although the transferor still has generation-skipping exemption left to allocate, a transferor will not want to allocate, such as where a trust is likely to terminate without any generation-skipping transfer. Therefore, added attention must be given to the impact of transfers for purposes of filing Form 709. In the past, practitioners could leave the GST allocation blank, knowing that no allocation would be made if they did not make it. Now it is necessary to develop the same sensitivity to electing out of automatic allocation as was necessary for direct skips. The taxpayer may make an election with respect to one or more trusts so as to insulate the transfers to those identified trusts in later years from the automatic-allocation provisions.

70 Treas. Regs. §26.2632-1(d)(1). 71 Treas. Regs. §26.2632-1(d)(2). 72 I.R.C. §2632(c).

Page 95: Advanced Selected Issues for Trusts, Estates, and Their ... · 28/07/1997  · 3. Availability of §179 expensing 2 C. Administrative stance 2 1. Estates 2 2. Trusts 3 D. Judicial

cpenow.com / [email protected] Copyright © 2016 Surgent McCoy CPE, LLC -- ITEB/16/01 5-17

h. Another provision of the 2001 tax law -- continued permanently -- permitted a transferor to elect to make a retroactive allocation of unused GST exemption to a previous transfer in a trust if a beneficiary of the trust is a child, niece, nephew, or child of a first cousin of the transferor, transferor’s spouse, or transferor’s former spouse if that beneficiary dies before the transferor.73 (i) A transferor generally will not make the allocation to a trust where it is anticipated

that a non-skip person, such as the transferor’s child, will receive all benefits from the trust. However, if the child dies while the transferor is alive, the ultimate termination of the trust will in most cases be in favor of grandchildren, i.e., skip persons, transforming the termination into an unanticipated taxable termination. The retroactive allocation permits the transferor to do what would have been done had it been known at the time of the transfer that the trust would ultimately result in a generation-skipping transfer.

(ii) Such allocation must be made on a timely filed gift tax return for the year of the non-skip person’s death, the amount of the unused exemption being determined immediately before the death of the non-skip person and the allocation also being effective immediately before the death of the non-skip person, using the gift-tax value as though it had been a timely allocation.

6. Nontaxable trust transfers

a. General rule -- Section 2503(c) trusts and Crummey withdrawal trusts qualify for the annual exclusion for gift-tax purposes. As a general rule, annual exclusion gifts also qualify for an equal annual GST tax exclusion.

b. Exception -- Transfers in trust are not eligible for an annual GST tax exclusion ($14,000, or $28,000 if gift-split) unless:74 • The trust benefits one individual; • No portion of the income or corpus may be distributed to anyone else during the

beneficiary’s lifetime; and • The corpus will be included in the beneficiary’s estate if the beneficiary dies

before the termination of the trust. c. The exception above effectively (if not completely) eliminates a multiple-beneficiary

Crummey power life-insurance trust for GST tax exemption.

D. Calculating the tax

1. Taxable terminations

For taxable terminations, the taxable amount is the amount of property with respect to which the termination occurred.75 The termination also occurs with respect to that property of the trust that is used to pay the tax. The trust is responsible for the generation-skipping-transfer tax.76

Example: Under Decedent’s will, the amount of $200,000 passes to a trust, inclusion ratio equal to one, according C, Decedent’s child, a life estate. The remainder goes to GC, C’s child (Decedent’s grandchild) with distribution on C’s death to C’s child,

73 I.R.C. §2632(d). This also applies to a descendant of a child, niece, or nephew who is treated as a member of the same

generation as the child, nephew, or niece by reason of the predeceased-parent rule. 74 I.R.C. §2642(c)(2). 75 I.R.C. §2622(a). This is reduced for certain deductions attributable to the property with respect to which the termination

has occurred. See I.R.C. §2622(b). 76 I.R.C. §2603(a)(2).

Page 96: Advanced Selected Issues for Trusts, Estates, and Their ... · 28/07/1997  · 3. Availability of §179 expensing 2 C. Administrative stance 2 1. Estates 2 2. Trusts 3 D. Judicial

cpenow.com / [email protected] Copyright © 2016 Surgent McCoy CPE, LLC -- ITEB/16/01 5-18

GC. The estate taxes properly apportioned to the trust are at a 40-percent rate, or $80,000, which leaves a net value in the trust of $120,000. When C dies, a taxable termination occurs. If the value in the trust at that time is $120,000, the generation-skipping-transfer tax is $48,000, leaving a net of $72,000.

2. Taxable distributions

The taxable amount of a taxable distribution is the amount received by the transferee, reduced by any expense incurred by the transferee in connection with the determination, collection, or refund of the tax imposed with respect to such distribution.77 The transferee is liable for the generation-skipping tax on taxable distributions. Because the transferee is liable for the related generation-skipping-transfer tax, there is a further taxable distribution to the extent the tax is paid out of the trust on a taxable distribution.78 If any portion of generation-skipping tax (including penalties and interest thereon) imposed on a transferee is paid from the distributing trust, the payment is an additional taxable distribution to the transferee.79 The additional distribution is treated as having been made on the last day of the calendar year in which the original taxable distribution is made (even though the actual payment occurs in the next calendar year when the tax is due).

Example: T creates a trust with an inclusion ratio of one. A taxable distribution is made to T’s grandchild, GC, in 2016 of $100,000. The applicable rate is 40 percent. GC is liable for the generation-skipping tax of $40,000, leaving GC with a net of $60,000.

Instead, the trustee pays the GST tax in the following year. What is the amount of the taxable distribution? Because the additional distribution is treated as having been made on the last day of the calendar year in which the original distribution was made even though the actual payment occurs in the next calendar year when the tax is due, the net amount received by the distributee remains $100,000, but the taxable amount must gross up the GST tax (X), or ($100,000 + X) (X represents the unknown GST tax); the GST tax is 0.4 ($100,000 + X), or $40,000 + 0.4 X. This equals the unknown X. Simplifying, 0.6X = $40,000, or X= $66,666.67. The taxable amount is $166,666.67, with respect to which the GST tax is $66,666.67.

Alternatively, early in the following year, the trustee makes an additional distribution to GC in the amount of $40,000 for GC to pay the GST tax. This is another taxable distribution, except in the year following the year of the $100,000 taxable distribution, subject to a GST tax in the amount of $16,000 (0.4 x $40,000). In the following year, the trustee repeats the process by making a taxable distribution of $16,000, triggering a GST liability in the amount of $6,400.00.

Planning point:

The only time when the second method -- annual additional taxable distributions -- results in an aggregate distribution out of the trust equal to the GST tax paid by the trust under the first method is after an infinite number of years; and taking into account the time-value of money, the first method only works when there is significant deflation over a period significant in relation to the rate of monetary deflation.

77 I.R.C. §2621(a). 78 I.R.C. §2621(b). 79 Treas. Regs. §26.2612-1(c)(1).

Page 97: Advanced Selected Issues for Trusts, Estates, and Their ... · 28/07/1997  · 3. Availability of §179 expensing 2 C. Administrative stance 2 1. Estates 2 2. Trusts 3 D. Judicial

cpenow.com / [email protected] Copyright © 2016 Surgent McCoy CPE, LLC -- ITEB/16/01 5-19

3. Direct skips

The prototypical direct skip is a transfer made during the transferor’s lifetime to a skip person. The generation-skipping tax is a liability of the transferor, based on the value of the gift, but in addition to the gift tax and the generation-skipping tax on the transfer, gift tax must also be paid on the generation-skipping-transfer tax paid by the transferor.80 Put another way, the gift-tax base is the sum of the value of the property transferred and the generation-skipping tax on such transfer.

Example 1: T, who already has made gifts and generation-skipping transfers in excess of $5,450,000, transfers $1,000,000 to T’s grandchild, GC, against which T has no exemption to apply. The generation-skipping-transfer tax is 40 percent of the $1,000,000 transferred in the direct skip to GC, and the amount of the transfer for gift-tax purposes is $1,400,000 ($1,000,000 property + $400,000 generation-skipping-transfer tax). Since T is already in the 40-percent rate bracket with respect to lifetime transfers, the gift tax is $560,000. The combined transfer tax on the transfer of $1,000,000 is $960,000.

The tax is imposed only on the amount received and does not include the generation-skipping tax as part of the value subject to the generation-skipping tax.

Example 2: T leaves the residue of T’s estate to T’s grandchild, GC, which after the payment of all estate taxes, but not the generation-skipping tax, amounts to $1,000,000. This results in a generation-skipping direct skip of ($1,000,000 - GST) and an inclusion ratio of 1.0 (as a result of lifetime application of the exemption). The GST tax is 0.4 x ($1,000,000 - GST).

GST = $400,000 - 0.4 x GST 1.4 x GST = $400,000 GST = $285,714

Example 3: The facts are the same as Example 2, except the residue is $1,500,000 and no

exemption was allocated during lifetime. The generation-skipping transfer is $1,500,000 - GST, and the generation-skipping-transfer tax is 0.4 x ($1,500,000 - GST).

GST = $600,000 - 0.4 x GST 1.4 x GST = $600,000 GST = $428,571

II. Planning to reduce the impact of the GST tax

A. Using the GST exemption

1. Maximizing the GST exemption for estates under $5,450,000

If a couple has less than $5,450,000 in assets, the wealthier spouse should make transfers that allow the less wealthy spouse to utilize the less wealthy spouse’s GST exemption.

Note:

There is no portability of GST exemption unlike the portability of the estate and gift tax applicable exclusion amount. Thus, when a spouse dies, any unused exemption amount dies with that spouse. Therefore, there will still be pressure to make inter vivos transfers to enable the poorer spouse to fully utilize GST exemption.

80 I.R.C. §2515.

Page 98: Advanced Selected Issues for Trusts, Estates, and Their ... · 28/07/1997  · 3. Availability of §179 expensing 2 C. Administrative stance 2 1. Estates 2 2. Trusts 3 D. Judicial

cpenow.com / [email protected] Copyright © 2016 Surgent McCoy CPE, LLC -- ITEB/16/01 5-20

a. The wealthier spouse can make taxable transfers to their children or grandchildren either in trust or outright, and allow the less wealthy spouse to gift split and use the less wealthy spouse’s exemptions.

b. Since the couple is probably concerned about their own financial security, the wealthier spouse can transfer property to the spouse outright or to a marital-deduction trust. The wealthier spouse must make the gift-tax QTIP election in order to avoid gift tax. However, the wealthier spouse should not make the GST-tax reverse-QTIP election so that the less wealthy spouse will be the transferor for GST tax purposes, allowing the use of the less wealthy spouse’s GST tax exemption.

2. Maximizing the GST exemption for estates over $5,450,000

a. Lifetime transfers -- If a couple’s combined estate exceeds $5,450,000, the wealthier spouse should take advantage of such spouse’s GST exemption by transferring assets to skip persons or to a trust benefiting skip persons. By making these transfers during life, the allocation of the GST exemption will shelter the future appreciation in the assets. However, if income is paid to a non-skip person, such as the transferor’s spouse, a portion of the GST exemption is wasted.

b. For qualified terminable interest property (QTIP) trusts, the donor or decedent’s executor can elect to have the donor or decedent to be treated as the transferor for GST tax purposes.81 In the ordinary case, the transfer by gift or bequest does not constitute a generation-skipping transfer at that time because the spouse is presumed to be in the same generation as the donor or decedent. When the spouse dies, the trust is subject to estate tax and a generation-skipping transfer may then take place for which the spouse is the transferor. By special election only with respect to QTIP trusts, the donor or the decedent’s executor may “reverse QTIP” for purposes of the GST tax, which permits the donor or decedent to be treated as the transferor. This election is irrevocable, and can only be applied to all of the property in the QTIP trust.82 This election is made at the time of and on the same form as the QTIP election.83 The difference in the identity of the transferor can also impact the character of the distribution on termination of the trust as a direct skip or taxable termination.

Example: H transfers an amount in trust for W that provides all income to the spouse for life

and H makes a §2523(f) election to treat the trust as a QTIP. The transfer is subject to estate tax at W’s death, because the trust is included in W’s estate and W is the transferor for generation-skipping-transfer purposes. However, if the reverse-QTIP election is made by H, H can be treated as transferor. If such trust was created under will, the QTIP election for estate-tax purposes would be made by H’s executor, and W would be treated as transferor for generation-skipping-transfer purposes. If H’s executor makes the reverse-QTIP election, the trust will continue to be included in W’s estate for estate-tax purposes, but H will be considered the transferor for generation-skipping tax purposes.

81 I.R.C. §2652(a)(3); Treas. Regs. §26.2652-1(a)(3). 82 Treas. Regs. §26.2652-2(a). 83 Treas. Regs. §26.2652-2(b).

Page 99: Advanced Selected Issues for Trusts, Estates, and Their ... · 28/07/1997  · 3. Availability of §179 expensing 2 C. Administrative stance 2 1. Estates 2 2. Trusts 3 D. Judicial

cpenow.com / [email protected] Copyright © 2016 Surgent McCoy CPE, LLC -- ITEB/16/01 5-21

Planning point:

One of the primary factors in making a reverse-QTIP election is to make sure that the full benefit of the decedent and spouse’s exemption is made. In estates where generation-skipping is an important consideration, a QTIP has advantages over both an outright disposition and a general-power-of-appointment trust, because the election is available only to QTIPs. The exemption is discussed below. The transferor may wish to have two marital trusts, both QTIPs, one in the amount of the difference between the generation-skipping-transfer exemption and the exemption equivalent of the applicable credit amount. Since the decedent is the transferor of the credit-shelter trust, a reverse-QTIP election as to the one QTIP trust permits full use of the exemption.

c. Reverse-QTIP marital deduction plan -- Many clients have a marital-deduction plan

deferring the federal estate tax to the surviving spouse’s estate and maximizing each spouse’s applicable credit amount. However, many clients are also in need of planning for the GST tax. These clients will want to establish trusts that allow them to utilize each of their $5,450,000 GST exemptions. At the first spouse’s death, two trusts are ordinarily created: (i) the credit-shelter trust; and (ii) the QTIP marital trust. The first spouse’s GST exemption is allocated to the credit-shelter trust, approximately $5,450,000. Because the exemptions are now equal, there is no excess that would have to be allocated to the QTIP marital trust because the return to pre-2002 law has been permanently repealed.

Note:

The effect of the equality of the GST exemption with the estate-tax exemption amount is actually somewhat more complex. Taxable gifts made to non-skip persons will reduce the available estate-tax exemption at death (and the amount allocated to the credit-shelter trust) but will not reduce the GST exemption. Consequently, wills continue to contain reverse-QTIP provisions for this eventuality. For example, if the decedent made $1,500,000 of taxable gifts to children, then if the decedent dies when the applicable exclusion amount is $5,000,000, only $3,500,000 will be allocated to the credit-shelter trust (and $1,500,000 more to the marital-deduction trust). The executor will allocate $3,500,000 of GST exemption to the credit-shelter trust and the remaining $1,500,000 of GST exemption to a portion of the QTIP trust for which the reverse-QTIP election is made.

(i) One QTIP marital trust -- The problem with allocating to the QTIP marital trust is

that the inclusion ratio of the trust is less than 1.0 and, more importantly, the surviving spouse will be the transferor of the QTIP trust for GST tax purposes when the surviving spouse dies. A transferor for GST tax purposes is the person who makes a transfer that is subject to gift or estate tax. In this case, the surviving spouse is taxed for estate-tax purposes on the QTIP trust and as a result becomes the transferor for GST tax purposes.

(ii) Two QTIP marital trusts -- Most estate planners will recommend two QTIP marital trusts. The first QTIP will be funded with assets equal to the unused GST exemption (referred to as the exempt-QTIP trust). The second QTIP will be funded with the remaining assets of the estate (nonexempt-QTIP trust). The executor of the first spouse’s estate will make a reverse-QTIP election on the exempt-QTIP trust, which will result in the first spouse being the transferor of that trust for GST tax purposes. As to the nonexempt-QTIP trust, the surviving spouse will be treated as the transferor for GST tax purposes. Thus, the inclusion ratio of the exempt-QTIP trust is 0.0 and the inclusion ratio of the nonexempt-QTIP trust is 1.0.

Page 100: Advanced Selected Issues for Trusts, Estates, and Their ... · 28/07/1997  · 3. Availability of §179 expensing 2 C. Administrative stance 2 1. Estates 2 2. Trusts 3 D. Judicial

cpenow.com / [email protected] Copyright © 2016 Surgent McCoy CPE, LLC -- ITEB/16/01 5-22

• To maximize the effectiveness of this strategy, all invasions of principal for the benefit of the surviving spouse should first be made out of the nonexempt-QTIP trust before any principal invasions are made from the exempt-QTIP trust. For GST tax purposes, it is always better to make distributions to non-skip persons (i.e., the surviving spouse) from trusts that have an inclusion ratio of 1.0.

• For similar reasons, it is advantageous that any estate taxes that become due on the death of the surviving spouse attributable to the inclusion of the exempt-QTIP trust in the surviving spouse’s estate should not be payable out of the exempt-QTIP trust. Under the regulations, the failure to exercise the right of recovery against the exempt-QTIP trust84 is not treated as an addition to the trust that would require a re-computation of the trust’s inclusion ratio.85

d. The reverse-QTIP election affects the type of generation-skipping transfer. For example, if following the surviving spouse’s interest, the exempt and nonexempt QTIP trusts distribute the trust to a grandchild, the termination of the exempt QTIP will result in a taxable termination since the decedent remains the transferor, while the nonexempt QTIP will be treated as a direct skip by the surviving spouse.86

e. If the inclusion ratio in a reverse QTIP is not zero, a reverse QTIP may be disadvantageous. (i) The amount of the taxable termination includes the amount of the tax. (ii) If a grandchild’s parent should die during the life of the surviving spouse, the

transfer from the nonexempt QTIP trust for which the spouse is the transferor is no longer a direct skip, while the distribution from the exempt QTIP remains a taxable termination.

3. Allocation of exemption

a. If an individual makes a direct skip during his or her lifetime, any unused generation-skipping-transfer tax exemption is automatically allocated to a direct skip to the extent necessary to make the inclusion ratio for such property equal to zero. An individual can elect out of the automatic allocation for lifetime direct skips. Any unused generation-skipping-transfer tax exemption, which has not been allocated to transfers made during an individual’s life, is automatically allocated on the due date for filing the decedent’s estate tax return. Unused generation-skipping-transfer tax exemption is allocated pro rata on the basis of the value of the property as finally determined for estate-tax purposes, first to direct skips treated as occurring at the transferor’s death. The balance, if any, of unused generation-skipping-transfer tax exemption is allocated pro rata, on the basis of the estate-tax value of the nonexempt portion of the trust property (or in the case of trusts that are not included in the gross estate, on the basis of the date-of-death value of the trust) to trusts with respect to which a taxable termination may occur or from which a taxable distribution may be made.

84 I.R.C. §2207A. 85 Treas. Regs. §26.2652-1(a)(6), Examples 7 and 8; see also Treas. Regs. §26.2652-1(a)(3). 86 Rev. Rul. 92-26, 1992-1 C.B. 314.

Page 101: Advanced Selected Issues for Trusts, Estates, and Their ... · 28/07/1997  · 3. Availability of §179 expensing 2 C. Administrative stance 2 1. Estates 2 2. Trusts 3 D. Judicial

cpenow.com / [email protected] Copyright © 2016 Surgent McCoy CPE, LLC -- ITEB/16/01 5-23

b. GST tax exemption is automatically allocated to transfers made during life that are “indirect skips.” An indirect skip is any transfer of property (that is not a direct skip) subject to the gift tax that is made to a GST trust. A GST trust is defined as a trust that could have a GST with respect to the transferor (e.g., a taxable termination or taxable distribution), unless it is one of the following: (i) The trust instrument provides that more than 25 percent of the trust corpus must

be distributed to, or may be withdrawn by, one or more individuals who are non-skip persons: (a) before the date that the individual attains age 46; or (b) on or before one or more dates specified in the trust instrument that will occur before the date that such individual attains age 46; or (c) upon the occurrence of an event that, in accordance with regulations prescribed by the Treasury secretary, may reasonably be expected to occur before the date that such individual attains age 46.

(ii) The trust instrument provides that more than 25 percent of the trust corpus must be distributed to or may be withdrawn by one or more individuals who are non-skip persons and who are living on the date of death of another person identified in the instrument (by name or by class) who is more than 10 years older than such individuals.

(iii) The trust instrument provides that if one or more individuals who are non-skip persons die on or before a date or event described in clause (i) or (ii), more than 25 percent of the trust corpus must either be distributed to the estate or estates of one or more of such individuals or it is subject to a general power of appointment exercisable by one or more of such individuals.

(iv) The trust is a trust that any portion of which would be included in the gross estate of a non-skip person (other than the transferor) if such person died immediately after the transfer.

(v) The trust is a charitable lead annuity trust, a charitable remainder annuity trust, or a charitable unitrust; or the trust is a trust with respect to which a deduction was allowed under §2522 for the amount of an interest in the form of the right to receive annual payments of a fixed percentage of the trust property’s net fair market value (determined yearly), and which is required to pay principal to a non-skip person if such person is alive when the yearly payments for which the deduction was allowed, terminate.

Note:

If any individual makes an indirect skip during the individual’s lifetime, then any unused portion of such individual’s GST tax exemption is allocated to the property transferred to the extent necessary to produce the lowest possible inclusion ratio for such property.

Planning point:

An individual may elect not to have the automatic-allocation rules apply to an indirect skip, and such elections will be deemed timely if filed on a timely filed gift tax return for the calendar year in which the transfer was made or deemed to have been made, or on such later date or dates as may be prescribed by the Service. An individual may elect not to have the automatic-allocation rules apply to any or all transfers made by such individual to a particular trust and may elect to treat any trust as a GST trust with respect to any or all transfers made by the individual to such trust, and such election may be made on a timely filed gift tax return for the calendar year for which the election is to become effective.

Page 102: Advanced Selected Issues for Trusts, Estates, and Their ... · 28/07/1997  · 3. Availability of §179 expensing 2 C. Administrative stance 2 1. Estates 2 2. Trusts 3 D. Judicial

cpenow.com / [email protected] Copyright © 2016 Surgent McCoy CPE, LLC -- ITEB/16/01 5-24

Note:

In connection with timely and automatic allocations of GST tax exemption, the value of the property for purposes of determining the inclusion ratio shall be its finally determined gift-tax value or estate-tax value, depending on the circumstances of the transfer. In the case of a GST tax exemption allocation deemed made at the conclusion of an estate-tax inclusion period, the value for purposes of determining the inclusion ratio shall be its value at that time.

4. Life insurance trusts

The annual exclusion for GST tax purposes only applies to outright transfers that are direct skips, and transfers in trust for the exclusive benefit of one individual where the trust will be included in the individual’s gross estate if the individual dies before the termination of the trust. Thus, in a typical multiple-beneficiary Crummey life insurance trust, the payment of the annual premium is not subject to gift tax, but will require an allocation of GST exemption if the life insurance trust is to have a zero inclusion ratio. Most estate planners recommend using the GST exemption in this manner because of the leverage that can be achieved. For example, a donor can place a $1 million policy in a trust that will be paid-up with ten $15,000 annual premium contributions. The grandparent can then shelter the transfer from GST taxes forever by using $150,000 of their GST exemption. Thus, the donor would be able to shelter $1 million of total property from GST taxes -- almost a 7:1 leverage.

Note:

A direct skip is automatically allocated GST exemption unless the donor specifically elects out of the allocation. The payment of premiums to a trust (all of the current beneficiaries of which are skip persons) would be a direct skip. Failure to elect out by the time the gift tax return, including actual extensions granted, is due will trigger the automatic allocation. Any election out is irrevocable. If the insurance trust is not a skip person because there are non-skip person beneficiaries, then the premiums might be automatically allocated unless exempted as above.

a. The transferee for GST tax purposes is the trust, not the individuals holding the Crummey

withdrawal powers.87 b. In order to qualify the contribution to the trust as a present interest for gift-tax purposes,

each beneficiary of the trust is given a Crummey right of withdrawal, which may be as much as $14,000 ($28,000 if gift-split). If a Crummey withdrawal power is allowed to lapse, the Crummey beneficiary has made a taxable gift to the extent the withdrawal right exceeds the greater of $5,000 or five percent of the trust assets. As a result, most estate planners avoid this result by allowing the right of withdrawal to lapse each year only to the extent it would not result in a gift (i.e., the greater of five percent or $5,000). For GST tax purposes, the exempt status of the trust is determined by reference to the transferor. The identity of the transferor is determined without regard to the withdrawal powers until and unless there is a lapse subject to estate or gift tax. Thus, if the lapse generates gift tax, it is treated as having been distributed to the beneficiary holding the withdrawal right who in turn transfers it back to the trust.88

87 Treas. Regs. §26.2612-1(f), Example 3. 88 Treas. Regs. §26.2612-1(c)(1).

Page 103: Advanced Selected Issues for Trusts, Estates, and Their ... · 28/07/1997  · 3. Availability of §179 expensing 2 C. Administrative stance 2 1. Estates 2 2. Trusts 3 D. Judicial

cpenow.com / [email protected] Copyright © 2016 Surgent McCoy CPE, LLC -- ITEB/16/01 5-25

Example: T transfers $14,000 to a trust providing income to Child C for life, and the principal to be paid to Grandchild GC. The trustee has the discretion to distribute the principal for GC’s benefit during C’s lifetime. C has a right to withdraw $14,000 from the trust for the 60-day period following the transfer into the trust. The power lapses thereafter. C does not exercise the power. T is the transferor of $14,000. On the lapse, C is treated as making a gift of $9,000 ($14,000 -$5,000). C is treated as the transferor of $9,000. As to the portion of the trust for which T remains the transferor, the death of C is a taxable termination; as to the portion for which C is the transferor, the death of C does not trigger a generation-skipping transfer. Thus, T may have to allocate $14,000 of exemption even though only the portion allocable to $5,000 will generate a GST tax.

c. The donor may want to make direct gifts to children and grandchildren who in turn create

a life insurance trust. The donor would simply allocate GST exemption to the direct skips. However, there are at least three disadvantages: • Donees may be subject to estate tax on the trust by reason of an included power

in their estate; • Donees may similarly be delayed in allocating their own GST exemption by the

estate-tax inclusion period, which would reduce or eliminate any leverage; and • Making outright gifts to minors with the expectation that they will make transfers

to the trust is fraught with danger. d. Since the insurance policy may lapse (as in term insurance) before being collected or the

trust may be distributed to non-skip persons, some estate planners recommend allocating no GST exemption to the trust, relying solely on the leverage of premiums-to-proceeds.

e. The client may take advantage of the valuation of the generation-skipping transfer for a late-filed gift tax return to reduce the amount of GST exemption required to be allocated in the early years of a life insurance policy in a life insurance trust.

Example: G establishes a life insurance trust on April 1, 2016, with a $1 million life

insurance policy with respect to which there is an annual $25,000 premium. If T allocates $25,000 of GST exemption on or before April 15, 2017, on a timely filed gift tax return, there is a zero inclusion ratio. Suppose that at the end of the first year, the policy has an interpolated terminal reserve plus unearned premium of $100 on March 31, 2017. If T files a gift tax return on April 15, 2017 not allocating exemption and then follows this by filing on April 16, 2017 -- before contributing the $25,000 for the second year’s premium (a late gift tax return for the purpose of allocating exemption) -- the amount allocated should be only $100. Note, however, that if T dies between the due date of filing the return and the date the late return was to be filed, the entire amount of the proceeds would be subject to the GST tax.

The IRS would probably raise a host of issues concerning this technique, making this technique extremely aggressive.

5. Multiple skips

Trusts that provide for multiple skips increase the leverage of the exemption allocation, because a zero inclusion ratio will protect each skip from the tax.

Example: T transfers $5,450,000 to a trust for the benefit of Child C, Grandchild GC, and Great-grandchild GGC; and allocates enough exemption to have a zero inclusion ratio. When C dies, there is a taxable termination, but no tax due. When a distribution is made to GC or GGC there is a taxable distribution, but no GST tax because of the zero inclusion ratio. Finally, when GC dies, there is a taxable termination, but again no GST tax is due because of the GST zero inclusion ratio.

Page 104: Advanced Selected Issues for Trusts, Estates, and Their ... · 28/07/1997  · 3. Availability of §179 expensing 2 C. Administrative stance 2 1. Estates 2 2. Trusts 3 D. Judicial

cpenow.com / [email protected] Copyright © 2016 Surgent McCoy CPE, LLC -- ITEB/16/01 5-26

6. Layering

In order to preserve leverage, distributions to non-skip persons should not be made from a zero inclusion trust. Since these distributions could have remained exempt from GST tax if distributed to a skip person, the distribution to a non-skip person, to the extent not expended prior to the death of the non-skip person, will generally be subject to the estate tax.

7. Retroactive allocation of the GST tax exemption

The GST tax exemption may be allocated retroactively when there is an unnatural order of death. If a lineal descendant of the transferor predeceased the transferor, then the transferor may allocate any unused GST tax exemption to any previous transfer or transfers to the trust on a chronological basis. The provision allows a transferor to retroactively allocate GST tax exemption to a trust where a beneficiary: (i) is a non-skip person; (ii) is a lineal descendant of the transferor’s grandparent or grandparent of the transferor’s spouse; (iii) is a generation younger than the generation of the transferor; and (iv) dies before the transferor. Exemption is allocated under this rule retroactively, and the applicable fraction and inclusion ratios under this provision are determined based on the value of the property on the date that the property was transferred to the trust.

Note:

To take advantage of this provision, the allocation by the transferor must be made on a gift tax return filed on or before the date prescribed by law for gifts made within the calendar year within which the non-skip person’s death occurred (April 15 of the calendar year following the year of such death).

8. Relief from late elections

In general, an election to allocate a generation-skipping-transfer tax exemption to a specific transfer may be made at any time up to the time for filing the transferor’s estate tax return. If an allocation is made on a gift tax return filed timely with respect to the transfer to trust, then the value on the date of transfer to the trust is used for determining generation-skipping-transfer-tax-exemption allocation. However, if the allocation relating to a specific transfer is not made on a timely filed gift tax return, then the value on the date of allocation must be used. Relief for an inadvertent failure to make an election on a timely filed gift tax return to allocate the generation-skipping-transfer tax exemption is now allowed. Treasury is authorized and directed to grant extensions of time to make the election to allocate GST tax exemption and to grant exceptions to the time requirement. If such relief were granted, then the value on the date of transfer to a trust would be used for determining GST tax exemption allocation. In determining whether to grant relief for late elections, the Treasury secretary is directed to consider all relevant circumstances, including evidence of intent contained in the trust instrument or instrument of transfers and such other factors as the Treasury secretary deems relevant. For purposes of determining whether to grant relief, the time for making the allocation (or election) is treated as if not expressly prescribed by statute.

9. Substantial compliance

Substantial compliance with the statutory and regulatory requirements for allocating GST tax exemption will suffice to establish that GST tax exemption was allocated to a particular transfer or a particular trust. If a taxpayer demonstrates substantial compliance, then so much of the transferor’s unused GST tax exemption will be allocated to the extent it produces the lowest possible inclusion ratio. In determining whether there has been substantial compliance, all relevant circumstances will be considered, including evidence of intent contained in the trust instrument or instrument of transfer, and such other factors as Treasury deems appropriate.

Page 105: Advanced Selected Issues for Trusts, Estates, and Their ... · 28/07/1997  · 3. Availability of §179 expensing 2 C. Administrative stance 2 1. Estates 2 2. Trusts 3 D. Judicial

cpenow.com / [email protected] Copyright © 2016 Surgent McCoy CPE, LLC -- ITEB/16/01 5-27

B. Segregation of assets

1. In general

Since the GST tax is applied against the entire value of the trust assets and the tax rate is effectively adjusted by the inclusion ratio, there is an advantage in creating trusts with inclusion ratios of zero and with inclusion ratios of one.

Example: Widower T has an estate of $10,900,000 after estate taxes. He transfers his estate to a trust for the benefit of Child C with the remainder to be divided between Grandchild GC and a charity. If T’s executor allocates the $5,450,000 exemption to the trust, the trust has an inclusion ratio of 1/2. Because of C’s interest, the transfer to the trust is not a direct skip. When C dies, there is a taxable termination. Assuming the trust is still worth $10,900,000, there is a termination in the amount of $5,450,000, taxed at 20 percent (40 percent x 1/2) or $1,090,000.

If T had created two separate trusts, each of $5,450,000, one with the remainder to the grandchild, the other with remainder to the charity, T’s executor could allocate the entire $5,450,000 exemption to the first trust. This means the inclusion ratio of the first trust is zero and the inclusion ratio of the second trust is one. Now when C dies, the taxable termination on the first trust is taxed at zero percent (40 percent x 0). By segregating assets, the executor has saved $1,090,000 in taxes. If the trust assets had appreciated significantly during the lifetime of the child, the tax savings would be even greater.

2. Separate trusts

In order to achieve tax savings from segregating assets into two or more trusts, a trust must be treated as separate trusts.

a. A trust will be treated as two trusts for GST tax purposes if the terms of the governing instrument provide for the trust to be divided and treated as two separate trusts.89

b. A trust will be treated as two trusts for GST tax purposes if the executor or trustee is permitted to sever the trust by the terms of the governing instrument. The executor or trustee must be certain that: • The trusts provide for the same succession of interests and beneficiaries as the

original trust; • The severance occurs prior to the date for filing (including any actual extensions

granted) the federal estate tax return of the transferor; and • Either the trusts are severed on a fractional basis, which need not be pro rata as

long as it reflects actual values on the date of funding, or in a manner that fairly reflects net appreciation or depreciation in assets. Alternatively, the trust must be severed on the basis of a pecuniary amount.90

Example 1: T transfers $100,000 to a trust under which income is to be paid in equal shares

for 10 years to T’s child, C, and T’s grandchild, GC (or their respective estates). The trust does not permit distributions of principal during the term of the trust. At the end of the 10-year term, the trust principal is to be distributed to C and GC in equal shares. The shares of C and GC in the trust are separate and independent, and therefore, are treated as separate trusts. The result would not be the same if the trust permitted distributions of principal unless the distributions could only be made from a one-half separate share of the initial trust principal and the

89 Treas. Regs. §26.2654-1(b)(1)(i). 90 Treas. Regs. §26.2654-1(b)(1)(ii).

Page 106: Advanced Selected Issues for Trusts, Estates, and Their ... · 28/07/1997  · 3. Availability of §179 expensing 2 C. Administrative stance 2 1. Estates 2 2. Trusts 3 D. Judicial

cpenow.com / [email protected] Copyright © 2016 Surgent McCoy CPE, LLC -- ITEB/16/01 5-28

distributee’s future rights with respect to the trust are correspondingly reduced. T may allocate part of T’s GST exemption under §2632(a) to the share held for the benefit of GC.

Example 2: The facts are the same as in Example 1, except the trustee holds the

discretionary power to distribute the income in any proportion between C and GC during the last year of the trust. The shares of C and GC in the trust are not separate and independent shares throughout the entire term of the trust and, therefore, are not treated as separate trusts for purposes of the GST tax.

Example 3: T creates a lifetime revocable trust providing that on T’s death $500,000 is

payable to T’s spouse, S, with the balance of the principal to be held for the benefit of T’s grandchildren. The value of the trust is includable in T’s gross estate upon T’s death. Under the terms of the trust, the payment to S is required to be made in cash, and under local law S is entitled to receive interest on the payment at an annual rate of six percent, commencing immediately upon T’s death. For purposes of the GST tax, the trust is deemed to be created at T’s death, and the $500,000 payable to S from the trust is treated as a separate share. The result would be the same if the payment to S could be satisfied using noncash assets at their value on the date of distribution. The result would also be the same if the decedent’s probate estate poured over to the revocable trust on the decedent’s death and was then distributed in accordance with the terms of the trust.

Example 4: The facts are the same as in Example 3, except the bequest to S is to be paid in

noncash assets valued at their values as finally determined for federal estate-tax purposes. Neither the trust instrument nor local law requires that the assets distributed in satisfaction of the bequest fairly reflect net appreciation or depreciation in all the assets from which the bequest may be funded. S’s $500,000 bequest is not treated as a separate share and the trust is treated as a single trust for purposes of the GST tax.

3. Severing of trusts holding property having an inclusion ratio of greater than zero

If the value of transferred property exceeds the amount of the generation-skipping-transfer tax exemption allocated to that property, then the generation-skipping-transfer tax generally is determined by multiplying a flat tax rate equal to the highest estate-tax rate by the “inclusion ratio” and the value of the taxable property at the time of the taxable event. The “inclusion ratio” is the number one, minus the “applicable fraction.” The applicable fraction is a fraction calculated by dividing the amount of the generation-skipping-transfer tax exemption allocated to the property by the value of the property.

a. A trust may be severed into two or more trusts (e.g., one with an inclusion ratio of zero and one with an inclusion ratio of one) only if: (i) the trust is severed according to a direction in the governing instrument; or (ii) the trust is severed pursuant to the trustee’s discretionary powers, but only if certain other conditions are satisfied (e.g., the severance occurs or a reformation proceeding begins before the estate tax return is due).

b. In addition, a trust may be severed in a “qualified severance.” A qualified severance is defined as the division of a single trust and the creation of two or more trusts if: (i) the single trust was divided on a fractional basis; and (ii) the terms of the new trusts, in the aggregate, provide for the same succession of interests of beneficiaries as are provided in the original trust. If a trust has an inclusion ratio of greater than zero and less than one, a severance is a qualified severance only if the single trust is divided into two trusts, one of which receives a fractional share of the total value of all trust assets equal to the applicable fraction of the single trust immediately before the severance. In such case, the

Page 107: Advanced Selected Issues for Trusts, Estates, and Their ... · 28/07/1997  · 3. Availability of §179 expensing 2 C. Administrative stance 2 1. Estates 2 2. Trusts 3 D. Judicial

cpenow.com / [email protected] Copyright © 2016 Surgent McCoy CPE, LLC -- ITEB/16/01 5-29

trust receiving such fractional share shall have an inclusion ratio of one. Under the provision, a trustee may elect to sever a trust in a qualified severance at any time.

4. Inter vivos trusts

If a grantor creates an irrevocable inter vivos trust in an amount equal to or less than the unused exemption, the trust can have a zero inclusion ratio. If the trust’s inclusion ratio is zero, the entire trust, including any appreciation of the trust assets, will not be subject to the GST tax. Through gift-splitting, this transfer technique could utilize both spouse’s applicable credit amounts and GST exemptions. This technique would also protect against the spouse predeceasing the grantor and not having enough assets to utilize both exemptions at the spouse’s death. This technique would not work if the grantor retained an income interest from the trust.

a. The estate-tax inclusion period limits the timing of the allocation in cases in which the grantor retains an includable interest under §§2036-2038, such as a grantor-retained-income interest.

b. The estate-tax inclusion period includes property that would be included in the gross estate of the spouse. Thus, if the trust will be included in the estate of the spouse, such as an inter vivos marital-deduction trust or a marital-deduction trust under will, the allocation of the GST exemption will be delayed until the death of the spouse. There are three exceptions to this rule: • If the possibility the property will be included in the spouse’s gross estate is so

remote as to be negligible. If the possibility can be ascertained by actuarial methods to be less than five percent, it will be deemed remote;91

• If the reason for including the trust in the spouse’s gross estate is solely due to a right to withdraw the greater of $5,000 or five percent, and that right terminates within 60 days of the transfer to the trust;92 and

• A QTIP trust for which the reverse-QTIP election has been made is not subject to this rule.93

Planning point:

Transfers to a trust should generally be made so as to avoid an estate-tax inclusion period by not retaining any includable power or interest or having the spouse have any includable power or interest. If this is not practicable, no exemption should be allocated when the trust is created. Instead, any exemption should be allocated at the end of the estate-tax inclusion period, which will coincide with the surviving spouse’s death.

5. Transfers at death

At death, the will or trust can define the amount of the funding of a trust by a formula that allocates all of the decedent’s unused exemption so that the trust will have an inclusion ratio of zero; other assets pass outright or to other trusts having an inclusion ratio of one. This is often accomplished through the use of pecuniary-fractional-share clauses, which often also coordinate the use of the unlimited marital deduction and the applicable credit amount.

91 Treas. Regs. §26.2632-1(c)(2)(ii)(A). 92 Treas. Regs. §26.2632-1(c)(2)(ii)(B). 93 Treas. Regs. §26.2632-1(c)(2)(ii)(C).

Page 108: Advanced Selected Issues for Trusts, Estates, and Their ... · 28/07/1997  · 3. Availability of §179 expensing 2 C. Administrative stance 2 1. Estates 2 2. Trusts 3 D. Judicial

cpenow.com / [email protected] Copyright © 2016 Surgent McCoy CPE, LLC -- ITEB/16/01 5-30

C. Making choices

1. Other transfer taxes

a. The estate tax has the following additional advantages: • Section 2013 credit for property previously taxed; • Other credits against the estate tax; • Basis step-up; • Sections 303 and 6166; and • A later due date than gift tax if death occurs later in the year.

b. The gift tax has the following advantages: • Annual exclusions; • The ability to obtain the spouse’s annual exclusion; and • A lower effective tax rate than taxable terminations or taxable distributions

because it is tax-exclusive. c. The generation-skipping tax has the following advantages:

• Multiple skips; • More potential deferral; • Some states have no generation-skipping tax; • Applies only to certain transfers; • Direct skips are tax-exclusive while estate tax is tax-inclusive; • The due date may be later than the estate tax depending on when the transferor

dies; and • Valuation rules under Chapter 14 do not apply.

2. Using the other tax systems

a. Give outright gifts to non-skip persons. b. Authorized distributions of principal to non-skip persons who in turn may make gifts to

persons who are skip persons as to the original transferor, but who are non-skip persons as to the new transferor.

c. General power of appointment held by a non-skip person.94

3. Using direct skips

The tax on a direct skip is lower than that of a similar taxable termination or taxable distribution because the taxable amount on a direct skip is limited to the amount of the transfer without regard to the tax itself. The latter two include the tax itself and are tax-inclusive. On the other hand, taxable terminations and taxable distributions are advantageous because they are almost always transfers in trust that defer the GST tax and give the trustee discretion as to when to make distributions, and whether to add beneficiaries.

D. Handling “grandfathered” trusts

1. In general

Inter vivos irrevocable trusts created prior to September 25, 1985, or those created under a will executed before October 22, 1986, for a decedent dying before January 1, 1987, are grandfathered, except for contributions made after those dates. 94 PLR 9527024.

Page 109: Advanced Selected Issues for Trusts, Estates, and Their ... · 28/07/1997  · 3. Availability of §179 expensing 2 C. Administrative stance 2 1. Estates 2 2. Trusts 3 D. Judicial

cpenow.com / [email protected] Copyright © 2016 Surgent McCoy CPE, LLC -- ITEB/16/01 5-31

2. Additions

Additions to the trust will cause partitioning of such trust into pre-September 25, 1985 and post-September 25, 1985 portions. The post-September 25, 1985 portion is subject to the GST tax. The pre-September 25, 1985 portion has an inclusion ratio of zero. Additions are defined as follows:

• Actual additions of any property added to trust, with the exceptions of: (i) capital appreciation; (ii) accumulated income; and (iii) nontaxable gifts that qualify as direct-skip annual exclusions.

• Constructive additions can also occur when: (i) amounts are left in the trust when a taxable power of appointment lapses; (ii) the term of the trust is extended beyond the common-law rule; (iii) perpetuities through the exercise of a limited power; and (iv) a liability owed by the trust is paid by another.

3. Effect of an addition

A grandfathered trust with post-September 25, 1985 additions that has a taxable termination or distribution must be calculated by determining the amount taxable under the GST tax rules. The “allocation fraction” determines the amount taxable as follows: A + B x D = taxable amount T A Amount of latest addition. B Value of the nonexempt portion of the trust just prior to the current addition (this portion

became nonexempt because of other post-September 25, 1985 additions, if any). T Total value of the trust (fair market value in excess of any expenses that would be

allowable under §2053). D The total amount of any generation-skipping transfer from the trust.

Example: On August 16, 1980, A established an irrevocable trust. Under the trust instrument, the trustee is required to distribute the entire income annually to A’s child, C, for life, then to A’s grandchild, GC, for life. Upon GC’s death, the reminder is to be paid to GC’s issue. On October 1, 2016, when the total value of the entire trust was $400,000, A transferred $100,000 to the trust. The allocation fraction is computed as follows:

Value of addition = $100,000= 0.2 Total value of trust $500,000

Thus, immediately after the transfer, 20 percent of the value of future generation-skipping transfers under the trust will be subject to the GST tax.

4. Planning

a. Annual distributions of income should be avoided or be made to the youngest generation beneficiaries.

b. Avoid invading principal for beneficiaries in older generations. c. Avoid court reformations that change the quality, value, or timing of any powers,

beneficial interests, rights, or expectancies originally provided for under the terms of the original trust.

Page 110: Advanced Selected Issues for Trusts, Estates, and Their ... · 28/07/1997  · 3. Availability of §179 expensing 2 C. Administrative stance 2 1. Estates 2 2. Trusts 3 D. Judicial
Page 111: Advanced Selected Issues for Trusts, Estates, and Their ... · 28/07/1997  · 3. Availability of §179 expensing 2 C. Administrative stance 2 1. Estates 2 2. Trusts 3 D. Judicial

Let Surgent Help Your Associates Become CPAs Faster

Our New Approach to CPA Review Will Improve Your Bottom Line

How Surgent CPA Review Is Different:

• Uses exclusive “adaptive learning” softwarethat customizes each student’s study plan according to identified knowledge gaps

• Focuses students specifically on what theydon’t know, which saves 50+ hours of studytime per part, while driving an 89% pass rate (almost 2x the national average!)

• Comes with over 350 video lectures (most are 10-20 minute sessions) presented by suchtop instructors as Jack Surgent and Liz Kolar

• Includes over 6,700 multiple-choice questions& 260 task-based simulations with thoroughexplanations to ensure exam readiness

• Includes FREE flashcard app with 2,200+terms and definitions (for iOS and Android)

Give your firm and your associates a competitive advantage with Surgent CPA Review: the fastestway to pass the first time.

1. Online: surgent.com/CPAreview2. Over the phone: 1.800.7.SURGENT3. Through your state society contact

Tuition payment plans available – call for info!

“I’ve received countless failing scores over the past three years…with Surgent CPA Review I was able to pass all four sections in a row. I’ve studied with three other review courses and can say with confidence that this is the best.”

- Lauren F @ Ernst & Young

• SURGENT.COM/CPAREVIEW • 1.800.7.SURGENT

3 Easy Ways to Enroll:

Page 112: Advanced Selected Issues for Trusts, Estates, and Their ... · 28/07/1997  · 3. Availability of §179 expensing 2 C. Administrative stance 2 1. Estates 2 2. Trusts 3 D. Judicial
Page 113: Advanced Selected Issues for Trusts, Estates, and Their ... · 28/07/1997  · 3. Availability of §179 expensing 2 C. Administrative stance 2 1. Estates 2 2. Trusts 3 D. Judicial

Can Surgent Firm Solutions Help You?

Surgent provides a complete range of convenient,

high-quality CPE programs to support firms of all sizes.

Training options include:

• On-Site Training – use Surgent’s

top-rated materials and instructors

• Content Only – have your own

instructor use Surgent’s materials

• Private Webinars – schedule a

private showing of one of our

webinars on a date of your choice

• Group Discounts – get a group

rate for 5+ attendees joining the

same Surgent public webinar

• Flexible Firm Access – provide

your firm with easy access to

Surgent webinars and self-study

courses at a preferred rate

(starts at 100 CPE hours)

• Firm Licensing – customized

content solutions for firms with

their own training platform

Contact the

Surgent Firm Solutions team

to learn more:

Kris Moretti

610-994-9626

[email protected]

Joe Rastatter

610-994-9618

[email protected]

cpenow.com • 484-588-4197

Page 114: Advanced Selected Issues for Trusts, Estates, and Their ... · 28/07/1997  · 3. Availability of §179 expensing 2 C. Administrative stance 2 1. Estates 2 2. Trusts 3 D. Judicial
Page 115: Advanced Selected Issues for Trusts, Estates, and Their ... · 28/07/1997  · 3. Availability of §179 expensing 2 C. Administrative stance 2 1. Estates 2 2. Trusts 3 D. Judicial

Three Practical Toolkits That Solve Critical Needs for Firms

Downloadable, Customizable, and Affordable!

The Essential Nonattest Engagement Letter Toolkit – $59

Most firms’ engagement letters are not designed to cover nonattest services suchas bookkeeping and payroll processing, potentially leaving them vulnerable to malpractice claims. This toolkit addressesthat gap, providing practical, real-life examples and sample language that CPAscan tailor to specific client circumstances.

The Essential SSARS 21 Implementation Toolkit – $99

This very popular toolkit includes 65 pagesof customizable documents that will getyour firm ready quickly, even if you missedthe December 15th, 2015 effective date ofSSARS 21. The toolkit includes the brand-new nonattest service of Preparation of Financial Statements, as well existing attestservices of Compilation Engagements andReviews of Financial Statements.

If you’re not in compliance yet, you can’tafford to miss this toolkit. Download these toolkits now at

cpenow.com/practice-toolsor call customer service at (800) 778-7436 to order.

The Essential Third Party Verification Toolkit: Responding

to Comfort Requests Without Putting Your Practice at Risk – $99Do you ever receive requests from a client’sbank or other third party asking you to verify the client’s self-employment incomeor authenticate the client’s business ownership percentage? Responding or even attempting to provide a “non-answer”to these comfort requests could result in liability if not properly worded.

The practice aids contained within thisdownloadable resource help CPAs determine which statements of fact to third parties are allowable and when an alternative approach might need to be suggested, so that you can make more informed decisions about the nature andscope of certain services you provide to clients.

Page 116: Advanced Selected Issues for Trusts, Estates, and Their ... · 28/07/1997  · 3. Availability of §179 expensing 2 C. Administrative stance 2 1. Estates 2 2. Trusts 3 D. Judicial
Page 117: Advanced Selected Issues for Trusts, Estates, and Their ... · 28/07/1997  · 3. Availability of §179 expensing 2 C. Administrative stance 2 1. Estates 2 2. Trusts 3 D. Judicial

cpenow.com / [email protected] Copyright © 2016 Surgent McCoy CPE, LLC -- ITEB/16/01 6-i

Charitable Lead Trusts

Learning objectives 1 I. Charitable lead trusts 1

A. Background 1 1. In general 1 2. Types 1 3. Focus 1

B. Rationale for a CLT 2 1. Basic concepts 2 2. How to do it 2

C. Technical requirements -- CLT 2 1. In general 2 2. Writing 2 3. Types of CLTs 3 4. Unitrust versus guaranteed annuity 3 5. Grantor trusts and nongrantor trusts 4 6. Allowable beneficiaries 5 7. IRS provides sample forms for CLATs 5

D. Valuation and the computation of charitable deductions 5 1. In general 5 2. Valuation of guaranteed annuity interest 6 3. Valuation of unitrust interest 6 4. Reporting requirements 7 5. Generation-skipping transfer (GST) tax 8 6. Nonqualifying nongrantor trust 8 7. Grantor trust 8 8. Property contributed to a charitable trust 8

Page 118: Advanced Selected Issues for Trusts, Estates, and Their ... · 28/07/1997  · 3. Availability of §179 expensing 2 C. Administrative stance 2 1. Estates 2 2. Trusts 3 D. Judicial
Page 119: Advanced Selected Issues for Trusts, Estates, and Their ... · 28/07/1997  · 3. Availability of §179 expensing 2 C. Administrative stance 2 1. Estates 2 2. Trusts 3 D. Judicial

cpenow.com / [email protected] Copyright © 2016 Surgent McCoy CPE, LLC -- ITEB/16/01 6-1

Charitable Lead Trusts

Learning objectives

After completing this chapter, you will be able to: • Discuss the rationale for establishing a Charitable Lead Trust (CLT); • List the technical requirements involved in setting up CLTs; • Determine the valuation of a guaranteed annuity interest and the valuation of a unitrust

interest; and • Explain the reporting and filing requirements of CLTs.

I. Charitable lead trusts

A. Background

1. In general

A charitable lead trust (CLT) is a split-interest trust that provides for the payment of a guaranteed annuity or unitrust amount to a qualified charitable organization for a fixed or ascertainable period (the charitable term) followed by a reversion or remainder in a noncharitable person.1

2. Types

The two types of CLTs recognized are the charitable lead annuity trust and the charitable lead unitrust. a. The charitable lead annuity trust expresses the annuity as a fixed-dollar amount; for

example, $10,000 per year or a fixed percentage of the net fair market value of the residue of the estate, such as an amount equal to eight percent of the net fair market value of the assets initially transferred to the trust, as finally determined for federal estate-tax purposes.2

b. By contrast, the charitable lead unitrust expresses the sum paid yearly as a fixed percentage of the value of the total trust property (principal and income), determined annually. An example would be an amount equal to eight percent of the net fair market value of all trust assets, valued as of the first day of each taxable year of the trust.

c. There is no statutory minimum or maximum annuity regardless of whether the trust is classified as a charitable lead annuity or unitrust. Nevertheless, the present value of the annuity must be less than the value of the property transferred in trust to pay the annuity.

3. Focus

Because of the high excise-tax costs of transferring property to one’s heirs, one practical technique to ease the tax burden is a charitable lead trust. The purpose of this chapter is to depict and analyze the CLT as a vehicle for passing wealth to subsequent generations while also satisfying the donor’s philanthropic interests.

1 See generally I.R.C. §§170, 2055, and 2522. 2 Treas. Regs. §§20.2055-2(e)(2)(v) and 25.2522(c)-3(c)(2)(v).

Page 120: Advanced Selected Issues for Trusts, Estates, and Their ... · 28/07/1997  · 3. Availability of §179 expensing 2 C. Administrative stance 2 1. Estates 2 2. Trusts 3 D. Judicial

cpenow.com / [email protected] Copyright © 2016 Surgent McCoy CPE, LLC -- ITEB/16/01 6-2

B. Rationale for a CLT

1. Basic concepts

Charitable lead trusts involve two simple concepts: (i) the time-value of money; and (ii) charitable deductions.

a. First, money due in the future, discounted for the value of its use while waiting for it, is worth less today.

b. Second, gifts or contributions to qualified charitable organizations are not subject to gift tax.

2. How to do it

The approach is quite simple: The client transfers property into a trust that provides for a dollar amount or percentage of the value of the property to be paid to specified charities each year for a stated period, after which the balance is to be paid to the client’s desired beneficiaries. In effect, the transfer of property into a trust allows the time-value discount to reduce the taxable present value of the deferred gift to the noncharitable remaindermen. The following illustration of a nongrantor CLT depicts the interplay among time, money, and taxes.

Example 1: X would like to transfer $1,000,000 of Home Depot stock to his children who have no immediate need for it. X transfers the stock into the trust, whereby the dividends of $50,000 a year are paid to one or more qualified charities each year for 15 years. X is allowed to specify the charities, the amount donated, and the time period. After 15 years, the balance of the trust is paid to the children. Assuming a §7520 rate of 4 percent, a $50,000 annuity for 15 years is worth approximately $555,920 today. This leaves $444,080 out of the $1,000,000 as the present value of the gift to his children on which the gift tax is paid. Assuming a 40-percent transfer-tax bracket, only $177,632 will be paid on the $1,000,000 transfer and X has reduced his taxable estate by $1,177,632, which will also reduce X’s estate-tax liability.

C. Technical requirements -- CLT

1. In general

Many of the technical rules for a CLT are related to the rules and concepts applicable to charitable remainder trusts. While most of the requirements for CLTs are set forth in the Internal Revenue Code and regulations, many of the requirements have been established as a result of information provided in private rulings. A review of the judicial record concerning CLTs reveals that there has not been much activity in this area within the last 10 years. Due to the complexity of the rules and requirements for CLTs, it may be advantageous for the grantor to request an advance ruling from the IRS before executing documents to create a CLT.

2. Writing

Although nothing in the Code provides that a CLT must be formalized in a written document, common practice and a litigious society dictates that a well-written document is necessary to insure compliance with the rules and to document the grantor’s wishes. For a trust to qualify as a CLT, it must be a valid trust under local law. As previously noted, a CLT is a split-interest trust, thereby subject to requirements of §508(e), which state that the governing document of the trust must expressly prohibit the violation of the private-foundation rules.

Page 121: Advanced Selected Issues for Trusts, Estates, and Their ... · 28/07/1997  · 3. Availability of §179 expensing 2 C. Administrative stance 2 1. Estates 2 2. Trusts 3 D. Judicial

cpenow.com / [email protected] Copyright © 2016 Surgent McCoy CPE, LLC -- ITEB/16/01 6-3

3. Types of CLTs

There are three general categories of CLTs. They are: (i) qualifying nongrantor CLT, both inter vivos

and testamentary; (ii) grantor CLT; and (iii) nonqualifying nongrantor CLT. A grantor CLT and nonqualifying nongrantor CLT can only be created as inter vivos trusts. Each type of CLT affords the grantor a different type of charitable tax deduction upon funding the trust and may be taxed differently during the term of the trust.

a. A qualifying CLT receives either a gift- or estate-tax deduction depending on whether it is created during life or at death.

b. A grantor CLT is subject to the grantor-tax rules in §671-678 and receives an income-tax deduction at the time of funding.

c. A nonqualifying CLT does not meet the payout requirements for a CLT and is thus disqualified from taking a charitable tax deduction upon the funding of the trust. However, a nonqualifying CLT is allowed to take a gift-tax deduction each year when charitable distributions are made.

4. Unitrust versus guaranteed annuity

When a client has decided to create a charitable lead trust, there are several factors to consider when choosing whether to use a unitrust or a guaranteed annuity.

a. If it is believed that the transferred property has high-growth potential or that an inflationary climate will prevail during the term of the trust, the guaranteed annuity may be preferable to the unitrust format. As the value of the corpus appreciates, income derived from the corpus may begin to exceed the required amount that must be paid with the annuity. The excess is accumulated for the noncharitable beneficiaries.

Example 2: If in Example 1, the Home Depot stock grows eight percent per year, X’s children

will receive more than $3 million at the termination of the trust. Remember, the total tax cost on this gift was just $177,632. In comparison, if X had not set up the trust, he would require around $5 million in assets in his estate, before taxes, to provide an equivalent $3 million legacy.

b. However, if the property in the trust does not produce enough income to pay the annuity,

the trustee must invade corpus to satisfy it. The use of the unitrust alternative may be preferable if the fair market value of the assets transferred is expected to decline. Specifically, the amount of the annual unitrust payment fluctuates according to the net fair market value of the trust assets. As a result, when the net assets decline in value, the amount required to be paid to the charitable interest also decreases. However, if the unitrust property does not produce enough income to pay the charity, the balance must also come from corpus.

c. Another factor to consider when choosing either an annuity or unitrust is the administrative costs. The net fair market value of the assets of the property used in a unitrust must be valued annually.3 Depending on the type of assets contributed to the trust, valuation could be time consuming and expensive, especially when contributed assets are stock in closely held corporations. An annuity trust requires a valuation only when the trust is created. Consequently, an annuity trust could be a more cost-effective tool.

3 I.R.C. §170(f)(2)(B).

Page 122: Advanced Selected Issues for Trusts, Estates, and Their ... · 28/07/1997  · 3. Availability of §179 expensing 2 C. Administrative stance 2 1. Estates 2 2. Trusts 3 D. Judicial

cpenow.com / [email protected] Copyright © 2016 Surgent McCoy CPE, LLC -- ITEB/16/01 6-4

5. Grantor trusts and nongrantor trusts

After the client decides whether to use a guaranteed annuity or unitrust, the next decision that must be made is which type of trust to employ. As mentioned previously, each format generates different tax consequences. An analysis of the technical rules and tax results follow.

a. The grantor charitable lead trust is governed by the grantor-trust rules.4 Under this format, the grantor obtains an immediate income-tax deduction equal to the present value of the “lead” interest passing to charity. Because the grantor is deemed the owner of the income interest, the grantor remains taxable on the income earned by the trust. In effect, the deduction is “recaptured” over the term of the trust as the grantor pays the tax on the income. The use of this format may be advisable when the grantor is in a high tax bracket in the year the trust is created and expects to be in a lower tax bracket in subsequent years. In this example, the charitable deduction will offset income subject to high rates (39.6 percent), while income reported in later years could be taxed at rates as low as 10 percent. The charitable deduction is limited to either 30 percent or 20 percent, depending on the type of property contributed to the trust.5 The excess may be carried forward five years.

b. Unlike grantor trusts, the income of nongrantor trusts is taxed to the trust itself. These trusts are taxed as complex trusts under the rules of Subchapter J. Therefore, the grantor is not entitled to an income-tax charitable deduction, but the assets of the trust corpus will not be included in the grantor’s estate. Nongrantor trusts can be considered “qualifying” or “nonqualifying” depending on the structure of the payment to the charitable income interest. To qualify for a charitable estate- or gift-tax deduction, the trust must provide a payment in the form of a guaranteed annuity or unitrust payment. A nonqualifying nongrantor trust is structured so that the trust only pays the income earned in that year. Although they are both nongrantor trusts, qualifying and nonqualifying have many differences. (i) A qualified nongrantor trust must pay a guaranteed annuity or unitrust interest to

qualify for the estate- and gift-tax charitable deductions. A qualified nongrantor trust may be established either as an inter vivos or testamentary trust. • Each of these trusts offer different tax benefits. An inter vivos trust does

not provide income-tax charitable deductions but does provide gift-tax charitable deductions. A testamentary trust provides estate-tax charitable deductions for the value of the lead interest.

• Section 2055 allows unlimited estate-tax charitable deductions. As a result, with proper planning, a grantor could eliminate estate-tax liability with a testamentary charitable lead trust. By setting the term of the charitable income interest for a sufficient length of time, the present value of the income stream could equal the amount of the trust’s corpus. This was a tactic used by Jacqueline Kennedy Onassis. Naturally, the grantor would want to make sure that the noncharitable remaindermen were financially secure to forego receipt of the remainder.

• Both the inter vivos and testamentary trusts are taxed as complex trusts and are eligible for the charitable deduction.6 If the amount of income earned by the trust equals the amount that must be paid for the annuity

4 I.R.C. §§671-678. 5 I.R.C. §170(b)(1)(B). 6 I.R.C. §642(c).

Page 123: Advanced Selected Issues for Trusts, Estates, and Their ... · 28/07/1997  · 3. Availability of §179 expensing 2 C. Administrative stance 2 1. Estates 2 2. Trusts 3 D. Judicial

cpenow.com / [email protected] Copyright © 2016 Surgent McCoy CPE, LLC -- ITEB/16/01 6-5

or unitrust interest, then the trust will be allowed to deduct the full amount paid. However, if the trust does not earn enough to satisfy the annuity or unitrust interest, the trustee may have to invade corpus to cover the difference. Conversely, if the amount of income that the trust earns exceeds the amount that must be paid to charity, then the trust will be taxed on the excess. Unfortunately, a qualified nongrantor trust is not eligible to deduct any additional charitable contributions in excess of the annuity or unitrust payments. The court ruled that advanced payments to charity do not qualify for the §642(c) deduction.7

(ii) A nonqualifying nongrantor trust is also taxed as a complex trust. Since this trust does not provide for a guaranteed annuity or unitrust payment, it will not provide an initial charitable gift-tax deduction to the grantor. Nevertheless, this vehicle does have several benefits. A major benefit is that the trust is not “locked” into providing a set amount of income to the charity. Therefore, a trustee does not have to worry about income shortfalls or excess income, unlike the qualified nongrantor trust. Also, the grantor will receive a gift-tax charitable deduction at the time of the charitable contribution because it is gift of money, and not just a gift of an interest in the trust. This vehicle is especially appropriate when the income from the corpus is uncertain or may vary from year to year. Another benefit this trust provides is the unlimited charitable deduction.8 This allows the grantor to circumvent the individual percentage restrictions.

6. Allowable beneficiaries

A CLT can have one or more charitable beneficiaries and one or more noncharitable remaindermen. However, the failure of the grantor to name specific charitable beneficiaries will not disqualify the CLT. The remaindermen can be individuals (including the grantor), partnerships, corporations, trusts, or estates. The governing trust document should allow for the selection of alternate charitable beneficiaries in case one of the original charitable beneficiaries were to lose its tax-exempt status or cease to exist. If the grantor was also the trustee, the ability to name alternate beneficiaries would cause the transfer to the trust to be considered an incomplete gift, thereby disallowing the charitable tax deduction. Nonetheless, a grantor can be the trustee if another special trustee is named for the sole purpose of selecting substitute charitable beneficiaries.9

7. IRS provides sample forms for CLATs

The IRS has issued sample trust forms for both inter vivos CLATs in Rev. Proc. 2007-45, 2007-2 CB 89, and for testamentary CLATs in Rev. Proc. 2007-46, 2007-2 CB 102. Generally, the Service will not issue a letter ruling on whether a CLAT qualifies for estate and/or gift tax charitable deductions.

D. Valuation and the computation of charitable deductions

1. In general

In general, the charitable deduction of the grantor is equal to the present fair market value of the guaranteed payment or unitrust interest. To maximize the deduction, the term of the CLT should be long and the payout rate should be high. This will serve to increase the amount paid to charity over the term of

7 Rebecca K. Crown Income Charitable Fund (93-2 USTC ¶50,645). 8 I.R.C. §642(c). 9 PLR 9629009.

Page 124: Advanced Selected Issues for Trusts, Estates, and Their ... · 28/07/1997  · 3. Availability of §179 expensing 2 C. Administrative stance 2 1. Estates 2 2. Trusts 3 D. Judicial

cpenow.com / [email protected] Copyright © 2016 Surgent McCoy CPE, LLC -- ITEB/16/01 6-6

the trust, but will lower the value of the remainder of the trust’s assets passing to the private individual upon termination of the trust. There is no maximum term of years for a CLT unlike a CRT, which, if the term is stated in a number of years, cannot exceed 20 years. The valuation of a guaranteed annuity or unitrust interest after April 30, 1989 is calculated using §7520, which requires the use of the valuation tables issued by the IRS with an assumed interest rate (rounded to the nearest 0.2 percent) equal to 120 percent of the §1274(d)(1) federal midterm rate for the month in which the transfer occurs, compounded annually. The grantor can opt to use the §7520 rate from either of the two months preceding the month of transfer if it would be more advantageous.10 Examples of the valuation of a guaranteed annuity payment for a unitrust interest are depicted below.

2. Valuation of guaranteed annuity interest

The valuation method is outlined in the regulations.11 The present value of the annuity is calculated by multiplying the amount of the annuity by the appropriate annuity factor found in Table B (for a term of years) or Table S (for the life of an individual) in IRS Publication 1457, Actuarial Values Aleph Volume. The annuity factor to be used will depend upon the applicable §7520 rate and the term of the annuity. If Publication 1457 is unavailable, an annuity factor may be calculated from Table B or Table S in Treas. Regs. §20.2031-7(d)(6) by: (i) subtracting the applicable Table B or Table S remainder factor from 1.0; and (ii) dividing the result by the applicable §7520 interest rate.

Example 1: G establishes an inter vivos CLT to pay a guaranteed annuity of $250,000 per year at year-end for 15 years to Charity C. G transfers property with a fair market value of $5 million to the CLT. The §7520 interest rate 4 percent and the annuity factor from Publication 1457 is 11.1184. The values of the guaranteed annuity and remainder interests for income-, estate-, and gift-tax purposes are computed below:

Fair market value of property $5,000,000 less: Value of annuity interest 11.1184 x $250,000

2,779,600

Value of remainder interest $2,220,400

Example 2: The facts are the same as in Example 1, except the term of the trust is 10 years. Therefore, the annuity factor changes to 8.1109.

Fair market value of property $5,000,000 less: Value of annuity interest 8.1109 x $250,000

2,027,725

Value of remainder interest $2,972,275

3. Valuation of unitrust interest

The valuation method is outlined in the regulations.12 The present value of a unitrust interest is calculated by subtracting the present value of all interests in the transferred property other than the unitrust interest (i.e., the remainder interest) from the fair market value of the transferred property at the date of transfer. Both the frequency of the payments and the number of

10 Treas. Regs. §§1.7520-2(b), 20.7520-2(b), and 25.7520-2(b). 11 Treas. Regs. §§1.170A-6(c)(3)(i), 20.2031-7(d)(2)(iv), and 25.2512-5(d)(2)(iv). 12 Treas. Regs. §§1.170A-6(c)(3)(ii), 20.2055-2(f)(2)(v), and 25.2522(c)-3(d)(2)(v).

Page 125: Advanced Selected Issues for Trusts, Estates, and Their ... · 28/07/1997  · 3. Availability of §179 expensing 2 C. Administrative stance 2 1. Estates 2 2. Trusts 3 D. Judicial

cpenow.com / [email protected] Copyright © 2016 Surgent McCoy CPE, LLC -- ITEB/16/01 6-7

months by which the valuation date precedes the first payment in the first full year of the trust are relevant in determining the value of the unitrust interest. These facts are then applied to Table F in Treas. Regs. §1.664-4(e)(6) to arrive at a factor for an adjusted payout rate. The actual payout rate is multiplied by that factor to arrive at the adjusted payout rate. The adjusted payout rate, as well as the term of the income interest, are applied to Table D in Treas. Regs. §1.664-4(e)(6) for income interests measured by a term of years or to Table U(1) in the same regulation in the case of unitrust interest measured by a single life. In most cases, it will be necessary to use an interpolated factor from Table D since the adjusted payout rate will probably not exactly match the percentages shown in Table D. IRS Publication 1458, Actuarial

Values Beth Volume, contains the unitrust factors for two lives.

Example: John Henry established an inter vivos charitable lead trust on January 1 to pay a unitrust interest of eight percent of the trust’s corpus (as valued at the end of the preceding quarter) to Louisiana Tech University. Payment will be quarterly and payable for 12 years. He transfers $1 million to the trust. Assume the §7520 rate for January is 4 percent. Under Table F, the appropriate adjustment factor is 0.0990243 for quarterly payments. The adjusted payout rate is 7.922 (8 percent x 0.0990243). Based on the remainder factors in Table D, the present value of the remainder interest is $371,276 and the value of the unitrust interest is $628,724, computed as follows:

Table D Factor at 7.8 percent for 12 years 0.377373 Factor at 8.0 percent for 12 years 0.367666 Difference 0.009707

Interpolation adjustment:

7.922%-7.8% x 0.2% = 0.009707

x = 0.006097

Factor at 7.8 percent for 12 years 0.377373 Less: Interpolation Adjustment 0.006097 0.371276

PV of remainder interest: $1,000,000 x 0.371276 = $371,276

Value of the unitrust interest = $1,000,000 - $371,276 = $628,724

4. Reporting requirements

Each year the CLT must file a 1041 income tax return. Any trust that claims a §642(c) charitable deduction or any taxable split-interest trust must also file a Form 1041-A, Trust Accumulation of

Charitable Amounts. Furthermore, all taxable split-interest trusts, which includes all qualified and grantor CLTs, are treated as private foundations under §4947 and must file Form 5227, Split-Interest Trust

Information Return. All of these forms are due on or before the fifteenth day of the fourth month following the end of the taxable year of the CLT. If the CLT is treated as a grantor trust under §§671-678, all of the trust income is treated as income of the grantor and is reported by the grantor on the grantor’s personal income tax return. A Form 1041 must be filed with a separate statement identifying the grantor and listing the income and expense information that will be reported on by the grantor.

Page 126: Advanced Selected Issues for Trusts, Estates, and Their ... · 28/07/1997  · 3. Availability of §179 expensing 2 C. Administrative stance 2 1. Estates 2 2. Trusts 3 D. Judicial

cpenow.com / [email protected] Copyright © 2016 Surgent McCoy CPE, LLC -- ITEB/16/01 6-8

5. Generation-skipping transfer (GST) tax

The grantor of a split-interest charitable trust is making two simultaneous gifts upon creation of the trust. That is, the grantor is making one to charity and one to a noncharitable beneficiary who is generally a family member. In the event that a noncharitable beneficiary can be defined as a skip person, the GST tax will apply to distributions made from the trust (§§2613(a) and 2651). The impact of this tax can be offset by allocating a portion of the grantor’s GST exemption to the trust.

6. Nonqualifying nongrantor trust

A CLT can be nonqualified if it does not have a lead interest that is structured as a guaranteed annuity or unitrust interest. There are no estate-, gift-, and/or income-tax charitable deductions at the funding of the trust, but the grantor receives a gift-tax charitable deduction each year when income is distributed to the charitable beneficiary. The governing trust document for a nonqualified CLT requires that all the trust income be paid to the charitable beneficiary each year. Despite the fact that there is no upfront charitable deduction, there are several benefits of creating a nonqualified CLT. One of these benefits is that the trust is taxable on the income instead of the grantor. As a result, the grantor, via the trust, receives an unlimited §642(c) deduction equal to the amount of income earned, leaving the trust with zero tax liability. Second, because the gift-tax charitable deduction generated by a nonqualifying CLT is considered a transfer of money as opposed to an interest in a trust, the private-foundation rules do not apply.13

7. Grantor trust

The grantor CLT allows the grantor to take a charitable income-tax deduction at the time of funding the trust. However, this deduction is limited to 30 percent of the grantor’s contribution base (usually adjusted gross income (AGI)) in the year of the transfer. The typical 50-percent-of-AGI limitation is not applicable to contributions from a grantor CLT’s due to a Tax Court holding.14 In this case, the court found that the contributions from a grantor CLT were for the use of (rather than to) a charitable organization, and therefore, subject to 30-percent-of-AGI limitation. In addition, any unused portion of the deduction is subject to a five-year carryover.15

8. Property contributed to a charitable trust

The property contributed to a split-interest trust should generate enough income to fund the guaranteed annuity payments or the unitrust interest. Property encumbered with debt can be contributed to a charitable trust. However, when the indebtedness exceeds the grantor’s basis in the property, then the grantor will recognize gain on the transfer. The ideal type of property to contribute to a CLT is property that is appreciating in value. This type of property should generate enough income to fund the charitable distributions and provide an enhanced capital asset to the remaindermen at the termination of the trust. Naturally, appreciating property is contributed to a CLT to remove the appreciation from the grantor’s estate.

13 See Rev. Rul. 77-275, 1977-2 C.B. 346. 14 James v. Commissioner (62 TC 209)(1974). 15 I.R.C. §170(b)(1)(B).

Page 127: Advanced Selected Issues for Trusts, Estates, and Their ... · 28/07/1997  · 3. Availability of §179 expensing 2 C. Administrative stance 2 1. Estates 2 2. Trusts 3 D. Judicial

cpenow.com / [email protected] Copyright © 2016 Surgent McCoy CPE, LLC -- ITEB/16/01 7-i

Hot Topics in Fiduciary Taxation

Learning objectives 1 I. Sham trusts 1

A. Introduction 1 B. Background 2

1. Legal trust qualities 2 2. Trustee duties 2

C. Legitimate trusts 3 D. Sham trusts 3

1. Characteristics of a sham 3 2. Additional warning signs 4 3. Types of sham trusts 5

E. Domestic trusts 6 1. Sham trust avoidance 7

F. Analyzing trusts 8 1. Substance versus form 8 2. Four factors 8 3. Other guiding principles 9

G. Additional judicial decisions 9 H. Recent news on criminal penalties 10 I. Works cited 11

II. Special needs trust 11 A. Introduction 11 B. Classification 14

1. d(4)(A) SNT 14 2. d(4)(C) SNT 15 3. Third party SNT 15

C. Taxation of SNT 16 1. The concept of income 16 2. Grantor trust vs. non-grantor trust 16

D. Self-settled SNT taxation 18 1. Income tax 18 2. Gift tax 19 3. Estate tax 19

E. Third party SNT taxation 19 1. Income tax 19 2. Gift and estate taxes 20

F. Conclusions 20 G. Works cited 21

Page 128: Advanced Selected Issues for Trusts, Estates, and Their ... · 28/07/1997  · 3. Availability of §179 expensing 2 C. Administrative stance 2 1. Estates 2 2. Trusts 3 D. Judicial
Page 129: Advanced Selected Issues for Trusts, Estates, and Their ... · 28/07/1997  · 3. Availability of §179 expensing 2 C. Administrative stance 2 1. Estates 2 2. Trusts 3 D. Judicial

cpenow.com / [email protected] Copyright © 2016 Surgent McCoy CPE, LLC -- ITEB/16/01 7-1

Hot Topics in Fiduciary Taxation Learning objectives

After reviewing this chapter, the reader will be able to: • Distinguish a Valid Trust from a Sham Trust arrangement; • Identify characteristics that could lead to a sham trust; • Explain the requirements of the new IRS Disclosure Initiative for hidden offshore

accounts or entities; • Distinguish between foreign and domestic sham trusts; • Determine the penalties and compliance forms for offshore transfers by individuals; • Identify the four factors that can be used to gauge the economic substance of a trust; • Understand the place of a special needs trust (SNT) in an estate plan; • Identify the items that a SNT may pay for and not disqualify the person from SSI or

Medicaid benefits; • Distinguish between self-settled and third party SNTs; • Determine which SNTs must contain a mandatory Medicaid payback provision; • Discuss both uses and misuses of SNTs; and • Understand how the failure to preserve public benefits eligibility can occur.

I. Sham trusts

A. Introduction The legitimacy of trusts has been a highly controversial topic for many years. Numerous court cases have focused on deciding whether an established trust is legitimate for tax purposes or a sham. Certain characteristics will distinguish the true trusts from the abusive ones. Once a set of requirements is met, the Internal Revenue Code will recognize the trust as a taxable entity and tax the income to the trust and/or beneficiaries but not to the grantor. Trusts can be considered shams from the beginning or develop into such over a period of time. Advertisers of abusive trusts can be very persuasive. They will promote benefits such as keeping control of assets in a trust, while not having to pay taxes on the income from the assets. The IRS hopes that taxpayers will be skeptical of these abusive trust arrangements and realize that these benefits are not realistic. Participating in abusive trusts will subject the taxpayer and the seller of such trusts to civil and even criminal repercussions.1 Sham trusts have been presented in an array of forms over the years. With many warning signs and characteristics of sham trusts available in the literature, taxpayers/advisors can hopefully segregate a legitimate taxable trust from a sham. When taxpayers fail to properly analyze the characteristics of a trust, numerous judicial decisions have demonstrated that the courts have no problem drawing a conclusion for them. After examining qualities and types of lawful trusts, the traits and forms of sham trusts will be discussed. This discussion will be followed with ways to avoid shams, suggestions on how to analyze a trust, and further cases and recent news surrounding trusts.

1 CCH, “Taxpayers Hiding Assets Offshore Have Until August 31 to Disclose,” Federal Tax Course Letter, March 2011,

Volume 25 Issue 3, 1-5.; “IRS Trust Sham Notice,” The Tax Prophet, 1997.

Page 130: Advanced Selected Issues for Trusts, Estates, and Their ... · 28/07/1997  · 3. Availability of §179 expensing 2 C. Administrative stance 2 1. Estates 2 2. Trusts 3 D. Judicial

cpenow.com / [email protected] Copyright © 2016 Surgent McCoy CPE, LLC -- ITEB/16/01 7-2

B. Background

1. Legal trust qualities

A trust is formed when a grantor transfers assets to a trustee to preserve for the care of a beneficiary. Trusts can be formed during the grantor’s life, inter vivos, or by a decedent’s will, testamentary. Based on certain requirements, trusts are considered either simple or complex. Trusts are often created to manage assets for young children, incompetent individuals, or taxpayers not interested in handling the assets. For a trust to be considered legal for tax purposes, it must consist of “a manifestation of intention of the trustor to create a trust, a lawful trust purpose, trust property, and at least one identifiable beneficiary.”2 Failure to meet all four of these requirements dismisses a trust as a legal taxable entity. Taxpayers with a legal trust must file a trust federal income tax return, Form 1041. Lawful reasons for creating a trust include facilitating estate planning, transferring charitable property, and holding assets for other taxpayers.3 Trusts are usually formed to hold income-generating property. The principal amount remains in the trust, while the produced income is paid out to a beneficiary. Valid trusts are allowed a deduction from taxable income for the amount of distributions paid to the beneficiaries. To be considered a taxable trust under trust law, a trust must have a legitimate business purpose and contain economic substance. Taxpayers should be constructing trusts with a sensible possibility of making a profit. The intentions of the taxpayers creating trusts are crucial. The lone objective of the trust cannot be tax avoidance or else the IRS will ignore the trust and enforce the tax liability on the grantor or beneficiary.4 For example, grantors may try to create multiple trusts to distribute income to more than one trust and in return get multiple exemptions. Where trusts are created essentially by the same grantor for the same beneficiaries, the multiple trusts will be deemed a single trust and taxed, accordingly.

2. Trustee duties

After considering the true purpose of the trust, another important issue to evaluate is the role of the trustee. The trustee is granted an array of decision making options when managing trust assets. Coinciding with the trust agreement, trustees must perform actions such as operating the trust to solely benefit the beneficiary, keeping up with trust assets, and filing appropriate tax returns for the trust. The trustee should be aware of and follow the reason for the creation of the trust. Even though the trustee has been assigned to manage the trust assets, trustees should not be hesitant to seek outside assistance when needed. Discussing and documenting decisions made by the trustee is always a good idea so everyone is informed of how the assets are being handled. Because of the responsibilities involved, a trustee is often allowed realistic compensation.5 Since the trustee is accountable for stewardship of the assets of a trust, the beneficiary has an inactive role. The lone responsibility of the beneficiary is to accept the benefits provided by the trustee’s handling of the trust assets. Beneficiaries are classified as either income beneficiaries or remainder beneficiaries. Examples of agreements generally lacking the special relationship include trusts formed to hold real estate property, custodial arrangements, and agencies. The only circumstance where a real estate trust will be treated as a trust for income tax purposes is if the trust is passively undertaking real estate actions. The agency and custodial arrangements are not considered trusts for similar reasons. Even though the agent is acting for the principal in an agency agreement, the principal still maintains extensive control

2 Sommers, Robert L., “Sham Trust Analysis,” The Tax Prophet, 2000. 3 “IRS Trust Sham Notice,” The Tax Prophet, 1997. 4 Martindale-Hubbell, “Trust Shams and IRS Investigations,” Lawyers.com, 2010. 5 Sommers, Robert L., “Sham Trust Analysis,” The Tax Prophet, 2000.

Page 131: Advanced Selected Issues for Trusts, Estates, and Their ... · 28/07/1997  · 3. Availability of §179 expensing 2 C. Administrative stance 2 1. Estates 2 2. Trusts 3 D. Judicial

cpenow.com / [email protected] Copyright © 2016 Surgent McCoy CPE, LLC -- ITEB/16/01 7-3

over the agents’ actions. Likewise in a custodial arrangement, the custodian has minuscule responsibilities in overseeing the property.

C. Legitimate trusts Trusts can be formed to benefit many different individuals such as an alimony trust. One category of trusts that is focused more on the intention of the trust than the beneficiaries is the purpose trust. This type of trust is formed for a particular motive rather than to benefit beneficiaries. Purpose trusts can be constructed for charitable or non-charitable reasons. Examples of purpose trusts include funeral trusts and cemetery trusts. A funeral trust consists of an arrangement initiated by the grantor to prepay for funeral expenses prior to death. Cemetery trusts are further broken down into two types, including the endowment care cemetery trust and the service and merchandise trust. The endowment care cemetery trust is established to use the earnings, not the trust principal, to pay for the maintenance and upkeep of the gravesite. Similar to the funeral trust, the service and merchandise trust is created to compensate for merchandise such as a tombstone.6 Another kind of trust is the blind trust, which is often utilized by public officials and business executives to diminish concerns surrounding conflict of interests and other regulations. These individuals normally want to hold a particular type of investment but desire to restrict their knowledge of how the investment is managed. Blind trusts can be considered either a qualified blind trust or a qualified diversified trust. A further example is the Coogan trust, a unique type of trust created specifically for child actors and actresses. As to no surprise, the Coogan trust originated in Hollywood, named after child actor Jackie Coogan. This trust is basically a blocked trust account that is enacted to prevent the parents of a child performer from wasting all of the child’s earnings. The Coogan trust is required in Louisiana, New York, California, and New Mexico.7 Additional examples of eligible trusts for income tax purposes are liquidating trusts, mortgage pools, and investment trusts. A liquidating trust, consisting of an agreement to liquidate and allocate assets, is taxed like a trust when the actions follow liquidation. If the investments of mortgage pools and investment trusts are set and not allowed to be altered by the trustee, they are taxed like a trust. A further illustration of a valid trust is the life insurance trust. The beneficial idea behind this trust is a taxpayer releases asset ownership to enhance the amount of assets excluded from the taxable estate. This type of trust is beneficial because of its ability to turn small transfers of money into hefty tax-free sums. After the trust pays the insurance premiums (usually, via gifts to the trusts with Crummey powers), the insurance proceeds ultimately will transfer to the beneficiaries free of estate taxes. To avoid having assets brought back into the estate because of excessive control, life insurance trusts should have restrictions on incidents of ownerships, such as the right to alter beneficiaries.8

D. Sham trusts

1. Characteristics of a sham

Trusts that are established with no true purpose and are absent of substance are considered shams. Because the IRS does not recognize sham trusts for income tax purposes, the income and expenses are

6 Goffe, Wendy S., “An Introduction to Lesser-Known But Useful Trusts – Part 2,” Estate Planning, v. 37, August 2010, 3-

11. 7 Goffe, Wendy S., “An Introduction to Lesser-Known But Useful Trusts – Part 2,” Estate Planning, v. 37, August 2010, 3-

11. 8 Burg, Brad, “Can You Spot a Phony Trust,” Medical Economics, v. 76, January 1999, 137-141.

Page 132: Advanced Selected Issues for Trusts, Estates, and Their ... · 28/07/1997  · 3. Availability of §179 expensing 2 C. Administrative stance 2 1. Estates 2 2. Trusts 3 D. Judicial

cpenow.com / [email protected] Copyright © 2016 Surgent McCoy CPE, LLC -- ITEB/16/01 7-4

instead usually assigned either to the grantor or beneficiary. Abusive trusts are notorious for promising tax benefits. Several sham trusts attempt to take advantage of the deduction allowed to a trust for the distributions to beneficiaries by designating other trusts as beneficiaries. After making distributions from trust to trust and taking the subsequent deductions, the end result is minimal taxable income. As the IRS persistently investigates sham trust allegations, common benefits offered consist of lowering or removing taxable income, allowing deductions for personal expenses paid by the trust, lessening or abolishing self-employment taxes, and reducing or eliminating gift and estate taxes.9 Certain signs can indicate a shady trust promoter or even an outright sham trust. Creating numerous layers of trusts for no apparent reason other than to puzzle the IRS can be indicative of a sham. An additional selling point for sham trust advocates is insisting on placing a business in a trust. Even though doing so is not illegal, generally there is not a lawful reason to perform such an action. Also, suggesting that the taxpayer’s adviser (CPA and/or attorney) is not aware of a particular type of trust arrangement because of the complexity is not a positive sign. Another warning sign is when an advertiser states that a taxpayer can control assets while not actually owning them. In an effort to reduce his/her taxable estate, such a statement would sound extremely appealing to a taxpayer. However, a claim of this sort is normally not true since the controller of the assets is the one who usually pays the taxes.10 Sandvall v. Commissioner11 demonstrates a situation where the taxpayers did not relinquish control of the assets placed in a trust. A chiropractor and his wife tried to evade taxes by establishing a foreign trust, while using the assets for personal benefit. The foreign trust was designed to allow the husband’s income from his business to flow through it before being paid back to the couple. Because the trust was determined to be a sham, the couple was responsible for paying taxes on the gross income of the trust. The court ruled that the taxpayers “did not relinquish ownership and control of their earnings, they merely created a fictitious paper trail by which they hoped to disguise or hide their taxable income.”12

2. Additional warning signs

A further area of concern is when a taxpayer is told income can be assigned to a trust and not be taxed to the taxpayer who actually earned the income. This statement proved to be false in U.S. v. Buttorff,13 where the court determined that the “Constitutional Pure Equity Trusts” were shams. The seller of the trusts alleged that individuals could assign future earned income to the trust. The Circuit Court of Appeals drew support for their decision from the Code’s provision that “income is taxed to the person who earns it.”14 The Keefover v. Commissioner15 case further supports this tax principle. The court informed the individuals that a taxpayer cannot escape paying taxes by shifting income to another entity. A taxpayer is not allowed to transfer tax obligations to a trust when the taxpayer is in complete control of earning the income.16 Sham trusts can be labeled by an array of names such as pure trusts, freedom trusts, contract trusts, or common law trust organizations. As illustrated above, sham trusts will often include descriptions

9 Martindale-Hubbell, “Trust Shams and IRS Investigations,” Lawyers.com, 2010. 10 Novack, Janet, “If It Sounds Too Good,” Forbes, 2000, 178-179. 11 90-1 USTC (1990). 12 Sommers, Robert L., “Sham Trust Analysis,” The Tax Prophet, 2000. 13 761 F2d 1056 (1985). 14 Sommers, Robert L., “Sham Trust Analysis,” The Tax Prophet, 2000. 15 65 T.C.M. 2999 (1993). 16 Sommers, Robert L., “Sham Trust Analysis,” The Tax Prophet, 2000.

Page 133: Advanced Selected Issues for Trusts, Estates, and Their ... · 28/07/1997  · 3. Availability of §179 expensing 2 C. Administrative stance 2 1. Estates 2 2. Trusts 3 D. Judicial

cpenow.com / [email protected] Copyright © 2016 Surgent McCoy CPE, LLC -- ITEB/16/01 7-5

surrounding subjects like religion, patriotism, or constitutional rights.17 When analyzing the issue of potential sham trusts, the following are additional characteristics that could lead to a sham trust:

• No paper trail exists, which suggests the trust has not been correctly managed; • The grantor or trustee is also a beneficiary; • The trust designer does not fully comprehend the nature of the trust; • The trustee, without asking questions, complies with any instruction given by the grantor; • Using a “letter of wishes” to take precedence over the trust instrument, which shows the

granter is still in control of the assets; and • The decisions are made “unilaterally without a majority decision” or without following the

established guidelines. A trust has greater potential of being ruled a sham when any of the preceding elements exist.18 However, a trust possessing negative connotations does not automatically qualify that trust as a sham. Certain traits that are not exclusively indicative of a sham trust are careless execution of the trust, disapproval of the courts, and ulterior purposes.19

3. Types of sham trusts

The IRS believes sham trusts can be categorized into two general categories – foreign trusts and domestic trusts – with various forms of shams existing under each heading. The foreign trust is developed outside of the United States, frequently located in countries with minimal income tax on trusts. New York attorney Gideon Rothschild states, “There are special rules for foreign trusts, and they can be very restrictive.”20 Foreign trusts, also referred to as final trusts, involve income flowing through an assortment of trusts, offshore bank accounts, and other foreign entities before ultimately being paid to the grantor. Resources flow from one trust to the next through methods such as “management agreements, rental agreements, fees for services, purchase and sale agreements, and distributions.”21 The theory behind foreign trusts is income will maneuver through enough foreign accounts that the IRS will lose sight of who is actually controlling the assets of the trust. The promoters of this type of sham insist that the final distribution made to the grantor will not be taxed; however, the promoters are often proven wrong. That is, the Service is frequently able to show that the grantor is still in control of the trusts and will tax the income to that person. A good rule of thumb to remember is a legal taxable trust will not attempt to escape taxation by hiding assets or offering a tax deduction for “personal, living, or educational expenses.”22 Taxpayers hiding assets offshore had until August 31, 2011 to disclose. The Service recently announced its Second Offshore Voluntary Disclosure Initiative. This program is designed for taxpayers that setup undisclosed foreign accounts or entitles to avoid or evade tax into compliance with U.S. tax laws.23 This new initiative was only accessible through 8/31/11. Only individuals, corporations, partnerships, and trusts were eligible to take advantage of this disclosure initiative.

17 Goffe, Wendy S., “An Introduction to Lesser-Known But Useful Trusts – Part 2,” Estate Planning, v. 37, August 2010, 3-

11. 18 Jooste, Ruan, “Ain’t it a Sham(e),” Finance Week, May 2005, 98-99. 19 Kessler, James, “What is and What’s Not a Sham,” Drafting Trusts and Will Trusts, 1999, 125. 20 Burg, Brad, “Can You Spot a Phony Trust,” Medical Economics, v. 76, January 1999, 137-141. 21 Goffe, Wendy S., “An Introduction to Lesser-Known But Useful Trusts – Part 2,” Estate Planning, v. 37, August 2010, 3-

11. 22 Martindale-Hubbell, “Trust Shams and IRS Investigations,” Lawyers.com, 2010. 23 CCH, “Taxpayers Hiding Assets Offshore Have Until August 31 to Disclose,” Federal Tax Course Letter, March 2011,

Volume 25 Issue 3, 1-5.

Page 134: Advanced Selected Issues for Trusts, Estates, and Their ... · 28/07/1997  · 3. Availability of §179 expensing 2 C. Administrative stance 2 1. Estates 2 2. Trusts 3 D. Judicial

cpenow.com / [email protected] Copyright © 2016 Surgent McCoy CPE, LLC -- ITEB/16/01 7-6

Taxpayers may face numerous penalties when they fail to comply with the new disclosure initiative as depicted in Figure 1. Because the IRS was quite successful with its first initiative, that is, more than 15,000 taxpayers came forward voluntarily and reported their previously undisclosed assets and foreign accounts,24 the Service is hopeful that the new initiative will enjoy the same success as the previous program. In October of 2015, the IRS announced that the compliance programs had generated over $8 billion in revenue.

E. Domestic trusts A common domestic trust is the business trust or unincorporated business trust. This type of sham trust involves a grantor fictitiously handing over the operation of a business to a trust. Specifically, in substance, when in reality the grantor is still controlling the daily business operations through the trustee. The taxpayer receives units of beneficial interest in return for transferring the business. Then, the trust makes disbursements to the unit holders mimicking a deductible distribution. Not only is the taxpayer claiming to reduce taxable income but also self-employment taxes.

Figure 1 Major Penalties and Compliance Forms for Offshore Transfers by Individuals

Forms Required or Code Sections Violated

Penalties for Failure to File

Form TD F 90-22.1 (FBAR) The greater of $100,000 or 50% of the total balance of the foreign accounting per violation.

Form 3520 35% of the gross reportable amount or where gifts are involved, then 5% per month up to maximum of 25% of the gift.

Form 3520-A 5% of the gross value of trust assets determined to be owned by the U.S. person.

Form 5471 $10,000 for failure to file with $10,000 per month added for each month’s failure to file with a maximum of $50,000.

Form 5472 $10,000 for failure to file with $10,000 per month added for each month’s failure to file with a maximum of $50,000.

Form 926 Penalty of 10% of the value of the property transferred with a maximum of $100,000. However, no limit if failure to report was intentional.

Form 8865 Penalty of $10,000 for failure to file with $10,000 per month added for each month’s failure to file with a maximum of $50,000, and 10% of the value of any transferred property that goes unreported is subject to a $100,000 limit.

Code Section 6663(a) – Civil Fraud Penalty is 75% of the underpayment. Code Section 7201 – Criminal Fraud

Fine is $100,000 or imprisonment of up to 5 years, or both, coupled with the cost of prosecution.

Code Section 6662(a) – Accuracy-related Penalties

20% penalty on the full amount of underpayments of tax for all years.

Another abusive trust example is the equipment trust. This sham trust is designed to hold equipment that is leased, frequently at an inflated rate, to the business trust. The agreement is set up to allow the

24 CCH, “Taxpayers Hiding Assets Offshore Have Until August 31 to Disclose,” Federal Tax Course Letter, March 2011,

Volume 25 Issue 3, 1-5.

Page 135: Advanced Selected Issues for Trusts, Estates, and Their ... · 28/07/1997  · 3. Availability of §179 expensing 2 C. Administrative stance 2 1. Estates 2 2. Trusts 3 D. Judicial

cpenow.com / [email protected] Copyright © 2016 Surgent McCoy CPE, LLC -- ITEB/16/01 7-7

business trust the ability to make deductible payments to the equipment trust, thereby reducing taxable income.25 The family residence trust is an additional variation of sham trusts in which a couple places their house in a trust. The trust pretends to rent the home back to the couple, while the taxpayers claim to live in the home to benefit the trust. To reduce taxable income, the trust takes deductions for depreciation and expenses related to the maintenance of the home. Once this form of a sham trust is discovered by the IRS, the couple is taxed on the income and prohibited from taking the expense deductions. A final domestic sham trust example is the charitable trust. In this scenario, the taxpayer shifts assets into what is claimed to be a charitable organization. Then, the trust pays the taxpayer’s personal expenses and claims deductions for the expense payments. Since the so-called charitable organizations are not qualified as true charitable organizations, the contributions made are not deductible and are included in the taxpayer’s taxable income.26

1. Sham trust avoidance

Taxpayers should take every precaution to prevent falling into a sham trust trap. Avoid sham trusts in the early stages of drafting. The possibility of drafting oneself into a sham trust is much greater than the unlikelihood of drafting oneself out of such an arrangement. A trust can become a sham during drafting by generating false documentation for the trust or failing to accurately portray the true intention of the grantor. The possibility of a trust being ruled a sham increases as the difference between the trust documentation and the reality of the trust expands. To avoid a sham, another aspect to scrutinize is the administration of a trust. The grantor needs to fully understand and intend to create a trust. Naturally, this means that the grantor possesses the willingness to no longer own the assets. Also, the grantor should transfer all managerial powers over the trust assets to the trustee. A trustee should not allow a grantor to feel as if the grantor has all the power and the trustee will do as told. A wise decision for trustees is to maintain a paper record of their decision making process.27 Pristine record keeping by the trustee will make claims of sham trusts more difficult to prove. In an effort to avoid shams, taxpayers are turning more often to independent trustees to be protectors of trust assets. An independent trustee exists when no more than 50% of the trustees are related or subordinate parties. The increased need for trust overseers creates an opportunity for the accounting profession and Certified Public Accountants (CPAs). Because of the perceived independence and quality professional judgment, a CPA is thought to be one of the best choices as a protector of a trust. This new demand for a CPA’s ability fuels the growth of the assurance sector in the accounting profession. Accompanying the already established tax and audit divisions, assurance services are widely needed and continue to grow. One area where CPAs can utilize their tax expertise is when offering tax structure assurance. In this sense, a CPA guarantees the grantor that correct tax planning strategies have been implemented. Also, a CPA can offer assurance as an enforcer in partner and contract compliance situations when dealing with purpose trusts. Because purpose trusts do not have beneficiaries, these trusts need to fill the enforcer role. “Users turn to CPAs for assurance services because of their reputation for integrity, objectivity and due professional care.”28 As taxpayers try to avoid sham trusts, a recent area of concern is the blind discretionary trust. This style of trust is an irrevocable discretionary trust that initially has charitable organizations as the beneficiaries 25 “IRS Trust Sham Notice,” The Tax Prophet, 1997. 26 Martindale-Hubbell, “Trust Shams and IRS Investigations,” Lawyers.com, 2010. 27 Kessler, James, “What is and What’s Not a Sham,” Drafting Trusts and Will Trusts, 1999, 125. 28 Vernazza, J. Ben, “The Independent Protector in Overseas Trusts,” Trusts & Trustees, v. 6, February 2000, 22-27.

Page 136: Advanced Selected Issues for Trusts, Estates, and Their ... · 28/07/1997  · 3. Availability of §179 expensing 2 C. Administrative stance 2 1. Estates 2 2. Trusts 3 D. Judicial

cpenow.com / [email protected] Copyright © 2016 Surgent McCoy CPE, LLC -- ITEB/16/01 7-8

but maintains the authority to replace beneficiaries when deemed necessary. Blind discretionary trusts have raised the level of concern by the IRS surrounding offshore trusts. Specifically, trust regulators hope the use of this type of trust will decline, but some experts believe the use of blind discretionary trusts might actually increase as the push for accounting transparency intensifies.29 Regardless of which side of the argument is correct, the use of blind discretionary trusts is an issue that the IRS watches, closely.

F. Analyzing trusts

1. Substance versus form

A number of reputable tax principles direct the accurate taxing of sham trusts. That is, sham trusts present a classic example of substance over form. When evaluating the legitimacy of a trust, the IRS and the judicial system will consider the economic substance of the transaction and not just the form. Although a taxpayer has a right to minimize his/her tax liability, transferring assets to a trust lacking a business purpose does not constitute a viable tax avoidance method. The trust must be established for other reasons than just trying to avoid taxes.30 For example, the Brittain v. Commissioner31 case involved an irrevocable trust that was deemed unrecognizable by the court based on the nonexistence of economic substance. The court said that the individual did not successfully show “that property was held in a trust for the benefit of others. Further, the petitioner’s relationship to the trust’s property did not differ in any material aspect before and after the creation of the trust.”32 In Markosian v. Commissioner33, the trust was ruled a sham because the participants involved failed to follow the conditions of the trust. Also, the relationship between the grantors and the trust property did not change once the assets were transferred. The Zmuda v. Commissioner34 decision involved a variety of trusts. Because the taxpayer maintained total control over the trust property, the foreign business trusts were ruled shams. After considering substance over form, the court ruled that the assets of the business trust and foreign trust, the home in the family residence trust, and the equipment in the equipment trust would revert back to the grantor for taxation. Two other applicable cases are Gregory v. Helvering35 and Helvering v. Clifford.36 Based on the substance over form standard, the trusts in both cases were ruled to be shams and discarded by the IRS for income-tax purposes.

2. Four factors

When trusts are created, the grantor, the beneficiary, or the trust will be taxed on the income derived from the trust property. Valid trusts will not attempt to take frivolous deductions or avoid taxes.37 In determining whether or not a trust will be considered a sham for income tax purposes, four factors can be used to gauge economic substance. This test was presented in the case of F.L. Buckmaster v. Commissioner,38 where the taxpayer established a trust to collect his compensation. The taxpayer was denied the ability to shift the taxation on his earnings to the trust because he controlled the earnings and the trust bank account. The four questions to be answered are as follows:

• Does the grantor’s relationship to the property before the trust was created vary significantly compared to the relationship after the creation of the trust?

29 Lambert, Ian T.G., “The KPMG Report,” Trusts & Trustees, v. 7, November 2000, 8-11. 30 “IRS Trust Sham Notice,” The Tax Prophet, 1997. 31 63 T.C.M. 3004 (1992). 32 Sommers, Robert L., “Sham Trust Analysis,” The Tax Prophet, 2000. 33 73 T.C. 1235 (1980). 34 731 F.2d 1417 (9th Cir. 1984). 35 293 U.S. 465 (1935), XIV-1 C.B. 193. 36 309 U.S. 331 (1940), 1940-1 C.B. 105. 37 “IRS Trust Sham Notice,” The Tax Prophet, 1997. 38 T.C. Memo. 1997-236.

Page 137: Advanced Selected Issues for Trusts, Estates, and Their ... · 28/07/1997  · 3. Availability of §179 expensing 2 C. Administrative stance 2 1. Estates 2 2. Trusts 3 D. Judicial

cpenow.com / [email protected] Copyright © 2016 Surgent McCoy CPE, LLC -- ITEB/16/01 7-9

• Did the trust establish an independent trustee and restrict the grantor’s use of trust property?

• Did an economic interest transfer to other trust beneficiaries? • Was the grantor restricted by the laws of trust or the trust agreement?

After determining that a legal trust exists, answering these four questions will help guide a court when analyzing a trust as either a sham or a properly developed separate entity.39

3. Other guiding principles

Another guideline for taxing sham trusts states that grantors are able to be taxed as holders of the trust. In accordance with Code §§671 and 677, if the grantor maintains control over the trust, the grantor is considered the owner of the assets transferred to the trust. Code §679 further provides that a United States citizen transferring assets to a trust overseas is still the owner of the assets if the beneficiary is a United States citizen. The results would involve the owner reporting all income and expenses of the trust, while disregarding any tax deductions from losses among the owner and trust. A further principle deals with the taxation of a non-grantor trust. A valid trust not considered a grantor trust is taxed on the amount of income minus beneficiary distributions. In order to exist, the trust has to acquire a taxpayer identification number, file the required tax returns, and detail the beneficiary’s income deductions on the K-1 form.40 An additional taxing principle points out that trust transfers could be considered gifts for gift tax purpose, thus being liable for estate and gift taxes under the Internal Revenue Code. The federal estate tax would come in to play if the grantor held control of the property placed in the trust until death. Upon the grantor’s death, the property would then be levied a federal estate tax. A further point to remember for abusive trust tax guidance is that personal expenses are normally non-deductible. In contrast to what many sham trust promoters promulgates, Code §262 reminds taxpayers that personal expenses such as travel and education are not allowed as deductions from taxable income. Examples of cases that upheld this principles are Schulz v. Commissioner41 and Neely v. United States.42 Another standard is a valid charity must benefit in some manner in order for a legal charitable deduction to be consummated. Tax-exempt charitable trusts (Charitable Remainder Trusts and Charitable Lead Trusts) are well defined in the Internal Revenue Code under §§664, 508, 671-678, 642(c), 170, 2055, and 2522. As demonstrated in Fausner v. Commissioner,43 charitable trusts must be established exclusively to benefit a charity in order to secure exemption status. Moreover, trust payments to charities are not deductible if the payments are made to benefit the owner of the trust.

G. Additional judicial decisions The case of Federal Trade Commission v. Affordable Media, LLC44 involved a couple, Mr. and Mrs. Anderson, who were co-trustees of a trust in the Cook Islands. After being instructed by a district court to remove assets from their trust, the Andersons claimed that they could not follow the orders because of duress. The couple was sentenced to six months of jail time for contempt of court. The Ninth Circuit upheld the lower court’s decision. The Andersons were protectors of the trust, and had the power within

39 Sommers, Robert L., “Sham Trust Analysis,” The Tax Prophet, 2000. 40 “IRS Trust Sham Notice,” The Tax Prophet, 1997. 41 686 F.2d 490 (7th Cir. 1982). 42 775 F.2d 1092 (9th Cir. 1985). 43 55 T.C. 620 (1971). 44 179 F.2d 1228 (CA-9, 1999).

Page 138: Advanced Selected Issues for Trusts, Estates, and Their ... · 28/07/1997  · 3. Availability of §179 expensing 2 C. Administrative stance 2 1. Estates 2 2. Trusts 3 D. Judicial

cpenow.com / [email protected] Copyright © 2016 Surgent McCoy CPE, LLC -- ITEB/16/01 7-10

themselves to determine whether an event would be treated as an event of duress or not. Of course the finding implies that the trust was a sham. In the H. Paster45 case, a business trust organization was deemed null and void for federal income tax purposes. The taxpayer formed the sham trust to transfer the assets and income from his dental practice. The sham lacked economic reality based on the fact that the taxpayer’s relationship to the trust property stayed the same both before and after the transfer. G.E. Tatum46 demonstrates the attempted use of a trust to avoid self-employment taxes. The income earned by the sham business trusts was ruled to be income earned by partnerships operated by the taxpayer. Because the taxpayer ran the operations of the partnerships, he was responsible for self-employment taxes on the income. The taxpayer thought he could place the partnerships into business trusts to avoid the self-employment tax. The P.F. Spencer47 case involved a taxpayer structuring a trust so it would appear that the trust income was disbursed to foreign trust beneficiaries. Then, the income would be redistributed to other foreign trust beneficiaries. The trust in which the taxpayer transferred all his income and assets was ruled to be a sham. The IRS determined the trust’s only purpose was to eliminate income tax without relinquishing control of the transferred property.

H. Recent news on criminal penalties In the decision of H.L. Richardson,48 Homer Lee Richardson was sentenced to 30 months in prison on January 11, 2011 for marketing sham trusts for a former business, Aegis Company. In addition, he was assessed a $60,000 fine along with $61,212 in restitution. Also, the taxpayer filed false individual tax returns for himself which caused an understatement of income. The sham trusts lacked economic substance and business purpose, while pretending that Aegis members released control of the trust assets. Consequently, individuals who participated in the sham trusts filed falsified individual tax returns as a result. Mr. Richardson would help these clients escape being audited by sending threatening letters to the IRS and suggesting the clients not produce any documents the IRS requested.49 In December of 2010, Anne Marie Connor of Bethany Beach, Delaware was sentenced to two years in prison for filing false returns and utilizing sham trusts to escape taxes. Ms. Connor was required to pay $117,446 in restitution to the U.S. Treasury as well. Anna Conner funneled income from her two businesses, Bethany Massage and Healing Arts Center along with Wholesome Habits Health Food Store, to the established sham trusts. She also put the deed to her house in one of the trusts. While her money was in the trust, she maintained total control over the income. The promoter of the scheme, who worked for Innovative Financial Consultants, was also sentenced to prison. U.S. Attorney David C. Weiss said, “Today’s sentence makes it clear that those who scheme to evade taxes face substantial criminal penalties.”50 A press release from the U.S. Department of Justice states that a Utah taxpayer, Blayde Crockett, was sentenced to 70 months in federal prison for promoting and selling sham trusts. Bladye Crockett was convicted of preparing untrue tax returns and conspiracy to defraud the government. Evidence shows Mr. Crockett operated his scheme through two businesses, Business Research and Development as well as TNT Company. Details of the case explain that he reportedly sold abusive trusts to a family in Wyoming, who owned Nuway, Inc. Next, he told the family to put their wages into the sham trusts and pay for 45 20 T.C.M. 1239, T.C. Memo. 1961-240. 46 59 T.C.M. 52, T.C. Memo. 1990-119. 47 68 T.C.M. 1010, T.C. Memo. 1994-531. 48 91 T.C.M. 981, Dec. 56,475(M), T.C. Memo. 2006-69, aff’d, CA-6, Dec.11 2007. 49 “Cincinnati Area Man Pleads Guilty to Aegis-Related Tax Crimes,” PR Newswire, January 2011. 50 Whig, Cecil, “Bethany Beach Woman Gets Two Years in Prison for Using Sham Trust Accounts to Evade Taxes,”

TownNews.com, December 2010.

Page 139: Advanced Selected Issues for Trusts, Estates, and Their ... · 28/07/1997  · 3. Availability of §179 expensing 2 C. Administrative stance 2 1. Estates 2 2. Trusts 3 D. Judicial

cpenow.com / [email protected] Copyright © 2016 Surgent McCoy CPE, LLC -- ITEB/16/01 7-11

personal expenses through the trust. Mr. Crockett also filed fraudulent individual, trust, and corporate tax returns on the family’s behalf. The tax loss from the fraudulent scheme was estimated to be greater than $950,000. U.S. Attorney Paul Warner declared, “It is only fair to the vast majority of law abiding citizens who pay an honest and fair tax that we aggressively pursue those who do not.”51

I. Works cited • Burg, Brad, “Can You Spot a Phony Trust,” Medical Economics, v. 76, January 1999,

137-141. • CCH, “Taxpayers Hiding Assets Offshore Have Until August 31 to Disclose,” Federal Tax

Course Letter, March 2011, Volume 25 Issue 3, 1-5. • “Cincinnati Area Man Pleads Guilty to Aegis-Related Tax Crimes,” PR Newswire, January

2011. • Goffe, Wendy S., “An Introduction to Lesser-Known But Useful Trusts – Part 2,” Estate

Planning, v. 37, August 2010, 3-11. • “IRS Trust Sham Notice,” The Tax Prophet, 1997. Available from

http://www.taxprophet.com/archives/ hot/IRSNotice.htm. • Jooste, Ruan, “Ain’t it a Sham(e),” Finance Week, May 2005, 98-99. • Kessler, James, “What is and What’s Not a Sham,” Drafting Trusts and Will Trusts, 1999,

125. • Lambert, Ian T.G., “The KPMG Report,” Trusts & Trustees, v. 7, November 2000, 8-11. • Martindale-Hubbell, “Trust Shams and IRS Investigations,” Lawyers.com, 2010. Available

from http:// trusts-estates.lawyers.com/Trust-Shams-and-IRS-Investigations.html. • Novack, Janet, “If It Sounds Too Good,” Forbes, 2000, 178-179. • Rydalch, Melodie, “Sham Trust Promoter Sentenced,” Department of Justice, August

2004. Available from http://www.justice.gov/tax/usaopress/2004/ txdv04bcrocketttaxsent.pdf.

• Sommers, Robert L., “Sham Trust Analysis,” The Tax Prophet, 2000. Available from http://www. taxprophet.com/archives/pubs/sham_Trust_Analysis.htm.

• Vernazza, J. Ben, “The Independent Protector in Overseas Trusts,” Trusts & Trustees, v. 6, February 2000, 22-27.

• Whig, Cecil, “Bethany Beach Woman Gets Two Years in Prison for Using Sham Trust Accounts to Evade Taxes,” TownNews.com, December 2010. Available from http://www.cecilwhig.com/ business/article_0da250ae-026d-11e0-ac70-001cc4c03286.html.

II. Special needs trust

A. Introduction A special needs trust (SNT) is a discretionary trust providing for a disabled person’s needs beyond the support from the government. According to §1382(c)(a)(3)(A), a disabled individual is not able “to engage in any substantial gainful activity by reason of any medically determinable physical or mental impairment which can be expected to result in death or which has lasted or can be expected to last for a continuous period of not less than twelve months.” Furthermore, a child under the age of eighteen is considered disabled if he or she suffers from any “medically determinable physical or mental impairment.”52

51 Rydalch, Melodie, “Sham Trust Promoter Sentenced,” Department of Justice, August 2004. 52 §1382(c)(a)(3)(A).

Page 140: Advanced Selected Issues for Trusts, Estates, and Their ... · 28/07/1997  · 3. Availability of §179 expensing 2 C. Administrative stance 2 1. Estates 2 2. Trusts 3 D. Judicial

cpenow.com / [email protected] Copyright © 2016 Surgent McCoy CPE, LLC -- ITEB/16/01 7-12

There is a growing necessity for special needs trusts to maintain the quality of life of disabled persons caused by the increase of two independent factors: (i) the number of disabled persons; and (ii) the cost of health care. First, the percentage of disabled persons increased from 18.1 percent of the U.S. population in 2002 (or 51.2 million persons) to 18.7 percent of the population in 2005 (i.e., 54.4 million persons) as displayed in Table 1.53 Second, the national healthcare expenditures (NHEs) have increased linearly from $1.86 trillion in 2004 to $2.34 trillion in 2008 and projected to increase to $4.48 trillion in 2019 as indicated in Figure 1.54 In addition to hospitalization and other medical costs, annual nursing home cost for a disabled person can be about $32,120 in Shreveport, Louisiana to $120,815 in Anchorage, Alaska.55

Table 1 2002 2005 Americans with Disabilities 51.2 million 54.4 million Percent of U.S. Population 18.1% 18.7%

Figure 1

There are various government programs earmarked for the support of a disabled individual, including Supplemental Security Income (SSI), Social Security Disability Income (SSDI), Medicare, and Medicaid. Generally speaking, these public benefits can be classified into two categories – non-needs based and needs based. Non-needs based benefits, such as SSDI and Medicare, could be received by disabled people regardless of the recipient’s level of income or wealth. SSI and Medicaid are needs based, which means the recipient has a certain restriction in his income in order to be eligible for the benefits. Sometimes these restrictions are not even well understood by the state Department of Human Services (DHS). For example, a seemingly properly established SNT was created for Diana Harding’s $60,000 inheritance; however, the Spokane, Washington Housing Authority included the SNT for determination of state benefits. To avoid potential homelessness and undue hardship for Diana, the superior court judge

53 U.S. Census Bureau – Americans with disabilities: 2005, Issued December 2008. 54 U.S. Department of Health: Centers for Medicaid and Medicare: National Health Expenditure Data 2010. 55 CNNMoney.com, Long-term Care Cost Calculator.

0

500

1000

1500

2000

2500

3000

3500

4000

4500

5000

2004 2005 2006 2007 2008 2009 2010 2011 2012 2013 2014 2015 2016 2017 2018 2019

NHEs Linear (NHEs)

Page 141: Advanced Selected Issues for Trusts, Estates, and Their ... · 28/07/1997  · 3. Availability of §179 expensing 2 C. Administrative stance 2 1. Estates 2 2. Trusts 3 D. Judicial

cpenow.com / [email protected] Copyright © 2016 Surgent McCoy CPE, LLC -- ITEB/16/01 7-13

who approved her SNT issued a temporary injunction barring the housing authority from raising her subsidized rent until the court could decide her case.56 Currently, federal SSI and Medicaid regulations each require that the recipient possess less than $2,000 of assets for an individual or $3,000 for married couples. Furthermore, since SSI and Medicaid are state programs, their asset limitations for disabled persons vary from one jurisdiction to the next. Examining the relevant information for Florida, Louisiana, Texas, Arizona, and California in Table 1B reveals that the SSI limitations are generally consistent with the federal standard; however, the Medicaid limitations lack consistency.57

Table 1B State SSI Asset Limitation Medicaid Asset Limitation Florida $2,000 $5,000 for an individual and $6,000 for an eligible

couple. Louisiana $2,000 ($3,000 for two) $5,000 for an individual Texas $2,000 ($3,000 if married) $5,000 for an individual Arizona $2,000 ($3,000 for a couple) $2,000 for an individual California $2,000 ($3,000 for a couple) Twice the SSI allowance (e.g., $4,000 for an

individual and $6,000 for a couple). Since the purpose of the special needs trust is to supplement, not to replace the above mentioned public benefits, it is extremely important for the SNT to not jeopardize the public benefits a disabled individual could receive; the SNT should provide the recipient with additional resources not already provided. Based upon this rationale, there are usually a series of restrictions accompanying SNTs, which are listed in Table 2. Furthermore, a common list of acceptable SNT distributions is delineated in Table 3.58

Table 2 Special Need Trust Restrictions The distribution must be fully discretionary by the trustee according to the beneficiary’s needs. The distribution cannot be made in cash to the beneficiary; instead, payment must be made from the trust to disburse the beneficiary’s expenses. Food and shelter expenditures, which is in the scope of SSI, and medical expenses covered under the Medicare program are also forbidden expenditures.

The remainder of this chapter proceeds as follows. First, the chapter classifies the types of special needs trusts and provides general considerations for taxation. Thereafter, specific income tax, gift tax, and estate tax strategies are considered for self-settled and third party SNTs, respectively. Last, some conclusions are offered for tax planning to maximize the quality of life for our disabled loved ones.

56 Graman, Kevin. 2010. “Woman Sues over Housing Assistance”. McClatchy – Tribune Business News. Washington: Oct

12, 2010. 57 U.S. Department of Health Resources and Services Administration. 2011. 58 Zook, R., Crown, H., and Drucker M. 2008. Utilization of Special Needs Trusts for disabled loved ones. Journal of

Business & Economics Research (May 2008): 47-54.

Page 142: Advanced Selected Issues for Trusts, Estates, and Their ... · 28/07/1997  · 3. Availability of §179 expensing 2 C. Administrative stance 2 1. Estates 2 2. Trusts 3 D. Judicial

cpenow.com / [email protected] Copyright © 2016 Surgent McCoy CPE, LLC -- ITEB/16/01 7-14

Table 3 Common Distributions from SNT Recreational or vocational activities Hobbies and vacations Education and training Professional services for the beneficiary Payment for the acquisition and maintenance of a pet or service animal Purchase of a home by the trust with rent payments made by the beneficiary Health and life insurance premiums Medical services such as dental care, eyeglasses, or personal care attendant Purchase of a car, including maintenance

B. Classification With the general concept of SNT outlined, this chapter will now provide a detailed description of the classifications of SNT. In general, there are two types of SNTs: (i) self-settled SNT; and (ii) third party SNT. In addition, self-settled SNT can be further divided into two categories – d(4)(A) SNT and d(4)(C) SNT.

1. d(4)(A) SNT

First, a d(4)(A) SNT is authorized by 42 U.S.C. §1396p(d)(4)(A) and establishes a trust for the sole benefit of an individual, under the age of 65 with a disability, by the individual’s parent, grandparent, legal guardian, or the court. This trust is funded with the disabled beneficiary’s own assets. By definition, the grantor of the d(4)(A) SNT is also the beneficiary of the trust; hence, most likely creating a grantor trust as described below. The initial fund of the trust is typically from the settlement of the lawsuit resulting from the actions that created the beneficiary’s disability or an outright inheritance from a third party. For example, Margaret Flood intended to create a will leaving her disabled daughters an inheritance set up as a SNT. However, after the prolong illness of one of her daughters and her own sickness, she died before doing so which caused her daughters to lose their state benefits.59 One disadvantage of such trusts is that upon the death of the disabled beneficiary, the SNT must pay back to the state the remaining balance of the trust up to the amount the beneficiary received from the Medicaid program.60 An interesting application of a state Medicaid lien occurs in Norwest Bank North

Dakota v. David Doth.61 The United States Court of Appeals for the Eighth Circuit, affirmed the district court’s judgment that a Medicaid lien by the Minnesota Department of Human Services (DHS) for prior benefit paid be satisfied against the proceeds of the personal injury award “before” the remaining funds are placed in a d(4)(A) SNT rather than upon the death of the disabled beneficiary. Even though the SNT may be valid under §1396p(d)(4)(A), the disabled beneficiary may lose state benefits because of inappropriate distributions. The Tenth Circuit, in Steffan Hobbs v. Marsha

Zenderman,62 affirmed the district court’s judgment that the state is not required to exempt the SNT from asset calculations for Medicaid eligibility because Steffan Hobbs’ (disabled beneficiary) trust was not being used for his sole benefit. The trust paid Steffan’s mother $2,479 a month for caregiver services rendered and purchased a fifty percent interest in his parent’s home.

59 Estate of Margaret Flood in Matter of the Trusts to be Established, Docket No. A-1643-09T1 (N.J. Super. 12-29-2010). 60 Krooks, B. A., and Hook, A. H. 2009. Special Needs Trusts: The basics, the benefits, and the burdens. ALI-ABA Estate

Planning Course Materials Journal (December, 2009): 17-56. 61 159 F.3d 328. 62 579 F.3d 1171.

Page 143: Advanced Selected Issues for Trusts, Estates, and Their ... · 28/07/1997  · 3. Availability of §179 expensing 2 C. Administrative stance 2 1. Estates 2 2. Trusts 3 D. Judicial

cpenow.com / [email protected] Copyright © 2016 Surgent McCoy CPE, LLC -- ITEB/16/01 7-15

2. d(4)(C) SNT

Second, a d(4)(C) SNT, also referred to as a pooled SNT, is funded with the disabled beneficiary’s personal assets, and can be established by the disabled individual’s parents, grandparents, legal guardians, a court, or the disabled person himself or herself. This trust is managed by a non-profit organization which pools different individuals’ SNTs together for investment purposes and maintains each discrete account for payment purposes. The d(4)(C) SNT does not require an age limit for the disabled person making this option available to a larger class of disabled persons. Similar to the d(4)(A) SNT, the d(4)(C) SNT requires the disabled individual’s estate to repay the state Medicaid program up to the amount the beneficiary received during the life of the beneficiary. Amounts in the pooled SNT after all disbursements, including the state Medicaid program, are retained by the non-profit trustee. Even if the non-profit trustee donates the retained funds, the organization receives goodwill, publicity, and exposure to potential customers which may create a conflict of interest on the part of the non-profit trustee. To ward off any ill-effects, a trust protector may provide effective oversight of the non-profit trustee including some trust administrative processes. However, the benefits of such oversight must outweigh the ensuing costs.63

3. Third party SNT

A third party SNT is established and funded by someone other than the disabled beneficiary. The trust could be an inter vivos trust created during the third party’s lifetime or a testamentary trust established at the death of the third party. Comparable to a self-settled SNT, a third party SNT must be distributed under the sole discretion of the trustee. However, a third party SNT is not liable to repay the state for Medicaid assistance received and has no age limit for the disabled beneficiary.64 If a spouse would like to create a third party SNT for the benefit of a surviving spouse who is receiving Medicaid benefits, the SNT must be a testamentary trust contained in a will. Assets contained in an inter vivos SNT created by a spouse will be considered an available resource for public benefits purposes. If a spouse who is receiving Medicaid benefits is disinherited by the other spouse, the Medicaid recipient spouse is required to pursue augmented estate rights. Otherwise, the Medicaid recipient spouse will be deemed to have made a gift of the elective share, resulting in a period of ineligibility for Medicaid, or will be ineligible for Medicaid for failing to pursue an available resource. Table 4 summarizes the classification of the different types of SNTs.

Table 4

Criterion Self Settled SNT Third Party SNT d(4)(A) SNT d(4)(C) SNT

Establisher Parent, grandparent, guardian, or court

Parent, grandparent, guardian, court, or beneficiary

Any third party

Age Limit 65 None None Funder Beneficiary Any third party Distribution Discretionary Discretionary

Type of Trust Irrevocable inter vivos Irrevocable Inter vivos or Testamentary

Payback Provision Yes None

63 Berg, Rebecca and Scott Solkoff. 2010. The Importance of Trust Protectors for Pooled Special Needs Trusts. The

ElderLaw Report 22 (1): 1-6. 64 Zook, R., Crown, H., and Drucker M. 2008. Utilization of Special Needs Trusts for disabled loved ones. Journal of

Business & Economics Research (May 2008): 47-54.

Page 144: Advanced Selected Issues for Trusts, Estates, and Their ... · 28/07/1997  · 3. Availability of §179 expensing 2 C. Administrative stance 2 1. Estates 2 2. Trusts 3 D. Judicial

cpenow.com / [email protected] Copyright © 2016 Surgent McCoy CPE, LLC -- ITEB/16/01 7-16

C. Taxation of SNT After gaining a general idea of special needs trust, it is essential to explore the tax ramifications of different types of SNTs. Although SNTs are taxed comparatively to individuals, SNTs do not receive a personal exemption except if the SNT is a qualified disability trust. The income resulting from a SNT will be taxed to the grantor, the trust, or the beneficiary. Some further SNT taxation topics to be discussed include the planning techniques of establishing such a trust and clarifying some of the concepts related to the taxation of trusts. When planning for the taxation of an SNT, the first step to follow is to acquire an Employer Identification Number (EIN). This can be achieved by filing form SS-4 with the IRS. The next step requires identifying the grantor of the trust, which will vary according to the type of trust established. The final step in the process entails using the trust’s EIN as opposed to the grantor’s social security number when establishing SNT accounts.

1. The concept of income

When assessing the taxation of an SNT, one item that must be gauged is the income concept for tax purposes and for the eligibility of government assistance because it is different. According to SSA Pub. No. 68-0501120, if an individual does not have the legal authority to revoke the trust or direct the use of the trust assets for his or her own support and maintenance, the trust principal is not the individual's resource for SSI purposes. This statement creates a disconnect between income for tax purposes and assets that are counted as available for SSI claimants. Certain items that are reported as income for the disabled individual are not characterized as income for SSI qualification purposes, and vice versa. For example, a distribution from a SNT for a disabled beneficiary’s tuition payment is not considered income of the beneficiary for income tax purposes, neither is it includible in the beneficiary’s assets for SSI eligibility purposes. However, if distributions from the SNT are made for shelter, food, clothing, or medical expenses, though they are not counted as income for income tax purposes, they are considered income for purposes of determining SSI and Medicaid eligibility. Recall lodging, food, clothing, and medical care are under the purview of SSI and Medicaid; therefore, any distribution from the SNT for these items would reduce the assistance the beneficiary could receive from SSI.

2. Grantor trust vs. non-grantor trust

Another aspect that is extremely important to the taxation of trusts is to differentiate whether the trust is a grantor trust or a non-grantor trust, as the different classification of trust will result in different tax treatment for the trust income. A grantor trust is one in which the grantor (i.e., creator) of the trust has certain interests and related powers in the trust. The grantor would be considered the owner of the trust, and thereby, the trust income as a result would be taxed to the grantor in his or her individual tax return instead of being taxed to the trust. The powers retained by the grantor causing a trust to be a grantor trust are described in Table 5. As demonstrated by the regulation, if any of the conditions are met in Table 5, the trust income will not be taxed separately in the trust tax return, but will be taxed directly to the grantor in his or her individual income tax return. On the other hand, the assets of a non-grantor trust are not treated as owned by a person other than the trust. Thus, the trust is considered a separate taxable entity. The taxation of individual income versus trust income has its differences. One distinction is the trust tax schedule has a more condensed tax bracket (Table 6) as compared with that of a single individual (Table 7). A comparison of the trust and individual tax schedule reveals that a trust reaches its maximum marginal tax rate at a much lower net income than does an individual taxpayer. For example, in 2016, a trust reached its maximum marginal tax rate of 39.6% at only $12,400 of taxable income. In contrast, a single individual taxpayer would have to earn over $415,050 to be taxed at the same marginal tax rate.

Page 145: Advanced Selected Issues for Trusts, Estates, and Their ... · 28/07/1997  · 3. Availability of §179 expensing 2 C. Administrative stance 2 1. Estates 2 2. Trusts 3 D. Judicial

cpenow.com / [email protected] Copyright © 2016 Surgent McCoy CPE, LLC -- ITEB/16/01 7-17

Therefore, with income less than the floor amount of the individual’s 39.6% tax bracket (i.e., $415,050 for 2016), the 2016 tax liability may be reduced significantly by having the trust income taxed at the individual rate rather than the tax rate for trusts.

Table 5

Code Section Description

I.R.C. §673 Reversionary Interest: Grantor retains a reversionary interest in the trust that is worth 5 percent or more of the value of that interest.

I.R.C. §674 Power to Control Beneficial Enjoyment: Grantor could determine the beneficial enjoyment of the corpus or the income of the trust without the approval or consent of any adverse party.

I.R.C. §675 Administrative Power: The power to reacquire the trust corpus by substituting other property of an equivalent value.

I.R.C. §676 Power to Revoke: The grantor has the power to revoke or amend a trust.

I.R.C. §677 Income for Benefit of Grantor: Income of the trust is or could be distributed to the grantor or the grantor’s spouse without the approval or consent of any adverse party or in the discretion of the grantor or a non-adverse party.

Another variation is the exemption amount and deduction amount on individual tax may be greater than what is available for trusts. When the trust income is taxed to the individual, the taxpayer may utilize his or her personal exemption of $4,050 and standard deduction of $6,300 in 2016. The trust income is only eligible for an exemption of either $300 (simple trust) or $100 (complex trust) in addition to a certain amount of distributable net income (DNI) as a deduction ceiling if taxed to the trust. Pertaining to the DNI as a deduction ceiling required under I.R.C. §651(a), the amount is highly contingent upon the amount of the distribution. If the trust does not make a significant distribution to the beneficiary, it would have a small amount of DNI as a deduction ceiling. If this is the case, the amount that is excluded from taxable income under trust tax may be less than the exclusion amount under the individual income tax. Having addressed the basic tax issues, we will delve into the tax ramifications of the different types of SNTs.

Table 6

2016 Trust Tax Rates

Taxable Income Over But Not Over Pay + % on Excess Of Amount Over

$0 $2,550 $0.00 15 $0 $2,550 $5,950 $382.50 25 $2,550 $5,950 $9,050 $1,232.50 28 $5,950 $9,050 $12,400 $2,100.50 33 $9,050 $12,400 $3,206.00 39.6 $12,400

Page 146: Advanced Selected Issues for Trusts, Estates, and Their ... · 28/07/1997  · 3. Availability of §179 expensing 2 C. Administrative stance 2 1. Estates 2 2. Trusts 3 D. Judicial

cpenow.com / [email protected] Copyright © 2016 Surgent McCoy CPE, LLC -- ITEB/16/01 7-18

Table 7 2016 Single Individual Tax Rates Taxable Income Over But Not Over Pay + % on Excess Of Amount Over

$0 $9,275 $0.00 10 $0 $9,275 $37,650 $927.50 15 $9,275 $37,650 $91,150 $5,183.75 25 $37,650 $91,150 $190,150 $18,558.75 28 $91,150 $190,150 $413,350 $46,278.75 33 $190,150 $413,350 $415,050 $119,934.75 35 $413,350 $415,050 $120,529.75 39.6 $415,050.00

D. Self-settled SNT taxation

1. Income tax

Self-settled SNTs are generally grantor trusts according to the standards set forth for the qualification of grantor trust. First, considering I.R.C. §677 (Income for Benefit of Grantor), if the income of the trust is used for the benefit of the grantor or the grantor’s spouse or may be so used in the discretion of the grantor or a non-adverse party, or both, the trust is a grantor trust. For a self-settled SNT, since the grantor is also the sole beneficiary of the trust, and the grantor-beneficiary would get the benefit of the income of the trust at the discretion of the trustee. Therefore, a self-settled SNT is a grantor trust unless its trustee is an adverse party. According to this standard of grantor trust, we could say most self-settled SNTs are grantor trusts. Second, the fraction of self-settled SNTs which have trustees that are adverse parties to the beneficiary may also qualify as grantor trusts. In I.R.C. §675 concerning Administrative Powers, if the grantor has the power to reacquire the trust corpus by substituting other property of an equivalent value, the trust is a grantor trust. Since in a self-settled SNT, it is the grantor-beneficiary that funds the assets, he or she certainly has the power to reacquire the trust assets using other property with an equivalent value. Hence, this power of administration standard would mandate that all the remaining self-settled trusts receive the treatment of grantor trusts.65 Additionally, self-settled SNTs usually do not have reversionary interests or powers to revoke granted to the grantor-beneficiary. Although, giving the grantor-beneficiary more than five percent reversionary interest or revoking power would certainly qualify the trust as a grantor trust. These types of provisions could also cause the trust assets to be considered available for SSI eligibility purposes. Therefore, revocable trusts or reversion interest clauses are generally not used for special needs planning purposes.66 Pursuant to the aforementioned analysis that self-settled SNTs are grantor trusts, the income of the trust would be taxable to the grantor-beneficiary rather than to the trust. In normal circumstances, it will bring several advantages. First, as noted previously, the tax liability concerning trust income is usually greatly reduced when taxed at the individual rate rather than the trust rate. Second, since trust income is taxable to the grantor and the taxes are paid out of the beneficiary’s own assets rather than the trust assets, trust assets would grow income-tax-free at a faster rate without being depleted by income taxes. Third,

65 Fleming, R. B. 2008. “Taxation of Special Needs Trusts”. 1-12. 66 Sandoval, D. 2008. Taxation of Special Needs Trusts. Special Needs Alliance: 1-8.

Page 147: Advanced Selected Issues for Trusts, Estates, and Their ... · 28/07/1997  · 3. Availability of §179 expensing 2 C. Administrative stance 2 1. Estates 2 2. Trusts 3 D. Judicial

cpenow.com / [email protected] Copyright © 2016 Surgent McCoy CPE, LLC -- ITEB/16/01 7-19

although trust income is taxed at the beneficiary’s personal rate, there is usually not much additional tax liability because the disabled beneficiary is usually in a low tax bracket and has considerable medical expenses that can be used as itemized deductions to offset the additional income. Last, even if there may be additional tax liabilities for the grantor-beneficiary concerning the trust income, taxes paid for trust income by the beneficiary would reduce the beneficiary’s gross estate upon death.

2. Gift tax

Since the disabled grantor-beneficiary does not intend to make a gift from the trust, then the trust is intended to be for the sole benefit of the disabled grantor. Of course, the trustee has full discretion to use the income and principal of the trust for the benefit of the grantor-beneficiary. As a result, this is not a complete gift by the grantor-beneficiary for gift tax purposes and no gift tax is generated concerning the self-settled SNT.

3. Estate tax

With regard to estate tax consequences of a self-settled SNT, the grantor-beneficiary has not made a gift and the grantor-beneficiary has a retained life estate in the trust. Hence, I.R.C. §2036 would pull the value of the trust assets back into the grantor’s estate for estate tax purposes. Although the disabled beneficiary needs to pay the estate tax for a self-settled SNT, there is usually no estate tax due in most cases because the disabled person usually has a very limited amount of an estate. Also, the estate liability is reduced to zero by the estate tax credit of credit of $2,125,800 which is equivalent to an exemption amount of $5,450,000. Even if that is not enough, the claim by the state Department of Health Services (i.e., Medicaid program) for recovery against the remaining value of the self-settled SNT is also deductible from the beneficiary’s gross estate. Therefore, the estate tax is not normally a concern to the disabled beneficiary of a self-settled SNT.

E. Third party SNT taxation

1. Income tax

If a third party SNT is created during the third party grantor’s life time, it could either be a grantor trust or non-grantor trust depending on the language governing the trust instrument. If drafted as a grantor trust, the income from the SNT would be taxable to the third party grantor with taxes payable by such grantor, leaving the trust assets growing tax free. If drafted as a non-grantor trust, or the SNT is created upon the grantor’s death which makes it automatically qualify it as a non-grantor trust, the third party SNT would report its income on its own 1041 return and pay the trust income tax from trust income or principal. As discussed previously, if classified as a non-grantor trust, the third party SNT would be subject to the compressed tax rates for trusts which generally lead to a larger amount of tax liability than that for a grantor trust. Therefore, for tax purposes, it is usually more favorable to have a third party SNT drafted as a grantor trust. If circumstances do not allow the trust to meet grantor trust status, the trust could also be categorized as a Qualified Disability Trust (QDT) to minimize tax liability. A QDT must be a trust described in 42 U.S.C.§1396p(c)(2)(B)(iv) (i.e., it cannot be a grantor trust). A QDT is an irrevocable trust established for the sole benefit of a disabled person under the age of 65. For a QDT, the trust is entitled to an exemption equal to the personal exemption for an individual $4,050 in 2016 rather than a trust exemption of $300 or $100. Therefore, a non-grantor third party SNT which qualifies as a QDT may enjoy the advantage of reducing the taxable income of the trust by the larger individual personal exemption. The beneficiary also has the right to claim a personal exemption on his or her individual income tax return in addition to the

Page 148: Advanced Selected Issues for Trusts, Estates, and Their ... · 28/07/1997  · 3. Availability of §179 expensing 2 C. Administrative stance 2 1. Estates 2 2. Trusts 3 D. Judicial

cpenow.com / [email protected] Copyright © 2016 Surgent McCoy CPE, LLC -- ITEB/16/01 7-20

trust’s Form 1041. Therefore, the trustee can distribute to the beneficiary an amount of income equal to the personal exemption free of tax.

2. Gift and estate taxes

If the third party SNT allows the grantor certain powers such as a retained life estate, reversionary interests, or revocable transfers, no complete gift is made and there is no gift tax ramifications for the grantor. However, upon the death of the grantor, the value of the trust would be includible in the grantor’s gross estate according to Code §§2036, 2037 and 2038. When the third party grantor does not have a retained life estate in the trust and a complete gift is made through the third party SNT, the third party grantor is liable for gift taxes but no estate taxes. In this instance, because the gift is made through a trust and is not considered a present gift, no gift tax exclusion ($14,000 per donee per annum for 2016) is allowed. In addition, the technique of a Crummey power could not be used for the SNT which would entitle the beneficiary to a withdrawal right of a certain amount from the trust for a short period of time. This power is usually designed as a lapsing power to the beneficiary. Because the beneficiary is considered to have had the opportunity to withdraw the certain amount, it would render the gift subject to the amount as a current gift instead of a future one, and the annual gift exclusion can be utilized. Normally, a Crummey power is a common technique that is often built into the design of a life insurance trust. However, for a third party SNT, the Crummey power given to the disabled beneficiary to withdraw a certain amount from the trust could easily be counted by the state Department of Health Service as an available asset of the beneficiary for SSI purposes. Therefore, the Crummey power language should be avoided for a third party SNT.

F. Conclusions Since the number of disabled persons and health care costs are increasing, maximizing the quality of life for disabled individuals through the creation of special needs trusts will continue to be a concern for many Americans. A SNT is established to supplement state SSI and Medicaid assistance, not replace it. Remember, the case of the Estate of Margaret Flood that emphasizes the importance of actually creating a will to establish SNTs for disabled loved ones. Of course, the mere “intent” to set up such documents is not sufficient according to the judgment of the Superior Court of New Jersey and resulted in the loss of state benefits. Because a court approved self-settled SNT is established, it does not preclude a state from including the trust’s assets in the calculation for Medicaid eligibility when the trust is not used for the sole benefit of the disabled beneficiary as indicated in Hobbs v. Zenderman. Additionally, to ward off any ill-effects of potential conflicts of interest from a self-settled “pooled” SNT, a trust protector may provide effective oversight of the non-profit trustee. Also, neither “pooled” SNTs nor third party SNTs have age restrictions which make these trusts available to a broader range of disabled persons. Self-settled SNTs are generally classified as grantor trusts and their potential tax advantages include: (1) lower individual tax rates and (2) higher exemption amounts. Because SNTs are for the sole benefit of the disabled beneficiary, no complete gifts are made, and therefore, no gift tax is assessed. The remainder interest in the SNT is included in the disabled person’s estate; however, the estates are usually small and exempt from estate tax. In addition to grantor trusts, third party SNTs may be non-grantor trust and will most often be classified as a complex trust which files Form 1041. Complex trusts have $100 exclusions (rather than $300

Page 149: Advanced Selected Issues for Trusts, Estates, and Their ... · 28/07/1997  · 3. Availability of §179 expensing 2 C. Administrative stance 2 1. Estates 2 2. Trusts 3 D. Judicial

cpenow.com / [email protected] Copyright © 2016 Surgent McCoy CPE, LLC -- ITEB/16/01 7-21

exclusions available for simple trusts) and generally have limited distribution deductions. Distributions made by third parties to the disabled beneficiary are gifts of a future interest which do not qualify for the annual gift tax exclusion. Also, Crummey power language should be avoided for a third party SNT because this power may cause distributions to be counted towards state eligibility for SSI and Medicaid benefits. Special needs trusts have varying tax, legal, and personal consequences which must be considered for each individual when trying to maximize his or her quality of life.

G. Works cited • Berg, Rebecca and Scott Solkoff. 2010. The Importance of Trust Protectors for Pooled

Special Needs Trusts. The ElderLaw Report 22 (1): 1-6. • CNNMoney.com, Long-term Care Cost Calculator,

http://cgi.money.cnn.com/tools/elder_care/elder_care_cost_finder.html (last visited March 27, 2011) (calculations based on factors entered into online calculators).

• Estate of Margaret Flood in Matter of the Trusts to be Established, Docket No. A-1643-09T1 (N.J. Super. 12-29-2010).

• Fleming, R. B. 2008. “Taxation of Special Needs Trusts”. 1-12. http://www.elder-law.com/files/SNTtaxation.pdf.

• Graman, Kevin. 2010. “Woman Sues over Housing Assistance”. McClatchy – Tribune Business News. Washington: Oct 12, 2010.

• I.R.C. §§673-677. • I.R.C. §1382(c)(a)(3)(A). • I.R.C. §§2036-2038. • Krooks, B. A., and Hook, A. H. 2009. Special Needs Trusts: The basics, the benefits, and

the burdens. ALI-ABA Estate Planning Course Materials Journal (December, 2009): 17-56.

• Norwest Bank North Dakota v. David Doth. 159 F.3d 328; 1998 U.S. App. LEXIS 24535; 58 Soc. Sec. Rep. Service 783.

• Sandoval, D. 2008. Taxation of Special Needs Trusts. Special Needs Alliance: 1-8. • Steffan Hobbs v. Marsha Zenderman, 579 F.3d 1171; 2009 U.S. App. LEXIS 19932. • U.S. Census Bureau – Americans with disabilities: 2005, Issued December 2008,

www.census.gov/prod/2008pubs/p70-117.pdf. • U.S. Department of Health: Centers for Medicaid and Medicare: National Health

Expenditure Data 2010, https://www.cms.gov/NationalHealthExpendData/ 25_NHE_Fact_Sheet.asp.

• U.S. Department of Health Resources and Services Administration. 2011. Florida http://www.hrsa.gov/reimbursement/states/florida-eligibility.htm; Louisiana http://www.hrsa.gov/reimbursement/states/louisiana-eligibility.htm; Texas http://www.hrsa.gov/reimbursement/states/texas-eligibility.htm; Arizona http://www.hrsa.gov/reimbursement/states/arizona-eligibility.htm; California http://www.hrsa.gov/reimbursement/states/florida-eligibility.htm.

• Zook, R., Crown, H., and Drucker M. 2008. Utilization of Special Needs Trusts for disabled loved ones. Journal of Business & Economics Research (May 2008): 47-54.

Page 150: Advanced Selected Issues for Trusts, Estates, and Their ... · 28/07/1997  · 3. Availability of §179 expensing 2 C. Administrative stance 2 1. Estates 2 2. Trusts 3 D. Judicial
Page 151: Advanced Selected Issues for Trusts, Estates, and Their ... · 28/07/1997  · 3. Availability of §179 expensing 2 C. Administrative stance 2 1. Estates 2 2. Trusts 3 D. Judicial

cpenow.com / [email protected] Copyright © 2016 Surgent McCoy CPE, LLC -- ITEB/16/01 8-i

Rate Schedules and Computations

I. Income-tax rate schedules for use by estates and nongrantor trusts 1 II. Initial guidance on new valuation factors 1 III. May 1, 1989 and thereafter 4

Page 152: Advanced Selected Issues for Trusts, Estates, and Their ... · 28/07/1997  · 3. Availability of §179 expensing 2 C. Administrative stance 2 1. Estates 2 2. Trusts 3 D. Judicial
Page 153: Advanced Selected Issues for Trusts, Estates, and Their ... · 28/07/1997  · 3. Availability of §179 expensing 2 C. Administrative stance 2 1. Estates 2 2. Trusts 3 D. Judicial

cpenow.com / [email protected] Copyright © 2016 Surgent McCoy CPE, LLC -- ITEB/16/01 8-1

Rate Schedules and Computations I. Income-tax rate schedules for use by estates and

nongrantor trusts

For tax years beginning in 2016: If taxable income is: The tax is: $0 - $2,550 15 percent of taxable income $2,550 - $5,950 $382.50 + 25 percent of excess over $2,550 $5,950 - $9,050 $1,232.50 + 28 percent of excess over $5,950 $9,050 - $$12,400 $2,100.50 + 33 percent of excess over $9,050 $12,400 - and above $3,206.00 + 39.6 percent of excess over $12,400 For tax years beginning in 2015: If taxable income is: The tax is: $0 - $2,500 15 percent of taxable income $2,500 - $5,900 $375.00 + 25 percent of excess over $2,500 $5,900 - $9,050 $1,225.00 + 28 percent of excess over $5,900 $9,050 - $12,300 $2,107.00 + 33 percent of excess over $9,050 $12,300 - and above $3,179.50 + 39.6 percent of excess over $12,300

II. Initial guidance on new valuation factors

Example 1: Computation of a remainder interest -- Assume that a donor makes a gift of an interest in a trust for an eight-year term certain after April 30, 1989, and needs to value the remainder interest. Assume also that 120 percent of the applicable federal midterm rate for the month is 10.8 percent. The applicable factor for valuing a remainder interest based on a term of years may be determined by use of the following mathematical formula:

Remainder factor = 1 (1 + I)t

Where I equals 120 percent of the applicable federal midterm interest rate under §7520 and the exponential power t equals the number of years in the term.

Based upon interest at 10.8 percent per annum, the present worth of $1 due at the end of eight years is $0.440232.

Example 2: Computation of an income interest -- Assume the same facts as in Example

1, except that the donor needs to value the income interest. The valuation factor for a term certain interest may be computed by using this formula:

Income Factor = 1.000000 - Remainder Factor

Thus, if 120 percent of the applicable federal midterm rate for the month is 10.8 percent and the remainder factor as computed in Example 1 above is 0.440232, the income factor for an income interest in a trust for a term of eight years is 1.000000 minus 0.440232, which equals 0.559768.

Page 154: Advanced Selected Issues for Trusts, Estates, and Their ... · 28/07/1997  · 3. Availability of §179 expensing 2 C. Administrative stance 2 1. Estates 2 2. Trusts 3 D. Judicial

cpenow.com / [email protected] Copyright © 2016 Surgent McCoy CPE, LLC -- ITEB/16/01 8-2

Example 3: Computation of an annuity interest -- Assume that a donor makes a gift of an annuity interest for an eight-year term certain after April 30, 1989. Assume also that 120 percent of the applicable federal midterm rate for the month is 10.8 percent. The valuation factor for a term certain annuity interest may be computed by using this formula:

Annuity Factor = Income Factor I

Where I equals the applicable interest rate under §7520.

Thus, if 120 percent of the applicable federal midterm rate for the month is 10.8 percent and the income factor, as computed in Example 2 above, is 0.559768; the valuation factor for an annuity interest for a term of eight years is 0.559768 divided by 10.8 percent, which equals 5.1830.

Page 155: Advanced Selected Issues for Trusts, Estates, and Their ... · 28/07/1997  · 3. Availability of §179 expensing 2 C. Administrative stance 2 1. Estates 2 2. Trusts 3 D. Judicial

cpenow.com / [email protected] Copyright © 2016 Surgent McCoy CPE, LLC -- ITEB/16/01 8-3

Table B -- Term-Certain Remainder Factors -- Interest Rate

Years 10.2% 10.4% 10.6% 10.8% 11.0% 1 .907441 .905797 .904159 .902527 .900901 2 .823449 .820468 .817504 .814555 .811622 3 .747232 .743178 .739153 .735158 .731191 4 .678069 .673168 .668312 .663500 .658731 5 .615307 .609754 .604261 .598827 .593451 6 .558355 .552313 .546348 .540457 .534641 7 .506674 .500284 .493985 .487777 .481658 8 .459777 .453156 .446641 .440232 .433926 9 .417221 .410467 .403835 .397322 .390925

10 .378603 .371800 .365131 .358593 .352184 11 .343560 .336775 .330137 .323640 .317283 12 .311760 .305050 .298496 .292094 .285841 13 .282904 .276313 .269888 .263623 .257514 14 .256719 .250284 .244022 .237927 .231995 15 .232957 .226706 .220634 .214735 .209004 16 .211395 .205350 .199489 .193804 .188292 17 .191828 .186005 .180369 .174914 .169633 18 .174073 .168483 .163083 .157864 .152822 19 .157961 .152612 .147453 .142477 .137678 20 .143340 .138235 .133321 .128589 .124034 21 .130073 .125213 .120543 .1l6055 .111742 22 .118033 .113418 .108990 .104743 .100669 23 .107108 .102733 .098544 .094533 .090693 24 .097195 .093056 .089100 .085319 .081705 25 .088198 .084289 .080560 .077003 .073608 26 .080035 .076349 .072839 .069497 .066314 27 .072627 .069157 .065858 .062723 .059742 28 .065905 .062642 .059547 .056609 .053822 29 .059804 .056741 .053840 .051091 .048488 30 .054269 .051396 .048680 .046111 .043683 31 .049246 .046554 .044014 .041617 .039354 32 .044688 .042169 .039796 .037560 .035454 33 .040552 .038196 .035982 .033899 .031940 34 .036798 .034598 .032533 .030595 .028775 35 .033392 .031339 .029415 .027613 .025924 36 .030301 .028387 .026596 .024921 .023355 37 .027497 .025712 .024047 .022492 .021040 38 .024952 .023290 .021742 .020300 .018955 39 .022642 .021096 .019658 .018321 .017077 40 .020546 .019109 .017774 .016535 .015384 41 .018645 .017309 .016071 .014923 .013860 42 .016919 .015678 .014531 .013469 .012486 43 .015353 .014201 .013138 .012156 .011249 44 .013932 .012864 .011879 .010971 .010134 45 .012642 .011652 .010740 .009902 .009130 46 .011472 .010554 .009711 .008937 .008225 47 .010410 .009560 .008780 .008065 .007410 48 .009447 .008659 .007939 .007279 .006676 49 .008572 .007844 .007178 .006570 .006014 50 .007779 .007105 .006490 .005929 .005418

Page 156: Advanced Selected Issues for Trusts, Estates, and Their ... · 28/07/1997  · 3. Availability of §179 expensing 2 C. Administrative stance 2 1. Estates 2 2. Trusts 3 D. Judicial

cpenow.com / [email protected] Copyright © 2016 Surgent McCoy CPE, LLC -- ITEB/16/01 8-4

Example 4: On April 1, 1989, John Henry, grantor, transferred assets valued at $400,000 to a charitable remainder annuity trust. Martha Jones, a female, age 58, was named as the life beneficiary, and Grady Hospital as the remainderman. A payment rate of five percent was chosen, and Martha is to receive a distribution of $20,000 each year. The factor for the present value of the annuity is $153,644 (7.6822 X $20,000) from Table A in the regulations. The amount of the charitable-contribution deduction on John Henry’s income tax return (subject to the charitable-contribution deduction limitations) is $246,356 ($400,000 - 153,644). This amount is fully deductible on John Henry’s gift tax return. Naturally, the $246,356 could be an estate-tax deduction if this transfer was testamentary rather than inter vivos.

III. May 1, 1989 and thereafter

Remainder factor -- Table S Assume Age 58 at January’s AFR of 3.0 Percent

Remainder factor = .54034 Income factor = 1 - r Income factor = .45966 Annuity factor = income factor i Annuity factor = .45966 .030 Annuity factor = 15.3220 Annuity factor = 15.3220 x $20,000 Annuity factor = $306,644 Charitable contribution = $400,000 – 306,644 Charitable contribution = $93,356

Note:

The AFR is updated monthly. As of March 2016, the §7520 rate was 1.8 percent.

Page 157: Advanced Selected Issues for Trusts, Estates, and Their ... · 28/07/1997  · 3. Availability of §179 expensing 2 C. Administrative stance 2 1. Estates 2 2. Trusts 3 D. Judicial

cpenow.com / [email protected] Copyright © 2016 Surgent McCoy CPE, LLC -- ITEB/16/01 8-5

Table S Based on Life Table 80CNSMT Single-Life Remainder Factors Applicable after April 30, 1989

Interest Rate

Age 2.2% 2.4% 2.6% 2.8% 3.0% 3.2% 3.4% 3.6% 3.8% 4.0% 10 .25237 .22467 .20034 .17894 .16010 .14350 .12884 .11590 .10445 .09431 11 .25780 .22993 .20541 .18381 .16476 .14795 .13308 .11993 .10827 .09793 12 .26335 .23533 .21062 .18883 .16957 .15254 .13746 .12410 .11224 .10170 13 .26899 .24081 .21593 .19394 .17448 .15724 .14195 .12838 .11631 .10558 14 .27467 .24634 .22129 .19911 .17944 .16200 .14650 .13271 .12044 .10950 15 .28037 .25190 .22667 .20430 .18444 .16678 .15107 .13708 .12460 .11346 16 .28608 .25747 .23208 .20952 .18945 .17159 .15567 .14146 .12878 .11743 17 .29182 .26307 .23751 .21477 .19450 .17643 .16030 .14588 .13299 .12144 18 .29760 .26872 .24300 .22007 .19961 .18133 .16499 .15036 .13725 .12550 19 .30347 .27446 .24858 .22547 .20482 .18634 .16978 .15494 .14162 .12966 20 .30944 .28031 .25428 .23100 .21015 .19147 .17471 .15965 .14612 .13395 21 .31552 .28628 .26011 .23666 .21563 .19674 .17978 .16452 .15078 .13839 22 .32171 .29237 .26606 .24244 .22123 .20215 .18498 .16951 .15556 .14297 23 .32802 .29859 .27214 .24837 .22698 .20771 .19034 .17467 .16051 .14771 24 .33446 .30494 .27838 .25446 .23290 .21344 .19588 .18000 .16563 .15263 25 .34105 .31145 .28478 .26071 .23899 .21935 .20159 .18552 .17095 .15774 26 .34777 .31811 .29133 .26713 .24525 .22544 .20749 .19122 .17645 .16304 27 .35465 .32493 .29806 .27374 .25171 .23173 .21360 .19714 .18217 .16855 28 .36166 .33190 .30495 .28051 .25833 .23820 .21989 .20324 .18808 .17426 29 .36880 .33902 .31199 .28744 .26514 .24484 .22637 .20953 .19418 .18017 30 .37607 .34627 .31918 .29454 .27210 .25166 .23302 .21601 .20048 .18627 31 .38347 .35366 .32652 .30179 .27924 .25866 .23986 .22268 .20696 .19256 32 .39099 .36119 .33400 .30920 .28654 .26583 .24688 .22953 .21363 .19906 33 .39865 .36886 .34166 .31678 .29402 .27319 .25410 .23659 .22052 .20577 34 .40644 .37668 .34946 .32453 .30169 .28074 .26151 .24385 .22762 .21268 35 .41437 .38466 .35744 .33247 .30955 .28849 .26914 .25133 .23494 .21984 36 .42243 .39279 .36558 .34057 .31759 .29644 .27697 .25902 .24248 .22722 37 .43064 .40107 .37388 .34886 .32582 .30458 .28500 .26694 .25025 .23483 38 .43898 .40951 .38236 .35733 .33425 .31295 .29327 .27508 .25826 .24269 39 .44747 .41810 .39101 .36599 .34288 .32152 .30175 .28346 .26651 .25080 40 .45611 .42686 .39984 .37485 .35172 .33031 .31048 .29208 .27502 .25918 41 .46490 .43580 .40886 .38391 .36079 .33935 .31945 .30097 .28380 .26783 42 .47384 .44490 .41806 .39317 .37007 .34860 .32866 .31011 .29284 .27676 43 .48292 .45416 .42745 .40263 .37956 .35809 .33811 .31950 .30215 .28597 44 .49214 .46357 .43700 .41228 .38925 .36780 .34779 .32914 .31172 .29545 45 .50148 .47312 .44670 .42208 .39912 .37769 .35769 .33899 .32152 .30517 46 .51092 .48279 .45655 .43205 .40917 .38778 .36779 .34907 .33155 .31514 47 .52045 .49257 .46652 .44216 .41937 .39804 .37806 .35934 .34179 .32533 48 .53008 .50246 .47661 .45241 .42973 .40847 .38853 .36982 .35225 .33574 49 .53979 .51245 .48682 .46279 .44024 .41907 .39918 .38049 .36292 .34638 50 .54959 .52255 .49716 .47332 .45091 .42985 .41003 .39137 .37381 .35726 51 .55947 .53274 .50762 .48398 .46173 .44079 .42105 .40245 .38491 .36836 52 .56941 .54301 .51815 .49474 .47267 .45186 .43223 .41369 .39619 .37965 53 .57939 .55334 .52877 .50559 .48372 .46306 .44354 .42509 .40764 .39113 54 .58941 .56372 .53946 .51653 .49486 .47437 .45498 .43663 .41925 .40278 55 .59946 .57415 .55021 .52755 .50611 .48580 .46656 .44832 .43102 .41461 56 .60955 .58462 .56102 .53865 .51745 .49734 .47826 .46015 .44295 .42661 57 .61965 .59514 .57188 .54981 .52887 .50897 .49007 .47210 .45502 .43877 58 .62975 .60566 .58277 .56101 .54034 .52067 .50196 .48416 .46720 .45105 59 .63982 .61616 .59364 .57222 .55182 .53240 .51390 .49627 .47945 .46342

Page 158: Advanced Selected Issues for Trusts, Estates, and Their ... · 28/07/1997  · 3. Availability of §179 expensing 2 C. Administrative stance 2 1. Estates 2 2. Trusts 3 D. Judicial
Page 159: Advanced Selected Issues for Trusts, Estates, and Their ... · 28/07/1997  · 3. Availability of §179 expensing 2 C. Administrative stance 2 1. Estates 2 2. Trusts 3 D. Judicial

cpenow.com / [email protected] Copyright © 2016 Surgent McCoy CPE, LLC -- ITEB/16/01 9-1

Problems and Tax Forms

1. Problem 1: Complex trust – Calendar year (2015)

Dividends (Nonqualifying) $50,000 Passive Income 20,000 Exempt Interest 30,000 Capital Loss (L/T) 15,000 Trustee Commission (I) 5,000 Trustee Commission (P) 10,000 Paid to Beneficiary 50,000

• Determine FAI. • Determine DNI as an income ceiling. • Determine DNI as a deduction ceiling. • Determine the trust’s taxable income. • Determine DNI as a qualitative yardstick.

2. Problem 2: Fiduciary income tax return problem

On June 30, 1985, Jim Smith was killed in a plane crash. Under his will, a complex trust was established with ABC, Inc. as trustees. Its address is 100 Long Street, Atlanta, GA 30303. The trust’s Federal Identification Number is 58-1325067. Under the provisions of the will, 50 percent of the trust income is distributed to his widow Mary, 20 percent to the United Way, and 10 percent to his son Bill. The remainder may either be accumulated or distributed to his brother Bob, at the sole discretion of ABC. In his trust, Jim left an apartment building valued in the estate at $800,000 with a 40-year life and no salvage value. The trust uses straight-line depreciation. Jim purchased the building on January 1, 1985. The annual rents for the building for 2015 totaled $50,000. It paid $5,000 in interest, $10,000 for maintenance, and $5,000 for taxes. Also, in the trust, Jim left 10,000 shares of Trust preferred stock that paid (nonqualifying) dividends this year of $30,000. The trust sold 1,000 shares of Penn Mutual that were valued on date of death at $15,000. The trust sold the stock on October 19, 2015 for $5,000. The trust also had 500 shares of stock in AT&T with a basis of $10,000. The stock was sold on June 1, 2015 for $30,000. The trust instrument allocates all net capital gains to corpus. The trust has $100,000 face-value 10-percent bonds from the city of Atlanta, which paid $10,000 this year. The maturity date of the bonds is May 1, 2019. The trust also owns a $100,000, 10-percent CD from Trust Company Bank that paid interest of $10,000 this year. The trust instrument provides for allocation of unattached or floating expenses on a pro rata basis to the tax-exempt interest. All remaining expenses are to be allocated to the rental income. If this account is exhausted, the expenses may be allocated to any other income in the sole discretion of the trustee. Trustee’s commissions are to be allocated 50 percent to principal and 50 percent to income.

Page 160: Advanced Selected Issues for Trusts, Estates, and Their ... · 28/07/1997  · 3. Availability of §179 expensing 2 C. Administrative stance 2 1. Estates 2 2. Trusts 3 D. Judicial

cpenow.com / [email protected] Copyright © 2016 Surgent McCoy CPE, LLC -- ITEB/16/01 9-2

The total income scenario can be summarized as follows:

Rents $50,000 Nonqualifying Dividends

30,000

Taxable interest 10,000 Tax-exempt interest 10,000 Long-term capital gain 20,000 Long-term capital loss 10,000 Rental expenses 20,000 Trustee’s commissions 10,000 Depreciation 20,000

The trust instrument also requires a depreciation reserve of $5,000 per year.* The trust is to terminate at the death of Mary with all corpus and accumulated income to be distributed to Bob. This year the trust accumulated the excess income. This trust uses the cash receipts and disbursements method of tax accounting, and estimated taxes of $4,000 were paid on a quarterly basis. Additional information:

1. Mary Smith 2 Stone Alley Dunwoody, GA 34343 SSN: 234-34-1234

2. Bill Smith

Love Lane Apt. 15 Doraville, GA 36786 SSN: 678-89-5678

3. United Way

10 Lucky Street Atlanta, GA 31010

4. ABC, Inc.

EIN: 57-2478561 The following items are required in sequence.

1. Trust accounting income (ignore estimated taxes); 2. DNI as an income ceiling; 3. DNI as a deduction ceiling; 4. Trust taxable income; 5. DNI as a qualitative yardstick; 6. K-1 allocations; 7. Completed U.S. Fiduciary Income Tax Return (Form 1041); 8. Completed Schedule I; 9. Completed Schedule D; 10. Completed Schedule E; 11. Completed Form 4562; 12. Completed K-1 for Mary Smith; and 13. Completed K-1 for Bill Smith.

* The remaining depreciation follows income.

Page 161: Advanced Selected Issues for Trusts, Estates, and Their ... · 28/07/1997  · 3. Availability of §179 expensing 2 C. Administrative stance 2 1. Estates 2 2. Trusts 3 D. Judicial

cpenow.com / [email protected] Copyright © 2016 Surgent McCoy CPE, LLC -- ITEB/16/01 9-3

Page 162: Advanced Selected Issues for Trusts, Estates, and Their ... · 28/07/1997  · 3. Availability of §179 expensing 2 C. Administrative stance 2 1. Estates 2 2. Trusts 3 D. Judicial

cpenow.com / [email protected] Copyright © 2016 Surgent McCoy CPE, LLC -- ITEB/16/01 9-4

Page 163: Advanced Selected Issues for Trusts, Estates, and Their ... · 28/07/1997  · 3. Availability of §179 expensing 2 C. Administrative stance 2 1. Estates 2 2. Trusts 3 D. Judicial

cpenow.com / [email protected] Copyright © 2016 Surgent McCoy CPE, LLC -- ITEB/16/01 9-5

Page 164: Advanced Selected Issues for Trusts, Estates, and Their ... · 28/07/1997  · 3. Availability of §179 expensing 2 C. Administrative stance 2 1. Estates 2 2. Trusts 3 D. Judicial

cpenow.com / [email protected] Copyright © 2016 Surgent McCoy CPE, LLC -- ITEB/16/01 9-6

Page 165: Advanced Selected Issues for Trusts, Estates, and Their ... · 28/07/1997  · 3. Availability of §179 expensing 2 C. Administrative stance 2 1. Estates 2 2. Trusts 3 D. Judicial

cpenow.com / [email protected] Copyright © 2016 Surgent McCoy CPE, LLC -- ITEB/16/01 9-7

Page 166: Advanced Selected Issues for Trusts, Estates, and Their ... · 28/07/1997  · 3. Availability of §179 expensing 2 C. Administrative stance 2 1. Estates 2 2. Trusts 3 D. Judicial

cpenow.com / [email protected] Copyright © 2016 Surgent McCoy CPE, LLC -- ITEB/16/01 9-8

Page 167: Advanced Selected Issues for Trusts, Estates, and Their ... · 28/07/1997  · 3. Availability of §179 expensing 2 C. Administrative stance 2 1. Estates 2 2. Trusts 3 D. Judicial

cpenow.com / [email protected] Copyright © 2016 Surgent McCoy CPE, LLC -- ITEB/16/01 9-9

Page 168: Advanced Selected Issues for Trusts, Estates, and Their ... · 28/07/1997  · 3. Availability of §179 expensing 2 C. Administrative stance 2 1. Estates 2 2. Trusts 3 D. Judicial

cpenow.com / [email protected] Copyright © 2016 Surgent McCoy CPE, LLC -- ITEB/16/01 9-10

Page 169: Advanced Selected Issues for Trusts, Estates, and Their ... · 28/07/1997  · 3. Availability of §179 expensing 2 C. Administrative stance 2 1. Estates 2 2. Trusts 3 D. Judicial

cpenow.com / [email protected] Copyright © 2016 Surgent McCoy CPE, LLC -- ITEB/16/01 9-11

Page 170: Advanced Selected Issues for Trusts, Estates, and Their ... · 28/07/1997  · 3. Availability of §179 expensing 2 C. Administrative stance 2 1. Estates 2 2. Trusts 3 D. Judicial

cpenow.com / [email protected] Copyright © 2016 Surgent McCoy CPE, LLC -- ITEB/16/01 9-12

Page 171: Advanced Selected Issues for Trusts, Estates, and Their ... · 28/07/1997  · 3. Availability of §179 expensing 2 C. Administrative stance 2 1. Estates 2 2. Trusts 3 D. Judicial

cpenow.com / [email protected] Copyright © 2016 Surgent McCoy CPE, LLC -- ITEB/16/01 9-13

Page 172: Advanced Selected Issues for Trusts, Estates, and Their ... · 28/07/1997  · 3. Availability of §179 expensing 2 C. Administrative stance 2 1. Estates 2 2. Trusts 3 D. Judicial

cpenow.com / [email protected] Copyright © 2016 Surgent McCoy CPE, LLC -- ITEB/16/01 9-14

Page 173: Advanced Selected Issues for Trusts, Estates, and Their ... · 28/07/1997  · 3. Availability of §179 expensing 2 C. Administrative stance 2 1. Estates 2 2. Trusts 3 D. Judicial

cpenow.com / [email protected] Copyright © 2016 Surgent McCoy CPE, LLC -- ITEB/16/01 9-15

Page 174: Advanced Selected Issues for Trusts, Estates, and Their ... · 28/07/1997  · 3. Availability of §179 expensing 2 C. Administrative stance 2 1. Estates 2 2. Trusts 3 D. Judicial

cpenow.com / [email protected] Copyright © 2016 Surgent McCoy CPE, LLC -- ITEB/16/01 9-16

Page 175: Advanced Selected Issues for Trusts, Estates, and Their ... · 28/07/1997  · 3. Availability of §179 expensing 2 C. Administrative stance 2 1. Estates 2 2. Trusts 3 D. Judicial

cpenow.com / [email protected] Copyright © 2016 Surgent McCoy CPE, LLC -- ITEB/16/01 9-17

Page 176: Advanced Selected Issues for Trusts, Estates, and Their ... · 28/07/1997  · 3. Availability of §179 expensing 2 C. Administrative stance 2 1. Estates 2 2. Trusts 3 D. Judicial
Page 177: Advanced Selected Issues for Trusts, Estates, and Their ... · 28/07/1997  · 3. Availability of §179 expensing 2 C. Administrative stance 2 1. Estates 2 2. Trusts 3 D. Judicial

cpenow.com / [email protected] Copyright © 2016 Surgent McCoy CPE, LLC -- ITEB/16/01 10-1

Solutions

1. Problem 1: Solution

Page 178: Advanced Selected Issues for Trusts, Estates, and Their ... · 28/07/1997  · 3. Availability of §179 expensing 2 C. Administrative stance 2 1. Estates 2 2. Trusts 3 D. Judicial

cpenow.com / [email protected] Copyright © 2016 Surgent McCoy CPE, LLC -- ITEB/16/01 10-2

2. Problem 2: Solution

Page 179: Advanced Selected Issues for Trusts, Estates, and Their ... · 28/07/1997  · 3. Availability of §179 expensing 2 C. Administrative stance 2 1. Estates 2 2. Trusts 3 D. Judicial

cpenow.com / [email protected] Copyright © 2016 Surgent McCoy CPE, LLC -- ITEB/16/01 10-3

Page 180: Advanced Selected Issues for Trusts, Estates, and Their ... · 28/07/1997  · 3. Availability of §179 expensing 2 C. Administrative stance 2 1. Estates 2 2. Trusts 3 D. Judicial

cpenow.com / [email protected] Copyright © 2016 Surgent McCoy CPE, LLC -- ITEB/16/01 10-4

Page 181: Advanced Selected Issues for Trusts, Estates, and Their ... · 28/07/1997  · 3. Availability of §179 expensing 2 C. Administrative stance 2 1. Estates 2 2. Trusts 3 D. Judicial

cpenow.com / [email protected] Copyright © 2016 Surgent McCoy CPE, LLC -- ITEB/16/01 10-5

Page 182: Advanced Selected Issues for Trusts, Estates, and Their ... · 28/07/1997  · 3. Availability of §179 expensing 2 C. Administrative stance 2 1. Estates 2 2. Trusts 3 D. Judicial

cpenow.com / [email protected] Copyright © 2016 Surgent McCoy CPE, LLC -- ITEB/16/01 10-6

Page 183: Advanced Selected Issues for Trusts, Estates, and Their ... · 28/07/1997  · 3. Availability of §179 expensing 2 C. Administrative stance 2 1. Estates 2 2. Trusts 3 D. Judicial

cpenow.com / [email protected] Copyright © 2016 Surgent McCoy CPE, LLC -- ITEB/16/01 10-7

Page 184: Advanced Selected Issues for Trusts, Estates, and Their ... · 28/07/1997  · 3. Availability of §179 expensing 2 C. Administrative stance 2 1. Estates 2 2. Trusts 3 D. Judicial

cpenow.com / [email protected] Copyright © 2016 Surgent McCoy CPE, LLC -- ITEB/16/01 10-8

Page 185: Advanced Selected Issues for Trusts, Estates, and Their ... · 28/07/1997  · 3. Availability of §179 expensing 2 C. Administrative stance 2 1. Estates 2 2. Trusts 3 D. Judicial

cpenow.com / [email protected] Copyright © 2016 Surgent McCoy CPE, LLC -- ITEB/16/01 10-9

Page 186: Advanced Selected Issues for Trusts, Estates, and Their ... · 28/07/1997  · 3. Availability of §179 expensing 2 C. Administrative stance 2 1. Estates 2 2. Trusts 3 D. Judicial

cpenow.com / [email protected] Copyright © 2016 Surgent McCoy CPE, LLC -- ITEB/16/01 10-10

Page 187: Advanced Selected Issues for Trusts, Estates, and Their ... · 28/07/1997  · 3. Availability of §179 expensing 2 C. Administrative stance 2 1. Estates 2 2. Trusts 3 D. Judicial

cpenow.com / [email protected] Copyright © 2016 Surgent McCoy CPE, LLC -- ITEB/16/01 10-11

Page 188: Advanced Selected Issues for Trusts, Estates, and Their ... · 28/07/1997  · 3. Availability of §179 expensing 2 C. Administrative stance 2 1. Estates 2 2. Trusts 3 D. Judicial

cpenow.com / [email protected] Copyright © 2016 Surgent McCoy CPE, LLC -- ITEB/16/01 10-12

Page 189: Advanced Selected Issues for Trusts, Estates, and Their ... · 28/07/1997  · 3. Availability of §179 expensing 2 C. Administrative stance 2 1. Estates 2 2. Trusts 3 D. Judicial

cpenow.com / [email protected] Copyright © 2016 Surgent McCoy CPE, LLC -- ITEB/16/01 10-13

Page 190: Advanced Selected Issues for Trusts, Estates, and Their ... · 28/07/1997  · 3. Availability of §179 expensing 2 C. Administrative stance 2 1. Estates 2 2. Trusts 3 D. Judicial

cpenow.com / [email protected] Copyright © 2016 Surgent McCoy CPE, LLC -- ITEB/16/01 10-14

Page 191: Advanced Selected Issues for Trusts, Estates, and Their ... · 28/07/1997  · 3. Availability of §179 expensing 2 C. Administrative stance 2 1. Estates 2 2. Trusts 3 D. Judicial

cpenow.com / [email protected] Copyright © 2016 Surgent McCoy CPE, LLC -- ITEB/16/01 10-15

Page 192: Advanced Selected Issues for Trusts, Estates, and Their ... · 28/07/1997  · 3. Availability of §179 expensing 2 C. Administrative stance 2 1. Estates 2 2. Trusts 3 D. Judicial

cpenow.com / [email protected] Copyright © 2016 Surgent McCoy CPE, LLC -- ITEB/16/01 10-16

Page 193: Advanced Selected Issues for Trusts, Estates, and Their ... · 28/07/1997  · 3. Availability of §179 expensing 2 C. Administrative stance 2 1. Estates 2 2. Trusts 3 D. Judicial

cpenow.com / [email protected] Copyright © 2016 Surgent McCoy CPE, LLC -- ITEB/16/01 10-17

Page 194: Advanced Selected Issues for Trusts, Estates, and Their ... · 28/07/1997  · 3. Availability of §179 expensing 2 C. Administrative stance 2 1. Estates 2 2. Trusts 3 D. Judicial

cpenow.com / [email protected] Copyright © 2016 Surgent McCoy CPE, LLC -- ITEB/16/01 10-18

JIM SMITH TRUST TAX-EXEMPT INTEREST ALLOCATION

12/31/15

Form 1041, Page 2, Other Information, Line 1 - Allocation of Attributable Expenses:

For the tax year 2015, the Jim Smith Trust received tax-exempt interest of $10,000. The following is an allocation of the expenses attributable:

Tax-Exempt Interest Received $ 10,000 Less: Commissions Allocable ($ 1,000) (10,000/100,000 x 10,000) Charitable Contribution Allocable ($ 1,400) (10,000/100,000 x 14,000) Tax-Exempt Interest as Adjusted $ 7,600

($10,000 represents the total amount of tax-exempt interest received; $100,000 represents the gross amount of accounting income; $10,000 represents the total amount of trustee’s fees; and $14,000 represents the total amount of the charitable contribution.)